You are on page 1of 127

Answer to Question No.

1- Page 127

What the question setter might have expected from this question from examinee:

• Calculation Salary Tax of a non Nepalese person

What the student should know before answering this question:

• Is Mr. Wooster a resident? The method of tax calculation differs for resident and non-resident
• Knowledge of Sec. 8 (what are the amounts to be included in Income), Sec. 22 (basis of
accounting) Sec. 27 & Rule 13 (Quantification of benefits), Sec. 97 (person not required to submit
income tax return), and Sec. 6 (Amounts to be included in assessable income)

What the examiner tested?

• Application of Resident concept- Mr. Wooster resides in Nepal for 7 months during IY 2071/72
• Application of Sec. 8- Some concepts of students regarding treatment of Leave Encashment,
compensation from previous employer, etc.
• Application of Sec. 27 & Rule 13- Vehicle and Accommodation Facility

Basis for answer:

• Mr. Wooster is resident


• An assumption is required as to whether he shall opt for couple or individual assessment. We
assume it individual
• Students should prepare a statement of assessable income from business, statement of taxable
income, balance taxable income and tax liability so as to ensure whether his tax liability for the
year is Rs. 265,000 or not. In this note, a simple statement is prepared; not in examination style:

Particulars Amounts Reason for inclusion or What does it tested?


non inclusion

Basic Salary 350,000 As per Sec. 8 for 7 months Understanding of Sec. 8 (2)

Uniform Allowance 7,000 One- off Sec. 8, every It tested Sec. 8 (2), and the
allowance form part of concept of Uniform that is
employment income- exempted from inclusion in
remember Sec. 21, an Income as per Sec. 21 at once.
uniform that is wearable We have assumed the uniform
strictly in workplace is can be worn outside workplace
considered deductible and as well.
is not included in
employee’s income

Technical Allowance 420,000 As it is allowance as per Understanding of Sec. 8 (2)-


Sec. 8 (2) Allowances to be included in
Income

Flat rent of Mr. Wooster 7,000 2% of Basic Salary drawn Application and understanding
by him is included in of Sec. 27 & Rule 13. This
paid by employer income as per Sec. 27 & might confuse students
Rule 13, not the amount of
rent paid by employer

Telephone facility 3,500 Sec. 8 (2) & Sec. 27- Sec. The question is silent. We
8 (2) requires the facility to included the amount assuming
be included in income and that the facility is solely for
Sec. 27 describes the personal purpose. It tested
method of calculation of Sec. 27
such facility, which is
actual cost borne by
employer

Free Vehicle facility 1,750 0.5% of Basic Salary Application and understanding
drawn by him is included in of Sec. 27 & Rule 13.
income as per Sec. 27 &
Rule 13

Compensation to 100,000 Sec. 8 (2)- Treated as if Tested the understanding of


Previous Employer by Reimbursement of the provision
current employer on Personal Expenses
behalf of the employee

Advance Salary for 4/72 - We considered it to be It tested the basis of


and 5/72 advance against salary, not accounting- but we tricked
the salary advance using the concept of advance
against salary. Had we
considered amount to be
income received in advance,
we would have included it.

Leave pay 50,000 Sec. 8 (2) requires it to be It tested the difference between
included as it is paid during retirement payment (which is
the tenure of employment final withholding) and payment
during the tenure of
employment- Students are
often confused on this
treatment

Value of Accumulated - Remember Basis of Tested Sec. 22


leave provision Accounting, unless the
amount is received in cash-
it is not considered

Tuition Fee paid by 40,000 Sec. 8 (2)- Treated as if Tested understanding of Sec. 8
Employer Reimbursement of (2)- Reimbursement of
Personal Expenses Personal expenses

Air fare of Mr. Wooster 100,000 Sec. 8 (2)- Treated as if Tested understanding of Sec. 8
from USA to Nepal (not Reimbursement of (2)- Remember Sec. 21 that
for official purpose) Personal Expenses strictly make this type of
payment not deductible for the
employer unless that are
included in employee’s income

Canteen Bill paid by 40,000 Same as previous Same as previous


employer

Drivers Salary of - As it is part of vehicle Tested presence of mind and


vehicle facility- it is not included in the understanding of vehicle
income facility- the different items that
are covered by 0.5% of basic
Salary & Grade

Total Assessable 1,019,250 Sum of above


Income

Taxable Income 1,019,250 There are no deductions

Balance Taxable 1,119,250 There are no reductions


Income

Tax liability 209,812.50

1st Rs. 250,000 1%


2500

Next Rs. 100,000 15%


15,000

Balance Rs. 684,250


25% 167,312.5

We calculated the tax liability to be computed by the employer, which is Rs. 209,812.50; not Rs. 265,000.

Now:

Answer (a): No it is not correct. As shown in calculation above, the amount to be deducted by the
employer as per Sec. 87 of the Act is Rs. 209,812.50 instead of Rs. 265,000.

Answer (b): remember Sec. 97- List out who is not required to file tax return, out of those four- one is the
person mentioned in Sec. 4 (3) of the Act. In this case- Mr. Wooster has only one employer, have income
only from employment, he does not claim donation, contribution to ARF or MTC apart from that paid by
employer, and he is resident- all conditions satisfied, thus, not required to submit return.

Answer (C): Remember Sec. 6- A resident is required to pay tax on global income and a nonresident on
income having source in Nepal. In either case, Mr. Wooster is required to pay tax in Nepal, no matter
what his nationality is.
Answer to Question No. 1- Page 127

This question basically tests the students understanding on final withholding payments and the
calculation of income from investment. It also tests the student’s knowledge above the expenses that are
specifically deductible under Sec. 21 of the Act, i.e. amount incurred for collection of final withholding
payments. This is an understanding level question, and the question is complete in all respect. But there
is technical error in the question, as one cannot compute taxable income under any head. Taxable
Income means the total of assessable income under four different heads less two deductibles u/s 63 and
u/s 12 of the Act. It appears that the question setter is also not clear about the definition of taxable income
as per Sec. 5 of the Act. So the question to seek taxable income from investment is technically incorrect
as per the Act. But the students can assume that the examiner wants to examine the students’ ability to
calculate assessable income from investment.

Solution: This is not the perfect solution- just a synopsis of what student should have thought and written
for the correct answer. And it’s not in solution format as well.

What the question


Reason to include setter wanted to
Particulars: Inclusion
or not include test?

Amount to be included in Income (Sec.


9)

House rent income net of TDS - It is final withholding Whether the student
assuming that Mr. knows that house
Narayan does not rent is a final
conduct business of withholding payment
leasing or subleasing
of house
Bank interest income net of TDS- Nepal - This is final Whether the student
Bank Ltd. withholding- bank knows that house
interest received by rent is a final
natural person and withholding payment
not related to
operation of business
Natural resources payments net of TDS 200,000 Included as per Sec. WHT rate on natural
9 (2) and as its not resource payment,
final withholding. and knowledge of
Grossed up as it is gross up using
net of TDS. This arithmetical
payment attracts computation
15% WHT, means capability and the
170000 is 85% of students presence
actual payment, of mind that the
Gross payment is Rs. WHT is considered
200,000 as Advance Income
tax as per Sec. 93 of
the Act

Interest income net of TDS from ABC 2,500,000 This interest is not Conceptual clarity
Ltd. final withholding as it on the condition of
is not a resident bank interest considered
or FI, and the interest as final withholding.
is assumed not for However, there are
debenture or bonds. two options to
If we assume the students, whether or
interest is against not to include in
debenture and income writing a
bonds, and ABC Ltd. suitable assumption
is authorized to issue as explained in
such securities, it previous column. It
becomes final is included in this
withholding. WHT on solution.
such interest is 15% In case of inclusion
and thus 2125000 is it tests the rate of
grossed up to 100% WHT on such
interest and
students ability to
gross up such
payment

Compensation received from loss of last 25,000 Included as per Sec. Understanding of
year investment 9 & sec. 31 as the Sec. 31 and Sec. 9
amount is included (2)
when the
compensation is
actually received
Income from investment insurance net of - Final withholding as Understanding of
TDS- Rastriya Beema Sanstha Rastriya Beema Sec. 92
Sansthan is resident
insurance company
Gift related to investment income 50,000 Included as per Sec. Understanding of
9 (2)- gift received in Sec. 9 (2)
connection to
investment
Dividend income net of TDS from - Final withholding. Tested the
Sanima Bank Ltd. Dividend received understanding of
from resident Sec. 92 and Sec. 9
company is final (3)
withholding
A. Total Amount to be included in 2,775,000
Income
Deductions to arrive at Assessable
Income

Expenses related to collection of house - Not allowed as per Understanding of


rent Sec. 21, expense Sec. 21
related to collection
of Final withholding
payments
Expenses related to natural resources 8,500 Sec. 13- general Understanding of
expenses Sec. 13

Allowable depreciation allowance as per 5,500 Sec. 19- as it is said Students knowledge
the Act its allowable that depn is
allowable as per
Income Tax Act- if
student is confused
on it, its called silly
mistake in student’s
perception and
blunder in
examiners, that
might put negative
impression

Life insurance premium paid to Rastriya - As related to Final Understanding of


Beema Sanstha withholding and is Sec. 21
considered by
Beema Sansthan
while computing gain
on Investment
Insurance
B. Total deductible Expenses 14,000

Assessable Income from Investment (A- 2,761,000


B)
Less: Allowable Deductions Understanding of
a. Contribution to ARF - how taxable income
b. Donation paid to tax exempt 40,000 As per Sec. 12, lower is calculated and
organization of: how the donation is
a. Actual claimed for tax
b. 5% of ATI- purpose- i.e. Sec. 12
5% of
2,761,000
c. Rs. 100,000
Taxable Income 2,721,000 As the question says
there is no any
assessable income
from business or
employment
Tax Liability 242,100 Married, and the Understanding of
1st Rs. 300,000 Nil maximum tax rate for computation of tax
Balance Rs. 2,421,000 10% investment is 10%. liability as per
Schedule income in
There is no additional case the income is
surcharge of 40% on only from investment
tax above taxable - Marginal
income of Rs. 25 relief is
Lakhs as the allowed
maximum tax rate for - Tax rate is
investment is 10% as 10% at Max
explained by Sec. 1
(4) (Kha) of Schedule
1 which comes after
Sec. 1 (1) and Sec. 1
(2) of Schedule 1.
Advance tax paid (WHT considered as There is no net tax
advance tax) liability payable, the
Natural Resource payment 30,000 amount is refundable
Interest from ABC Co. 375,000

Answer to Question No. 1- Page 110

The question is much clear. This question aims to test the students’ ability to compute depreciation with
correct treatment of addition, disposal and correct application of depreciation rate. It also tests the
students understanding on the method of computation of depreciation rate for Pool E Asset and
application of rate for the first year. Altogether, the complete knowledge of depreciation calculation is
tested with an easier question.

At least the following notes are expected in answer:

a. Pool A to D is depreciated on WDV basis and Pool E on straight line basis.

Solution

Depreciation calculation of Pool A to D

Particulars Pool A Pool B Pool C Pool D

Opening Depreciable Base 3,000,000 500,000 1,500,000 700,000

Absorbed Additions:

Up to Poush (100% of Cost) 300,000

From Magh to Chaitra (2/3rd of Cost) 266,667

From Baisakh to Ashad (1/3rd of Cost)

Disposal Proceeds 50,000 175,000

Depreciable Basis 3,000,000 750,000 1,591,667 700,000

Rate of Depreciation 5% 25% 20% 15%

Calculate the Calculate the Calculate Calculate


value the value the value
value yourself yourself yourself yourself

Depreciation Calculation of Pool E

Rate of Depreciation- 100/5.5% (the rate is determined by dividing 100 by life of the intangible asset. The
life of intangible asset is to be determined at nearest half year- and normal mathematical logic is used to
determine nearest half year of 5 and half year in absence of clear instruction in Act, regulations and
Income Tax authority) Each intangible asset shall be depreciated individually.

Calculation:

Depn- 300,000 X 100/5.5*1/100= Calculate the value yourself


Chapter 2- Introduction to Income Tax Law of Nepal

Readers are expected to identify the answer of all other questions of this chapter from the book
referring pg. 421-423 except the one which is dealt below.

Question 14- Page 30

As per Sec. 2(AC), “Investment Insurance” means insurance of any of the following class:

a. Insurance where the specified event is the death of an individual who is the insured or an associated
person of the insured;
b. insurance where the specified event is an individual who is the insured or an associated person of the
insured sustaining personal injury or becoming incapacitated in a particular manner
Provided that: the insurance agreement is expressed to be in effect for at least five years or without
limit of time and is not terminable by the insurer before the expiry of five years except in special
circumstances prescribed in the insurance contract;
c. insurance under which an amount or series of amounts is to become payable to the insured in the
future;
d. Reinsurance of insurance as specified in (a), (b) or (c) above, and
e. Reinsurance of reinsurance as specified in (d) above

In view of the definition of investment insurance given by Income Tax Act as explained above, for any
policy to qualify as “investment insurance”, it shall satisfy any of the conditions. Since the term of
insurance contract is expressed to be in effect for more than 5 years, and can only be terminated in
special conditions as prescribed in the insurance contract, i.e. in case of failure to pay insurance
premium; the policy satisfies criteria (b) above- signifying the contract is an “investment Insurance
contract”.

Hence, policy obtained by Mr. Murli is an investment insurance contract.


Chapter 3- Residential Status

Question 1 (Page 34)

As per Article 4 of DTAA between India and Nepal, when an individual is resident of both contracting
states; then the residential status of the individual is determined as follows:

a. he shall be deemed to be a resident of the State in which he has a permanent home available to
him; if he has a permanent home available to him in both States, he shall be deemed to be a
resident of the State with which his personal and economic relations are closer (center of vital
interests);
b. if the State in which he has his center of vital interests cannot be determined, or if he has not a
permanent home available to him in either State, he shall be deemed to be a resident of the State
in which he has an habitual abode;
c. if he has an habitual abode in both States or in neither of them, he shall be deemed to be a
resident of the State of which he is a national;
d. if he is a national of both States or of neither of them, the competent authorities of the
Contracting States shall settle the question by mutual agreement.
e. Where by reason of the provisions of above, a person other than an individual is a resident of both
Contracting States, and then the competent authorities of the Contracting States shall determine that
the person is a resident of a Contracting State for the purposes of the Agreement by mutual
agreement.

The above conditions are applied in hierarchical order in order to determine the residential status, i.e.
condition (a) is applied first and when the status is finalized through condition (a), the other provisions are
not tested.

Here, Mr. Jharkhande is an Indian national and a businessperson in Mumbai. In this case, it can be
assumed that he has permanent home available in India. But in case he has no permanent home in India
or is available at both India and Nepal, his center of vital interests is in Mumbai (i.e. in India as he derives
his income from Mumbai). Therefore, Mr. Jharkhande is resident of India, not Nepal as per the provisions
of DTAA.
Chapter 4- Basis of Taxation

Question 1 (page 38)

Fact of the case:

In this case, Mr. Pradip Sharma has turnover from business not exceeding Rs. 20 Lakhs from where he
derives profit not exceeding Rs. 2 Lakhs.

Provision of the Act:

As per Sec. 4 (4), in case all the following conditions are satisfied; a person is not required to calculate
tax liability in detail and is required to pay tax as follows (u/s 1 (7) of Schedule 1 of the Act):

a. The person shall be a resident natural person,


b. The person shall derive income exclusively from business having source in Nepal,
c. The person shall not claim Medical Tax credit and advance tax arising out of withholding taxes,
d. The income (turnover) from business shall not exceed Rs. 20 Lakhs and the profit shall not
exceed Rs. 2 Lakhs during the Income Year,
e. The person shall elect this provision to be applied in writing, and
f. The person shall not be registered for VAT purpose.

The tax applicable is as follows:

a. In case of conduct of business in Metropolitan City/Submetro city- Rs. 5000


b. In case of conduct of business in Municipality- Rs. 2,500
c. In case of conduct of business in VDC- Rs. 1,500

Conclusion:

In this case Mr. Pradip is a natural person and is resident of Nepal as his habitual place of abode is
Birgunj, i.e. Nepal. Assuming he has no other income than business having source in Nepal and in case
he renounces medical tax credit and advance tax arising out of withholding taxes, and he wishes to pay
presumptive taxation; he shall pay tax of Rs. 5,000 merely declaring the revenue and profit for the year. In
forming the conclusion, it is assumed that Mr. Pradip is not registered for VAT purpose.

In case any of the above conditions would not be satisfied, he should have calculated tax as per
Schedule 1 of the Act, i.e. under progressive taxation system

In both the case, Mr. Pradip is required to file an income return.


Chapter 7- Tax Exempt Organizations & Exempted Amounts

SECTION “TAX EXEMPTED ORGANIZATIONS”

Question 1 (page 45)

Please refer to “Registration of Organization as TEO” Section of Page 43 and 44 of book.

Question 2 (page 45)

Please refer to “Income of TEO exempted from Tax” Section of Page of book.

Question 3 (page 45)

As per Sec. 10 (g), the following income of an Exempt Organization is exempted from income tax, and no
other:

1. Gift and Donation,


2. Any voluntary contribution received from any person by TEO, provided that:
a. The voluntary contribution is directly related to the objective of TEO, and
b. The person making such voluntary contribution does not obtain any consideration and does not expect
any consideration against such contribution
3. Amount accrued by Nepal Rastra Bank as per its objective (The objective of Nepal Rastra Bank is
prescribed in Section 4 of Nepal Rastra Bank Act, 2058)
4. Amount accrued by Securities Board of Nepal as per its objective (The objective of Securities Board of
Nepal is prescribed in Preamble of Securities Act, 2063)

In the given question, the trust formed by an ashram for old aged elders is TEO. Let us test the above
conditions to identify whether the income is exempted for tax purpose or not. The conditions of (3) and (4)
above are not tested as it is neither NRB nor SEBON.

a. Donation from Members: Exempted as it satisfies condition (1) above


b. The other income are:
• Sale of product
• Interest on Deposits
• Rent from Shop let out
• Ticket sale of cinema
• Profit from selling books to the public and students

The above incomes are neither donation nor gift. Similarly, the incomes do not satisfy conditions (2)
above as the incomes are not voluntary contribution at all since the contributor expects consideration
against any contribution for the income as above. Thus, these incomes are not exempted for tax
purpose.

c. As per Sec. 21 of the Act, any expenditure incurred for generating exempted amounts are not
deductible (refer page 90 of the book). Thus, let us test what expenditures are deductible to derive
taxable income of the Ashram.
• Expenditure for maintenance of inmates: it does not generate any income, i.e. not a business
expenses and does not satisfy all the conditions of Sec. 13
• Power & heating: it does not generate any income, i.e. not a business expenses and does not
satisfy all the conditions of Sec. 13
• Cleaning & Sanitation: it does not generate any income, i.e. not a business expenses and does
not satisfy all the conditions of Sec. 13
• Trustees Salary: it does not generate any income, i.e. not a business expenses and does not
satisfy all the conditions of Sec. 13
Chapter 7- Tax Exempt Organizations & Exempted Amounts

• Administrative Salary: as per additional information, Rs. 125K out of Rs. 250K is expenditure for
book shop, so the amount is deductible for tax purpose u/s 13 of the Act. It is assumed that the
expenditure is considered while calculating profit from selling of books to the public and students
as profit can only be derived after considering the expenses. Assuming the expenditure not
considered for calculation of profit seems illogical.
• Administrative Salary: Out of 75K expenditure, Rs. 50K is utilized towards the selling of books,
i.e. 50K is business expenditure and deductible u/s 13 of the Act. . It is assumed that the
expenditure is considered while calculating profit from selling of books to the public and students
as profit can only be derived after considering the expenses. Assuming the expenditure not
considered for calculation of profit seems illogical.

Calculation of Tax Liability- Considering above, the taxable income is:

Particulars Sec. Refer W. N. Amount


AMOUNT INCLUDED IN INCOME
Donation 10 1 -
Sale of Products 7 2 250,000
Interest on Deposits 92 3 -
Rent from Shop let outs 7 2 150,000
Ticket sales for Cinema 7 2 50,000
Selling of books to the public and students 7 2&4 60,000
A. TOTAL 510,000
Deductions
Expenses for Maintenance of Inmates 5 -
Power & heating 5 -
Cleaning & Sanitation 5 -
Trustees Salary 5 -
Administrative Salary 5 -
Administrative Expenses 5 -
B. Total Deductions -
ASSESSABLE INCOME FROM BUSINESS 510,000
TAXABLE INCOME 6 510,000
TAX LAIBILITY @ 25% 7 127,500
Working Notes:

1. Donation is an exempt income as per Sec. 10 (g) of Income Tax Act.


2. The treatments are explained in point (b) above.
3. Interest on deposit is assumed to be obtained from resident banks and financial institution, hence
considered final withholding as per Sec. 92. In case of alternative assumption, the amount is included
while calculating assessable income from business.
4. It is assumed that all deductions as per Income Tax Act is considered while arriving at profit from selling of
books to the public and students.
5. The treatments are explained in point (c) above
6. It is assumed that there no deductions u/s 12, 12A and 12B.
7. The trust is an entity as per the definition given in Sec. 2 of the Act, and attracting corporate tax rate of
25%.

Question 4 (page 45)

As per Sec. 10 (g) following amounts derived by an exempt organization (other than NRB & SEBON) is
exempted from corporate income tax:
Chapter 7- Tax Exempt Organizations & Exempted Amounts

1. Gift and Donation, or


2. Any voluntary contribution received from any person by TEO, provided that:
a. The voluntary contribution is directly related to the objective of TEO, and
b. The person making such voluntary contribution does not obtain any consideration and does not expect
any consideration against such contribution

In case of any revenue and activities that does not satisfy any of the above conditions, the gain from such
activities is taxable.

In the given case, an NGO (exempt organization) was awarded a contract under free competition to provide
computer education to street children.

Testing the conditions as above, the amount derived by the NGO is not gift or donation, so condition (1) is not
satisfied. Though the contribution received from INGO by the NGO is as per the objective of NGO (i.e.
objective clause of NGO includes an object as providing facilities and vocational training to street children),
which satisfied (a) of condition (2) above, but the person making contribution (i.e. INGO) expects consideration
(i.e. providing computer education to street children) against such contribution, thereby condition 2 (b) is not
satisfied.

In view of above, both the above conditions are not satisfied, hence, any gain from such activity of an exempt
organization is taxable.

Readers should note that in case any contribution is received by exempt organization for any specific activities
specified in its object clause, it is considered as expectation of consideration by the contributor.

Question 5 (page 45)

As per Sec. 10 (g) following amounts derived by an exempt organization (other than NRB & SEBON) is
exempted from corporate income tax:

1. Gift and Donation, or


2. Any voluntary contribution received from any person by TEO, provided that:
a. The voluntary contribution is directly related to the objective of TEO, and
b. The person making such voluntary contribution does not obtain any consideration and does not expect
any consideration against such contribution

Let us test the above conditions for the given income to determine whether the amounts are exempted or not:

Income Exemption Status Reason


Membership fee Exempted As it satisfies condition (2) above. Since it’s a contribution from
member who does not expect any consideration in return. The
amount can be freely utilized towards any objective of Nirdhan
Sewa Samaj as per the decision of its executive committee
Interest Not exempted As it is not gift or donation and it is investment return and as
investment of amount in bank cannot be an objective of any
organization, thus it can’t satisfy condition (2) as well. Hence, it’s
taxable, i.e. not exempted
Donation Exempted As donation is exempted amount
Dividend Income Not exempted As it is not gift or donation and it is investment return and as
investment of amount in shares cannot be an objective of any
organization, thus it can’t satisfy condition (2) as well. Hence, it’s
taxable, i.e. not exempted
Net profit from Not exempted Although the object clause may contain the provision of running a
Chapter 7- Tax Exempt Organizations & Exempted Amounts

running a school school, but the contribution from parents, i.e. school fee cannot be
free from expectation of consideration from the contributor, that
denies the satisfaction of condition (1) and (2) both specified above.

Conclusion:

Hence, in view of above, since the interest on deposit received by Nirdhan Sewa Samaj is not exempted (i.e.
taxable); it cannot get the refund of tax withheld by agents.

Legal provisions regarding filing of income tax return:

When TEO has taxable income, it is not relieved from submission of income tax return, and hence filing of
return is must. But in case when TEO has no any taxable income, it is not required to submit income tax return
u/s 97 (1) (a) of the Act, and requires to submit annual audited financial statements within three months of the
end of Income Year.

Renewal of Tax Exemption Certificate by TEO

There is no requirement as to the renewal of tax exemption certificate by TEO. Once audited financials are
submitted, it is deemed that exemption status is automatically renewed. Failure to submit audited financials
lead to revocation of exemption status until the audited financials is submitted.

Question 6 (page 45)

Definition of TEO:

Refer to Page 43 of the book.

Applicability of Sec. 87, 88, 88A and 89 for TEO

a. From the angle of TEO being payer:


The tax exempt organization is a resident entity; hence it is required to withhold tax on payments
attracting withholding taxes u/s 87, 88, 88A and 89 of the Act. Obtaining the status of TEO exempts
TEO from tax on specified incomes, but does not exempt the liability to withhold tax.
b. From the angle of TEO being Receiver:
In case of TEO being a Receiver:
• Sec. 87 is not applicable as TEO cannot be an employee and cannot generate employment
income
• The resident payer shall withhold tax on payments attracting WHT u/s 88 and 89 of the Act so far
as the amounts are not exempted u/s 10 of the Tax, and when the withholding of tax is relieved as
per Sec. 88 (4)
• The resident payer shall withhold tax in case it needs to pay windfall gain to TEO.

Question 7 (page 45)

In case the amounts so generated are exempted u/s 10 (g), i.e. the income satisfies any of the conditions
specified therein; there shall be no tax liability. However, in case the gain is from such income which are not
exempted u/s 10 (g), the tax liability shall be Rs. 125,000 (i.e. 25% of Rs. 500,000) as in the case of normal
entity.
Chapter 7- Tax Exempt Organizations & Exempted Amounts

SECTION “EXEMPTED AMOUNTS”

Question 1 (page 47)

As per Sec. 10 (b), income derived by a natural person deputed by foreign country in Nepal from employment
of a public service of a foreign government is not taxable in Nepal, in case both the following conditions are
satisfied:

a. The individual is resident in Nepal only because of his/her employment Nepal or is a nonresident of Nepal,
and
b. The payment of income is made from the public fund of foreign government.

In the given case, there is no doubt that Mr. Suresh’s income is paid from public fund of UK Government;
hence condition (b) above is satisfied.

With regard to the satisfaction of condition (a) above, we need to evaluate the following:

• Mr. Suresh, after doing his graduation in Nepal migrated to UK to further his study and for permanent
source of income,
• He was appointed by British Government in its service, and
• In the year 2068/69, he has been in Nepal with a group to make a study almost for whole year

In view of above fact, had he not been an employee of UK Government and had UK government not deputed
him in Nepal to make the study; he would not have been resident of Nepal. Hence, he is resident of Nepal
because of his/her employment only; thus, satisfying condition (a).

In light of above, Mr. Suresh is deriving exempted amounts u/s 10 (b) of the Act.

Question 2 (page 47)

Residential Status of Mr. Clinton for 20X1/X2

A natural person is resident in Nepal, in case s/he satisfies any of the following three conditions:

a. His/her habitual (normal) place of abode is in Nepal,


b. S/he resides in Nepal for 183 days or more during any period of consecutive 365 days, or
c. S/he is deputed by GON in foreign country

Since Mr. Clinton is residing in Nepal for more than 183 days during IY 20X1/X2 (since Ashwin 20X1 for 9
months), he is resident for the income year.

Calculation of taxable income of Mr. Clinton

Calculation of Assessable Income from Employment

Particulars Sec. Notes Amount


Salary 10 1 -
Dearness allowance 10 2 -
Foreign Allowance 10 2 -
Assessable Income from Employment Nil
Calculation of Assessable Income from Business

Particulars Sec. Notes Amount


Net profit from Handicraft business in Nepal 3 375,000
Chapter 7- Tax Exempt Organizations & Exempted Amounts

Assessable Income from Business 375,000


Calculation of Taxable Income

Assessable Income from Business 375,000


Assessable Income from Employment -
Assessable Income from Investment -
Assessable Income from Windfall gain -

TOTAL ASSESSABLE INCOME 375,000

LESS: ALLOWABLE DEDUCTIONS

a. Contribution to ARF -
b. Donation u/s 12B -
c. Donation u/s 12 (1) and 12 (2) 18,750
Lower of following:
• Actual 50,000
• 5% of Adjusted taxable income (5% of 375,000 less (a) less (b)) 18,750
• Max. 100,000

TAXABLE INCOME 356,250

Working Notes

1. Since Mr. Clinton is an employee of US Embassy in Nepal; he is resident in Nepal only as a result of his
employment of US government and salary derived by him is paid from public fund of US government; thus,
the amount is exempted u/s 10 (b).
2. As the dearness allowance and foreign allowance is paid to Mr. Clinton who is resident of Nepal only as a
result of employment of US government and the amount derived by him is paid from public fund of US
Government; the amount is exempted u/s 10 (b) and 10 (c) of the Act.
3. It is assumed that the net profit is derived after considering the deductions under Income Tax.
4. The gain of Rs. 150K from winning the game in Nepal is final withholding u/s 92 as it is windfall gain.
5. Renewal and subscription fee does not satisfy the conditions of Sec. 13 as it is not for business purpose,
hence not deductible for tax purpose.
6. Golf expense is incurred for other purpose than generating income from business, hence, not deductible
for tax purpose.
7. There shall be no assessable income from windfall gain so far as the windfall gain is having source in
Nepal for both resident and nonresident person since it is final withholding payment. In case the windfall is
having source outside Nepal, it shall form part of assessable income from windfall gain for resident
persons only since nonresidents are not required to pay tax on income other than those having source in
Nepal.
8. In absence of clear interpretation about the way to calculate adjusted taxable income for the purpose to
determine limit for donation, the ATI is calculated after deducting contribution to ARF, expenditure u/s 12 A
and 12B from total assessable income.
9. Tax deduction at source in Nepal is considered to determine the payable amount as tax after calculation of
tax liability, hence, not considered here as the question requires calculation of taxable income only.

Question 3 (page 47)

a. Since Mr. Khadka Bahadur Gurung is not a Nepali national, he is not entitled to facility u/s 10 (h) of Income
Tax Act on any of his income though he receives pension income after serving UK army. It is assumed that
Mr. Gurung has not obtained Nepalese citizenship after his retirement from UK Army.
b. Since Mr. Gurung resides in Nepal for whole of income year, he is resident of Nepal.
Chapter 7- Tax Exempt Organizations & Exempted Amounts

c. The question is silent about the nationality of Mrs. Mala Gurung; though she has not obtained any
exempted amount.
d. Since Mrs. Gurung resides in Nepal for a complete income year, she is resident of Nepal.
e. The provisions of Sec. 50 to opt Couple for tax assessment are designed for the benefit of taxpayer, when
one of the spouses is not generating income. It is normal for a rational taxpayer to plan tax is such a way
that it pays minimum tax. Hence, couple assessment of Mr. and Mrs. Gurung is not warranted.

Based on the above two facts, the calculation of Income of Mr. Gurung and Mrs. Gurung are calculated
hereunder:

Solution- Part (a) of question

A. Calculation of Tax Liability of Mr. Gurung


Assessable Income from Employment (WN1) 400,000
Assessable Income from Business (WN2) 300,000
Assessable Income from Investment -
Assessable Income from Windfall gain -
TOTAL ASSESSABLE INCOME 700,000
Less: Allowable Deductions:
a. Contribution to Approved Retirement Fund -
b. Expenditure u/s 12B -
c. Donation u/s 12 (3) -
d. Donation u/s 12 (1) and 12 (2) -
TAXABLE INCOME 700,000
Less: Allowable Reductions
a. Reductions for residing in Remote Area -
b. Reductions for Foreign Allowance -
c. Reductions in case of disability -
d. Reductions of Pension Income (62,500)
Lower of following:
• Pension Income 400,000
• 25% of Basic Exemption Limit (WN3) 62,500
e. Reductions for payment of Investment Insurance Premium -
f. Reductions for payment of Health Insurance Premium to resident entity -

BALANCE TAXABLE INCOME 637,500

Tax Liability
st
1 Rs. 250,000 (WN4) -
Next Rs. 100,000 15% 15,000
Balance Rs. 287,500 25% 71,875

TAX LIABILITY BEFORE TAX CREDITS 86,875

Less: Tax Credits -


a. Medical tax credit -
b. Female tax credit (Not eligible)
c. Foreign Tax Credit (WN5) -

TAX LIABILITY 86,875

Working Notes:

1. Pension income derived by him is the only income from employment, and hence detailed statement of
assessable income from employment is not produced. Pension income is related to past employment;
and as per cash basis- the amount is included in income.
Chapter 7- Tax Exempt Organizations & Exempted Amounts

2. Agricultural income is exempted only when it is not derived from conducting business from land except
that as per Sec. 12 (d) and 12 (e) of Land Act, 2021. As the amount is derived from business since he
hired land for cultivation and sells the produce, the amount is taxable.
3. As said earlier above, the assessment is individual assessment.
4. Effective from IY 2072/73, the social security tax on pension income is waived. Since there is no any
other employment income besides pension, there is no 1% tax.
5. In absence of information as to tax paid in UK for pension income, it is assumed that there is no any
tax paid by Mr. Gurung in UK.

B. Calculation of Tax Liability of Mr. Gurung


Assessable Income from Employment (WN1) 500,000
Assessable Income from Business -
Assessable Income from Investment -
Assessable Income from Windfall gain -
TOTAL ASSESSABLE INCOME 500,000
Less: Allowable Deductions:
a. Contribution to Approved Retirement Fund -
b. Expenditure u/s 12B -
c. Donation u/s 12 (3) -
d. Donation u/s 12 (1) and 12 (2) -
TAXABLE INCOME 500,000
Less: Allowable Reductions
a. Reductions for residing in Remote Area -
b. Reductions for Foreign Allowance -
c. Reductions in case of disability -
d. Reductions of Pension Income (62,500)
Lower of following:
• Pension Income 300,000
• 25% of Basic Exemption Limit (WN2) 62,500
e. Reductions for payment of Investment Insurance Premium -
f. Reductions for payment of Health Insurance Premium to resident entity -

BALANCE TAXABLE INCOME 437,500

Tax Liability
st
1 Rs. 200,000 (WN3) 1% 2,000
Rs. 50,000 -
Next Rs. 100,000 15% 15,000
Balance Rs. 87,500 25% 21,875

TAX LIABILITY BEFORE TAX CREDITS 38,875

Less: Tax Credits -


a. Medical tax credit -
b. Female tax credit (Not eligible) -
c. Foreign Tax Credit (WN4) -

TAX LIABILITY 38,875

Working Notes:

1. The Pension income and salary income derived by her forms part of assessable income from
employment, and hence detailed statement of assessable income from employment is not produced.
Pension income is related to past employment; and as per cash basis- the amount is included in
income.
2. As said earlier above, the assessment is individual assessment.
Chapter 7- Tax Exempt Organizations & Exempted Amounts

3. Effective from IY 2072/73, the social security tax on pension income is waived. Since there is other
employment income of Rs. 200,000 besides pension, social security tax of 1% tax is levied on such
income.
4. In absence of information as to tax paid in Hongkong for pension income, it is assumed that there is no
any tax paid by Mr. Gurung in UK.

Solution- Part (b) of Question

There will be no difference as pension income derived by a Nepali national from public fund of foreign
government is exempted only when the pension is received as a result of retirement serving to army or police
force of such foreign country. Since, Mrs. Gurung did not serve army or police service, the answer would be
same.

Question 4 (page 47)

As per Sec. 10 (g) (d), any amount derived by SEBON as per its objective is exempted for tax purpose. Hence,
any amount derived by SEBON as per its objective is not taxable at all.

Question 5 (page 47)

As per Sec. 10 (b), income derived by a natural person deputed by foreign country in Nepal from employment
of a public service of a foreign government is not taxable in Nepal (i.e. exempted), in case both the following
conditions are satisfied:

a. The individual is resident in Nepal only because of his/her employment Nepal or is a nonresident of Nepal,
and
b. The payment of income is made from the public fund of foreign government.

In the given case, Mr. Z is a Canadian national and deputed by Canadian government to work on Canadian
aided project. He satisfies Condition (a) above. Since he is paid in Canada for the service in Nepal, the
condition as to the payment to be made from public fund of foreign government is also met.

Similarly, as per Sec. 10 (c); any amount derived from public fund of foreign government by natural person or
his/her close relative as a result of his/her service to such government and satisfying the conditions as to (a)
and (b) above is exempted from tax.

Solution to part (a) of Question

As explained above, the salary, free accommodation and daily allowance received by Mr. Z in Canada is
exempted from tax as he satisfies both the conditions specified above.

Solution to part (b) of Question

Yes, in case of his employment to private contractor; the condition as to the payment of amounts from public
fund of foreign government is not satisfied, failure of which makes the amount taxable in Nepal.

Since he is nonresident of Nepal as he stays for only 94 days in Nepal, the salary, free accommodation and
daily allowance would be taxable at the rate 25%.
Chapter 8- Withholding Taxes & Collection of Advance Tax through Agents

Errors Rectified:
st
Page 53: 1 sentence after the definitions: The rate of withholding tax was mistakenly printed as 10%
instead of 5%. So, the 10% shall be replaced by 5%.

Page 53: Point 9- first sentence: The rate of withholding tax was mistakenly printed as 10% instead of
5%. So, the 10% shall be replaced by 5%.

Question 1- Page 63

a. As per Sec. 88 (1), interest having source in Nepal paid by a resident person attracts 15% withholding tax
at the time of payment. Sec. 88 (3) is an exception to Sec. 88 (1) when the rate of WHT is 5% in case of a
natural person if the conditions specified therein is satisfied. Hence, the rate of WHT to TEO by listed
resident entity at the time of payment of interest having source in Nepal is 15%.
As per Sec. 92, interest received by a TEO from a resident banks or financial institutions, listed company
or entities entitled to issue debenture is final withholding when such interest is paid against deposits,
bonds, debentures, government bonds or loan paper. Since the interest is paid by a resident listed
company, the interest is final withholding.
Hence the statement is TRUE.
b. The collection of advance tax by agents in case of sales of land and building is applicable only when such
property is owned by a natural person. In case of entity, it is entitled to calculate gain on disposal of asset
and liability u/s 37-49 as the case may be, and the agent is not required to collect advance tax on such
gain. The applicable tax rate is normal tax rate as specified in Schedule 1 (2) of the Act.
Hence, the statement is FALSE.

Question 2- Page 63

As per Sec. 89 (1), any payment by a resident person related to contract exceeding Rs. 50,000 shall attract
WHT at 1.5%. Sec. 89 (2) requires consideration of payment related to the same contract either made to the
contractor or associated person of contractor in last 10 days as well to determine the threshold of Rs. 50,000.

In this case, since the payment during a month is less than Rs. 50,000 (i.e. Rs. 40,000) assuming subsequent
payments are not made within 11 days of previous payments, there shall be no withholding of tax; and Sec. 89
is not applicable.

When the payments are made as given in question, the applicability of Sec. 89 is as follows:

• Rs. 60,000: Sec. 89 is applicable as the payment exceeds Rs. 50,000


• Rs. 50,000: Sec. 89 is not applicable as the payment does not exceed Rs. 50,000 as it is just Rs. 50,000
assuming there was no payment for the same contract in immediately preceding 10 days.
• Rs. 40,000: Sec. 89 is not applicable as the payment does not exceed Rs. 50,000 assuming there was no
payment for the same contract exceeding Rs. 10,000 in immediately preceding 10 days

Question 3- Page 63

i. As per 67 (6) (h), in case of operation of business related to transmission of information or news through
wire, optical fiber, radio or satellite; the payment is deemed having source in Nepal when the payment is
received for any such information or news transmitted through the equipments installed or established in
Nepal, regardless of the territory of origination of such news or information.
It means, the payment shall have source in Nepal only when such information are transmitted to
equipments installed in Nepal. In case of transmission of information through equipments installed outside
Nepal; the payment is deemed not having source in Nepal.
The two basic conditions for withholding of taxation as per Sec. 88 are as follows, both of which shall be
satisfied for the withholding of taxes:
• The payment shall be made by a resident person, and
• The payment shall have source in Nepal
Chapter 8- Withholding Taxes & Collection of Advance Tax through Agents

Though the company is resident in Nepal; since, it does not satisfy the source criteria, there shall be no
withholding of taxes on service fee paid for such services.
ii. As per Sec. 89; in case of payment exceeding 50,000 in relation to contract or agreement; there shall be
withholding of taxes at 1.5%. While calculating the threshold of Rs. 50,000; the payments made in the
immediately preceding ten days in relation to the same contract to the party or its associated person shall
also be considered.
As per the clarification clause to Sec. 89; Contract or Agreement includes supply of goods.
The basic criterion for withholding of taxes is the payer shall be resident; and source concept is not
applicable in this case.
Similarly as per Sec. 88, for any service having source in Nepal, a resident person shall withholding tax at
15% of the payment. In case of VAT registered service provider, the rate of withholding tax shall be 1.5%.
In the given situation, the company purchases various equipments and services from both domestic and
foreign suppliers. In this situation, the suggestion is provided as follows based on above provisions:
• Purchase of goods from domestic supplier: withholding of tax at 1.5% in case the payment
exceeds Rs. 50,000 in a moving 11 days period in relation to the contract of purchase of goods.
• Purchase of service from domestic supplier: withholding of tax at 1.5% of payment in case the
supplier is VAT registered. In all other case, the rate shall be 15%.
• Purchase of equipments from foreign suppliers: there shall be no withholding of taxes as it is
imported after duly complying custom formality and deemed as export sales for foreign suppliers.
The payment is not having source in Nepal, and as per Sec. 6 a nonresident is not required to pay
tax in income which does not have source in Nepal. Since such income is not taxable, it is
exempted from tax and no withholding of tax is required as per Sec. 89.
• Purchase of service from foreign suppliers: There shall be withholding of taxes at 15% as the
service is provided in Nepal, hence having source in Nepal and paid by resident. It also attracts
reverse VAT at 13% as per Sec. 8 (2).
iii. Maintenance work inside Nepal is service fee having source in Nepal and attracts WHT at 15%, except in
one case when the service provider is VAT registered.
In case of maintenance outside Nepal, the situs of service is outside Nepal; hence no WHT is required.
iv. As explained in answer (ii) above in relation to contract payment, except for vehicle maintenance and
advertisement which are services attracting WHT at 15% (except when the service provider is VAT
registered and attracting TDS at 1.5%), the contract is supply of goods. In case the payment exceeds Rs.
50,000 in moving 11 days period, it attracts withholding tax at 1.5%.

Question 4- Page 63

As per Sec. 88 (2), in case of payment of dividend by resident person; the rate of withholding tax is 5%
irrespective of the residential status of the person. As per Sec. 54, which is charging section for dividend; the
dividend paid by resident company and resident partnership shall be levied tax on final withholding basis. Sec.
92 specifies dividend as final withholding in case it is paid by resident company.

In the given situation, Mr. John is entitled to 40 Million dividend of the bank which is resident in Nepal. Banks
are company for the purpose of taxation. Hence the resident bank withholds tax at 5% while distributing
dividend. In this case, Worldwide Bank withholds tax of Rs. 2 Million and pays the balance of Rs. 38 Million to
Mr. John. The receipt of Rs. 38 million is final withholding.

Hence the tax liability of Mr. John is 2 Million as dividend distributed by the resident bank is the only income of
Mr. John for the Income Year.

The Worldwide bank withholds tax at 5% of Rs. 40 Million.

Question 5- Page 64

As per proviso to Sec. 88 (1), 10% WHT rate shall be applied while making payment against lease of aircraft
which has source in Nepal by a resident person. Similarly, the commission attracts WHT at 15% of payment, in
case the commission is having source in Nepal.
Chapter 8- Withholding Taxes & Collection of Advance Tax through Agents

In the given question, as the aircraft is used within the state of Nepal, the payment is having source in Nepal,
thus, Leisure Aviation (P) Ltd. shall withhold tax at 10% of Rs. 30 Lakhs per months while making the payment.

In case of payment of commission to German national, the service is performed outside the state of Nepal,
hence, no TDS shall be deducted.

Question 6- Page 64

As per proviso to Sec. 88 (1), the rate of withholding tax on gain while making payment of retirement payments
by ARF or GON shall be 5%. The formula for calculation of gain is given by Sec. 65.

In the given case, the gain shall be calculated as under:

Total payments by GON 2,000,000

Less: Benefit Accrued till 2058.12.18 (not taxable) 475,000

Taxable Retirement Payments 1,525,000

Less: Higher of following: (762,500)

a. 50% of Taxable Retirement payments 762,500


b. Maximum Amount 500,000

Taxable Gain 762,500

Tax @ 5% 38,125

Question 7- Page 64

Yes, commission having source in Nepal. The rate of withholding tax is 15%.

Question 8- Page 64

As per Sec. 67 (6), royalty is deemed having source in Nepal if it is paid against any asset situated in Nepal.
Since, the intangible asset is used in Nepal, it is situated within the state of Nepal, and any royalty payment
against such asset is deemed having source in Nepal.

As per Sec. 88 (1), the rate of WHT for payment of royalty having source in Nepal by resident person shall be
15%.

As such, M/s Fairly co. Ltd. shall withhold tax at 15% (i.e. Rs. 150,000) while making payment of Rs. 1 Million
to Zukuji Enterprises of Japan and remit the balance to the Japanese company.

Question 9- Page 64

Refer Page 60 & 61 of the book. There is list of 10 payments which are final withholding.

Question 10- Page 64

a. WHT shall be 1.5% of 300,000 (i.e. amount excluding VAT). The payment is not final withholding.
b. WHT shall be 5% of 550,000 (i.e. 50% of 1500,000 less 400000)- calculate using the formula given in
page 52 of the book. It is final withholding.
c. WHT shall be 5% of 200,000 (i.e. Total payments less benefit accrued till 2058.12.18 less contribution from
the beneficiary)- use the formula given in Page 53 of the book. It is final withholding.
d. As per Sec. 88 (4), a natural person cannot withhold tax on any payments except when the payments are
made in course of business. So, the question of withholding tax on personal borrowing of Binod does not
arise.
e. There shall be no withholding of taxes u/s 88 (4) of the Act. However, as Mr. Ramesh is employee of the
college, provisions of Sec. 87 are applicable on payments made from the college, and not of Sec. 88.
Hence, the amounts shall be included while calculating income from employment of Mr. Ramesh.
Chapter 8- Withholding Taxes & Collection of Advance Tax through Agents

Question 11- Page 64

a. The WHT shall be 1.5% of service fee excluding VAT, i.e. 1.5% of 200,000 per month.
b. The rate of WHT shall be 10%, and the payment is final withholding.
c. For the payment against publication of articles in Kantipur, there shall be no withholding of taxes u/s 88 (4),
however the amount is service fee and is taxable u/s 7 (2) (a). For the payment of part time lecture fee, the
rate of WHT shall be 15% which is final withholding.
d. The rate of WHT is 5%, which is final withholding.
e. The withholding tax shall be 5% of total distribution, i.e. 5% of Rs. 150,000 as the term distribution includes
both distribution of profit and capitalization of profit.

Question 12- Page 64

As per Sec. 95A, the Land Revenue Office shall collect advance tax on gain on disposal of Land or Building,
which is NBCA at 5% if the continuous ownership is less than 5 years and 2.5% if the continuous ownership is
5 years or more.

As per Sec. 2 (da), any private building satisfying any of the following three conditions is not NBCA:

a. In case the private building is owned by the person continuously for 10 years or more and the owner
resides in the private building for 10 years or more either continuously or intermittently.
Private building means, in case of building occupying area up to 1 ropani of land- the building and the area
occupied by the building & in all other case, the building and 1 ropani of land out of total area occupied by
the building.
b. In case the private building is disposed off for less than Rs. 3 Million, or
c. In case the private building is disposed by way of transfer within three generations, except by way of sales
or purchase.

Since, the private building in question does not satisfy any of the above three criteria, assuming the building
occupies whole of 1 ropani land and falls under the definition of private building, Land Revenue Office shall
collect advance tax on gain on disposal of such building.

As per Sec. 1 of Schedule 1 of the Act, the maximum tax rate for such gain is restricted to rate specified by
Sec. 95A of the Act, i.e. 5% of gain if the continuous ownership is less than 5 years and 2.5% of gain if the
continuous ownership is 5 years or more.

The gain on disposal is:

Incomings at the time of disposal on 31.12.20X-11 15,000,000

Less: Outgoings for the building (cost plus subsequent cost) (6,000,000)

Gain 9,000,000

Tax @ 2.5% (as ownership exceeds 5 years since 20.11.20X6) 225,000

In case the building were purchased on 20.11.20X1, the gain shall not be taxable as it does not fall under the
definition of NBCA, assuming Mr. Ramesh resided in building for 10 years or more during the ownership
period. Since, the asset does not fall under either of business asset, trading stock, depreciable asset or NBCA;
the gain is beyond the charging scope of Income Tax Act, 2058.

Question 13- Page 64

Gain on Disposal

A. Net Incomings (3500 * 500 – 8750) 1,741,250


B. Net Outgoings (700 * 500 + 2800) 352,800

Gain (A – B) 1,388,450
Chapter 8- Withholding Taxes & Collection of Advance Tax through Agents

Advance Tax to be collected (10% of Gain) 138,845

Since it is the shares of listed company and the owner is nonresident, as per Sec. 95A advance tax shall be
collected by NEPSE at 10% of gain.

The payment is not final withholding, hence, Mr. X is required to submit income tax return and pay tax @ 25%
less the advance tax collected by NEPSE.

Question 14- Page 65

Competition and Contribution are two different things. In case a person wins national and international level
prizes in recognition of his/her contribution in eight different fields listed in Sec. 88A (2), the prize up to Rs. 5
Lakhs is exempted from tax.

In the case, since it is prize won in case of competition which is not equivalent national or international level
prizes; the amount is taxable for tax purpose u/s 88.

Question 15- Page 65

House rental payment to Garibdash attracts WHT at 10% as per proviso to Sec. 88 (1) of the Act.

In case of payment of vehicle rental for hiring of vehicle owned by an individual paying tax u/s 1 (13) of
Schedule 1 of the Act, Income Tax Manual has specifically relieved the withholding of taxes. Since IRD is
bound to act as per its public circular, there shall be no withholding of tax while making payment of vehicle
rental to Mr. Daman Thapa. (Refer Page 282 of pdf version of Income Tax Manual 2068, example no. 16.2.26)

Question 16- Page 65

Amount of dividend net of TDS= 10% of paid up capital= 10% of 15,000,000= Rs. 1,500,000

As per Sec. 88 (2), the rate of withholding tax is 5% on dividend, i.e. the recipient is entitled to receive 95% of
the total payments.

Assuming “x” be total payment, then

95% of “x”= 1500,000

i.e. “x”= 1500,000 /.95= 1,578,947.37

Therefore, amount of cash dividend is Rs. 78,947.37

Question 17- Page 65

a. Gain on Disposal of Shares


Net Incomings (500*7000-17500) 3482500
Net Outgoings (500*1,000+4,000) 504000
Gain on Disposal of Shares 2978500
b. As per Sec. 95A, NEPSE shall collect advance tax on gain on disposal of listed shares. The rate of
advance tax shall be 5% on gain for resident natural person and 10% on gain for others. Assuming Mr. Anil
to be resident natural person, the rate shall be 5% on gain (calculate the amount of tax yourselves)
c. In case the shares were held and sold by an entity, the rate of advance tax to be collected by NEPSE shall
be 10% of gain instead of 5% as in the case of Mr. Anil, whom we assume is a resident natural person.

Question 18- Page 65

a. The amount to be received by M/s PC P. Ltd. is service fee having source in Nepal. As M/s M Ltd. is a
resident person, it shall withhold tax at 15% of the payment as there is no VAT invoice, neither the service
is exempted from VAT, i.e. 15% WHT shall be levied on service fee of Rs. 40,000.
As Rs. 20,000 is also part of service performed by the service provider, principally tax shall be withheld on
such amount as well, which means tax shall be withheld at 15% on Rs. 60,000.
Chapter 8- Withholding Taxes & Collection of Advance Tax through Agents

b. Distribution of iPhone is distribution of profit and is taxable u/s 54 of the Act, which means the company
shall collect WHT @5% of Market Value of iPhone (i.e. 5% of 38,000) from each shareholder.
c. As Mr. Mahesh is making payment to engineer in personal capacity and not for his business, though it is a
payment of service fee; Mahesh shall not withhold tax on such payment as per Sec. 88 (4) of the Act.
d. As per Sec. 88 (4), WHT is not required while making payment of interest to resident Banks or Financial
Institutions. As per general practice, service fee on loan is also part of interest, thus, MNO Co. P. Ltd. shall
not withhold tax while making payment of service fee and interest to Hamro Bank Ltd., a resident bank.
As definition of interest includes any excess payment above principal against debt obligation as “interest”,
the service fee is also treated as part of interest paid to banks.
e. The amount received by Mr. Ram to deliver lecture is a part time lecture fee, which is service fee and
attracts TDS @ 15%. In relation to payment received for setting up question paper and checking answer
sheets, Sec. 88 (4) relieved such payment from Withholding of Taxes, thus, WHT is not required.

Question 19- Page 65

a. As per Sec. 88 (4), WHT is not required while making payment of interest to resident Banks or Financial
Institutions. Thus, Z & Co. shall not withhold tax while making payment of service fee and interest to Nabil
Bank Ltd., a resident bank.
b. As Mr. Gokul is making payment to construction company in personal capacity and not for his business,
though it is a payment of service fee; Gokul shall not withhold tax on such payment as per Sec. 88 (4) of
the Act. The practice of Gokul is correct.
c. The insurance company shall withhold tax at 5% of gain. Calculate yourself the gain amount and TDS
amount.
d. As per Sec. 88A, there shall be withholding of 25% tax on windfall gain. As per Sec. 2, winning a lottery
falls under the definition of windfall gain. Thus, Mr. Smart is wrong as he is bound to pay tax in the form of
final withholding payments.
th th th
e. As the payment on 17 Shrawan exceeds Rs. 50,000 (since 7 Shrawan to 17 Shrawan- there is total
payment of Rs. 55,000 and as per Sec. 89-2, it must be considered while calculating the threshold of Rs.
th
50000), X & Co shall withhold tax on each of payments from 8 Shrawan as per Sec. 89.

Question 20- Page 65

In principle, the Withholding tax is applicable as applicable for rent.


Chapter 9- Tax Rates

Errors Rectified:

Page 67- last sentence (a) to be redrafted as:


a. The maximum tax rate of a natural person on his/her income from Disposal of Non Business
Chargeable asset is 10% subject to the following: …………
th
Page 68- 7 Line to be changed as:
Since the maximum tax rate for income from disposal of Non Business Chargeable Asset is 10%, there
shall be no additional 40% surcharge even when the balance taxable income exceed Rs. 25 Lakhs.

Page 68- Point (d) to be added after point (c):


In case a natural person carrying on operation of Special industry exports its products, the tax rate shall
be 11.25% instead of rates applicable to block of 15% and 25%, and 15.75% when the income is related
to the block where 35% is applicable.

Notes:

The rates of tax taken on this chapter are as applicable for IY 2072/73. Please note the same.

Question 1- Page 71

Provisions of Income Tax Act- facilities entitled to Mr. Ram

a. As he is handicapped, he is entitled to reduction of 50% of Basic Exemption Limit while calculating


Balance Taxable Income from Taxable Income.
b. Similarly, the rate of tax on export income on such income where rate of 25% is applicable is
15%,and since its export from the products of special industry, the effective rate instead of applicable
rate of 15% and 25% is 11.25% and that of applicable rate of 35% is 15.75% .
c. He is operating a special industry as defined by Clarification Clause of Sec. 11, thus, the rate of tax
on income from Special industry where rate of 25% is applicable is 20%.
d. Students should note that there are three methods of calculation of tax liability in this case. One
method benefits taxpayer, the other the tax receivers and the third method assumes the consumption
of deductions and reductions on proportionate basis. In this solution, the author has used the third
method that gives the result on trade off position.
e. Corrections in question, instead of 10 Million of export sales make it Rs. 100 Million.

Calculation of Taxable Income:


Assessable Income from Business
From Export Income: 10,000,000
From other Special Industry 15,000,000
Total Assessable Income 25,000,000
(as there is no information as to income from investment or employment)
Less: Allowable Deductions Nil
Less: allowable Reductions

a. Reduction as a result of incapacitated person 150,000


(50% of 300,000- BEL for Couple as the question suggests)

Balance Taxable Income 24,850,000

Calculation of Balance taxable income attributable to export sales and local sales
Chapter 9- Tax Rates

• Attributable to Export Sales= (Profit from Export included in Assessable income/ Total
Assessable income less NBCA Income * Balance Taxable Income excluding NBCA Income)=
10/25*24,850,000= 9,940,000
• Attributable to Local Sales= (Profit from Local Sales included in Assessable income/ Total
Assessable income less NBCA Income * Balance Taxable Income excluding NBCA Income)=
15/25*24,850,000= 14,910,000

Calculation of Tax Liability:


st
1 Rs. 300,000 -
Next Rs. 100,000 15% 15,000
Next Rs. 2,100,000 20% 420,000
(Assuming the amount up to Rs. 25 Lakhs exhausted by other Business Income, not export income)
Next Rs. (24,850,000- 9,940,000-2,500,000) 28% 3,474,800
Balance Rs. 9,9400,000 15.75% 1,565,550
Total Tax Liability 5,475,350

Note: There are two alternative ways to calculate tax liability, and the answer is not final. Please have a
look at Example in Page 73-76 of the book for the same.

Question 2- Page 71

a. It is assumed that each of the individual is opted Couple assessment for the purpose of tax as per
Sec. 50.
1. Calculation of Tax Liability of Mr. A
Since the total taxable income is less than Rs. 300,000; there shall be no tax liability. Had he opted
individual for tax assessment, the tax liability would be 5% of Rs. 40,000 (i.e. 5% of 290,000 less Rs.
250,000).
2. Calculation of Tax Liability of Mr. B
Mr. B has taxable income attributable to gain on disposal of land and building with ownership more
than 5 years of Rs. 290,000 which is subject to maximum tax rate of 2.5%. The balance of Rs.
100,000 is assumed to be business income. As suggested by Supreme Court decision based on
minimum tax liability to tax payer, the tax liability would be:
st
1 Rs. 300,000 0% -
Balance Rs. 90,000 2.5% 2,250
3. Calculation of Tax Liability of Mr. C
Mr. C has taxable income attributable to gain on disposal of listed shares of Rs. 290,000 which is
subject to maximum tax rate of 5%. The balance of Rs. 400,000 is assumed to be business income.
As suggested by Supreme Court decision based on minimum tax liability to tax payer, the tax liability
would be:
st
1 Rs. 300,000 0% -
Next Rs. 100,000 15% 15,000
Balance Rs. 290,000 5% 14,500
4. Calculation of Tax Liability of Mr. D
Mr. D has taxable income attributable to gain on disposal of land and building with ownership less
than 5 years of Rs. 290,000 which is subject to maximum tax rate of 5%. The balance of Rs.
1,800,000 is assumed to be business income. As suggested by Supreme Court decision based on
minimum tax liability to tax payer, the tax liability would be:
st
1 Rs. 300,000 0% -
Next Rs. 100,000 15% 15,000
Chapter 9- Tax Rates

Next Rs. 1,500,000 25% 375,000


Balance Rs. 290,000 5% 14,500
5. Calculation of Tax Liability of Mr. E
Mr. E has taxable income attributable to gain on disposal of unlisted shares of Rs. 290,000 which is
subject to maximum tax rate of 10%. The balance of Rs. 2,800,000 is assumed to be business
income. As suggested by Supreme Court decision based on minimum tax liability to tax payer, the tax
liability would be:
st
1 Rs. 300,000 0% -
Next Rs. 100,000 15% 15,000
Next Rs. 2,100,000 25% 525,000
Next Rs. 300,000 35% 105,000
Balance Rs. 290,000 10% 29,000
Please note that there shall not be application of 40% surcharge in taxable income above Rs. 25
Lakhs in Gain on Disposal of Non Business Chargeable Asset. Sec. 1 (4) of Schedule 1 contradicts
with Sec. 1 (1) and Sec. 1 (2) of Schedule 1, as it does not mention application of surcharge of 40%
on taxable income above Rs. 25 lakhs in Gain on Disposal of NBCA while determining the maximum
tax rate for NBCA income to a resident natural person of 10%. Hence as per interpretation of law, the
40% surcharge is not applicable on NBCA income.

Question 3- Page 71

a. Since Mr. Mohit Shrestha is a natural person paying tax u/s 1 (13) of Schedule 1 of the Act, he is not
required to make detailed calculation of tax under Sec. 4 of the Act. Hence, the tax paid by Mr.
Shrestha against each vehicle is final tax liability.
The tax liability of Mr. Shrestha for IY 20X1/X2 is:
Vehicle No. of Vehicles Amount per vehicle Total
Power Tiller 7 1,000 7,000
Tractor 11 1,000 11,000
Auto rickshaw 1 1,550 1,550
Micro bus 4 2,400 9,600
Car 8 2,400 19,200
Truck 4 3,000 12,000

Question 4- Page 71

Calculation of Total Assessable Income:

Assessable Income from Business 5,000,000


Assessable Income from Employment 1,000,000
Assessable Income from Investment 8,000,000
Assessable Income from Windfall Gain -
Total Assessable Income 14,000,000

Statement of Taxable Income


Total Assessable Income 14,000,000
Less: Allowable Deductions:
a. Contribution to Approved Retirement Fund -
b. Contribution u/s 12B -
c. Donation u/s 12 (3) -
d. Donation u/s 12 (1) & 12 (2) -
Chapter 9- Tax Rates

Taxable Income 14,000,000

Statement of Balance Taxable Income


Taxable Income 14,000,000
Less: Allowable Reductions under Schedule 1
a. Reduction against residing in Remote Area -
b. Reduction of “Foreign Allowance” -
c. Reduction to a disabled Individual (125,000)
(as he is divorcee, he has to opt individual for tax assessment)
d. Reduction against payment of Investment Insurance Premium (20,000)
(Lower of Rs. 20,000 or actual Rs. 25,000)
e. Reduction against pension Income (62,500)
(Lower of 25% of BEL, i.e. 25% of 250,000 or Actual Pension Income of Rs. 100,000)
f. Reduction against payment of Health Insurance Premium to a resident Insurance Co. (20,000)
(Lower of Rs. 20,000 or actual amount of Rs. 50,000)
Balance Taxable Income 13,772,500

Statement of Tax Liability


Calculation is based on minimum tax liability to tax payer, there are other two alternatives to the solution
st
1 Rs. 250,000 1% 2,500
(as the employment income after deducting pension income and
reductions exceed Rs. 250,000. In this case, the 1% SST is taken at
maximum as the employer is bound to collect SST on any employment
income of Rs. 9 Lakhs paid to him)
Next Rs. 100,000 15% 15,000
Next Rs. 1,422,500 25% 355,625
(Rs. 2 million- other business income & employment income less amount
in 1% and 15% slab and reductions)
Next Rs. 727,500 20% 145,500
(Out of Rs. 2 million business income from special industry to make it Rs.
25 Lakhs)
Next Rs. 1,272,500 28% 356,300
(Income from Special Industry above 25L Taxable income)
Next Rs. 2,000,000 21% 420,000
(all export sales above Rs. 25L taxable income)
Next Rs. 1,000,000 2.5% 25,000
(Maximum tax rate of 2.5% for gain on disposal of land and building with
ownership exceeding 5 years)
Next Rs. 2,000,000 5% 100,000
(Maximum tax rate of 5% for gain on disposal of land and building with
ownership less than 5 years)
Next Rs. 3,000,000 5% 150,000
(Maximum tax rate of 5% for gain on disposal listed shares)
Balance Rs. 2,000,000 35% 700,000
(Investment income other than gain on disposal of NBCA attract highest marginal tax rate)
Total (Sum up the above)
Less: Tax Credits -
Tax liability
Chapter 10- Business Concessions & Facilities

Notes:

The rates of tax taken on this chapter are as applicable for IY 2072/73. Please note the same.

Question 1- Page 82

a. As it is a special industry, the applicable tax rate for the income where 25% rate is applied is 20%. In
case of attributable income exceeding 25L, the effective tax rate for such excess income would be
28%.
b. In case of export income, the applicable tax rate for the income where 25% rate is applied is 11.25%.
In case of attributable income exceeding 25L, the effective tax rate for such excess income would be
15.75%.

Calculation of Business Income attributable to Export Sales and Local Sales:


Total Sales 15,000,000
Total Cost of Sales (9,500,000)
(Assuming all costs is deductible under Income Tax Act, 2058)
Income from Business 5,500,000
Business Income from Export Sales (25%) 1,375,000
(Assuming indirect costs are also incurred for export sales in the same proportion
as incurred for local sales)

The total taxable income and balance taxable income of Mr. Ramhari is equal to business income
calculated above in absence of information related to employment and investment income and deductions
and reductions as per Income Tax Act.

Calculation of Tax Liability (Individual assessment as suggested by question)


1st Rs. 250,000 0% -
Next Rs. 100,000 15% 15,000
Next Rs. 2,150,000 20% 430,000
Next Rs. 1,625,000 (5,500,000-1,375,000-2,500,000) 28% 455,000
Balance Rs. 1,375,000 15.75% 216,562.50
Note that the calculation is based on minimum tax liability to taxpayers; there are other two approaches to
calculation of tax liability as suggested in Chapter 9 of the book through an illustration.

Question 2- Page 82
Relevant information:

a. It’s a jute mill, which means it’s a special industry as defined by Sec. 11 of the Act. The applicable tax
rate is 20% as per Sec. 2 (3) of Schedule 1 of the Act.
b. It’s a 100% export based industry, which means the concessions to export based special industry is
applicable to it.
c. It employs 1200+ Nepalese national throughout the year, which means concessions on the basis of
direct employment is also applicable to it.
d. As per Sec. 11 (6) and 11 (7), if there are two or more exemptions or concessions are applicable to a
person; the person can choose one out of those options available. Students should note that every
taxpayer is rational, which means the taxpayers choose the alternative having less cash outlay as tax.

Based on above information, the effective tax rate on alternative (b) or (c) above shall be:
Chapter 10- Business Concessions & Facilities

Alternative (b) = There shall be rebate of 25% tax on the applicable tax rate, which means the effective
tax rate shall be 75% of applicable tax rate. That means, the effective tax rate as per this alternative is
15% (75% of 20%)

Alternative (c) = the effective tax rate shall be 80% of the applicable tax rate. That means, the effective
tax rate as per this alternative is 16% (80% of 20%)

Out of above two alternatives, the minimum tax liability will be on alternative (b) explained above, i.e.
Worthy Jute Industries Limited is entitled to pay tax at 15% of taxable income.

In the given question, the taxable income is Rs. 12,550,000. The tax liability shall be Rs. 1,882,500
instead of Rs. 2,008,000.

The accountant failed to identify its possibility of reducing tax liability.

Question 3- Page 82
As per Sec. 11 (1), there shall be no tax on agricultural income except in the following cases:

(See Page 77 of the book)

As any of the above criteria is not fulfilled in case of Mr. Ram Prasad, the income is exempted from tax
u/s 11 (1) of the Act.

In case of conduct of agricultural business through partnership concern after registering it, the amount is
taxable and the entity would be liable to pay tax.

Question 4- Page 82
As per Sec. 11 (2Ka) of the Act, the tax on the interest income derived by an individual up to Rs. 25,000
during any Income Year is exempted in case the interest is paid by Micro Finance Institutions, rural
development banks, post office saving banks and cooperatives based on rural areas.

In the given case, Ram Kumar in Parbat received Rs. 16,000 as interest from Grameen Bikas Bank which
is exempted from tax as per above provisions.

In case Mr. Ram Kumar received Rs. 30,000, which is above Rs. 25,000; the amount would be taxable.

Question 5- Page 82
The cooperative is in the list of exempted cooperatives (See Page 77, point 2.1 of the book), and hence,
the amount is not taxable.

In case of dividend distributed by such cooperatives, the applicable tax is also exempted.

Question 6- Page 82
The applicable tax rate for the management fee or technology transfer fee shall be exempted by 50% in
case of industries established in SEZ. Hence, the foreign investors are entitled to pay tax at half the
applicable rate on such fees. The rate shall be 7.5% after applying the concessions.

Question 7- Page 82
See page 80 (point 6.2 of the book)

Question 8- Page 82

a. The applicable tax rate is 20% as it is a special industry.


Chapter 10- Business Concessions & Facilities

b. As it provides direct employment to 300+ Nepalese nationals throughout the Year, the effective
tax rate shall be 90% of applicable tax rate, i.e. 18%
c. As it is based on underdeveloped area, it is entitled to effective tax rate of 30% of applicable tax
st
rate for 1 ten Income Years including the Income Year of first operation; which means the
effective tax rate for first ten Income Years shall be 6%.

Based on above analysis, ABC Ltd. is required to pay tax at 6% for first ten Income Years including
the first Income year of Operation and 18% thereafter, provided the information given in the question
is not changed.
Chapter 11- Income from Business

Errors Rectified:

Page 97 line 25/26: the lines to be replaced as follows:

“In case of person following Cash basis of Accounting, the cost of production shall be determined
using Prime Costing (or Marginal Costing or Variable Costing) Method or Absorption Costing
Method”.

Notes:

The rates of tax taken on this chapter are as applicable for IY 2072/73. Please note the same.

Question 1- Page 84

Provision of the Act (Sec. 22 (6))

In case of change in basis of accounting, the taxpayer shall make necessary adjustments in income
calculation in the Income Year when the basis of accounting is changed in such a way that no amount
included, deducted, to be included or to be deducted in calculating the person’s income of the Income
Year is omitted or repeated.

Based on the above provisions:

a. As per accrual basis of accounting, the amount shall be included in Income in the Income Year
20X1/X2. However, as the person followed cash basis of accounting in that Income Year and the
amount was not received in cash, the amount was not included in Income. As such, the amount shall
be included in income as per Sec. 22 (6) in IY 20X2/X3 (No omission/no repeat)
b. As per accrual basis of accounting, the amount shall be included in Income in the Income Year
20X2/X3. However, as the person followed cash basis of accounting in that Income Year and the
amount was received in cash, the amount was included in Income. As such, the amount shall not be
included in income as per Sec. 22 (6) in IY 20X2/X3 (No omission/no repeat)
c. As per accrual basis of accounting, the amount of house rent for Shrawan 20X2 till Poush 20X2 shall
be deductible in the Income Year 20X2/X3. However, as the person followed cash basis of
accounting in that Income Year and the amount was paid in cash, the amount was deducted while
deriving the Income from business. As such, the amount shall not be deducted in income as per Sec.
22 (6) in IY 20X2/X3 (No omission/no repeat)

Question 2- Page 84

Provision of the Act (Sec. 22 (6))

In case of change in basis of accounting, the taxpayer shall make necessary adjustments in income
calculation in the Income Year when the basis of accounting is changed in such a way that no amount
included, deducted, to be included or to be deducted in calculating the person’s income of the Income
Year is omitted or repeated.

Based on the above provisions:

a. As per accrual basis of accounting, the amount shall be included in Income in the Income Year
20X1/X2. However, as the person followed cash basis of accounting in that Income Year and the
amount was not received in cash, the amount was not included in Income. As such, the amount shall
be included in income as per Sec. 22 (6) in IY 20X2/X3 (No omission/no repeat)
Chapter 11- Income from Business

b. As per accrual basis of accounting, the amount shall be included in Income in the Income Year
20X2/X3. However, as the person followed cash basis of accounting in that Income Year and the
amount was received in cash, the amount was included in Income. As such, the amount shall not be
included in income as per Sec. 22 (6) in IY 20X2/X3 (No omission/no repeat)
c. As per accrual basis of accounting, the amount of house rent for Shrawan 20X2 till Poush 20X2 shall
be deductible in the Income Year 20X2/X3. However, as the person followed cash basis of
accounting in that Income Year and the amount was paid in cash, the amount was deducted while
deriving the Income from business. As such, the amount shall not be deducted in income as per Sec.
22 (6) in IY 20X2/X3 (No omission/no repeat)
Chapter 11- Income from Business

Question 1- Page 88

The question is not clear, whether the company is claiming the amount added by the employer as
contribution of employers towards Retirement Fund account of employee or the deducted amount was
contribution of employees in unapproved retirement fund.

In the first case, as the amount is employee expenses of the company; the amount is deductible u/s 13 of
the Act as it satisfies all the three conditions prescribed by Sec. 13 of the Act.

In the second case, as the contribution is the expenditure of employee and hence, it does not satisfy a
criteria of “to be incurred by the person”, the amount is not deductible.
Chapter 11- Income from Business

Question 1- Page 91

Please refer to point no. 3 of Page 90 (Sec. 21-1 (gha))

Question 2- Page 91

Please note that Sec. 21 specifies of the “payments” not only “expenditures”. Hence, any payment above
Rs. 50,000 at a time when the turnover of the person exceeds Rs. 20 Lakhs during the Income Year is
not deductible except in the six exceptions mentioned in Sec. 21 (2) of the Act.

In the given case, as the payment exceeds Rs. 50,000; in case if the turnover exceeds Rs. 2 Million: the
amount is not deductible for tax purpose.

Question 3- Page 91

Refer Page 89 of the book

Question 4- Page 91

In your answer, write down the six exceptions from page 90 of the book.

Conclusion:

As the payment is made to farmer, the amount is deductible for tax purpose even when the produces are
primarily processed. However, if the payment is for readymade items; the amount would not be
deductible.
Chapter 11- Income from Business

Question 1- Page 95

Copy all provisions of Sec. 14 of the Act.

Question 2- Page 95

As per Sec. 14 (2), in case a “resident entity controlled by exempt organization” pays interest to the
“Controlling entity”, the maximum amount of interest that can be claimed as expenses shall be limited to
the sum of following:

a. Interest Income derived by the person during the Income Year, and
b. 50% of Adjusted taxable income derived for the person excluding interest income and interest
expenses.

In the given question, Fewa P. Ltd. is a “resident entity controlled by exempt organization” as 80% of its
shares are held by nonresidents, where it would be “resident entity controlled by exempt organization”
even when 25% of its shares were held by nonresident persons. As it paid interest to its foreign investors
Sec. 14 (2) is applicable.

The calculation of adjusted taxable income is done as prescribed by Income Tax Manual 2066 (Updated
2068):
Sales Income 5,000,000
Less: Deductions
Cost of Sales (assuming calculated as per Sec. 15) 2,000,000
Admin Expenses (assuming as per Sec. 13) 1,100,000
Pollution Control Expenses 300,000
Adjusted Taxable Income (assuming there are no other expenses) 1,600,000

Therefore, eligible expenses u/s 14 (2): Lower of following

a. Actual Interest paid to foreign investors 1,400,000


b. Sum of following: 875,000
• Interest Income 75,000
• Add: 50% of ATI Calculated above 800,000

Eligible Expenses u/s 14 (2) 875,000


Eligible Interest Expenses u/s 14 (1)- assuming paid for loan used for business 100,000
Total Eligible Expenses u/s 14 975,000

Ineligible expenses 525,000

Treatment of Ineligible Expenses:


As per Sec. 14 (3), the excess of actual interest paid to controlling person over the interest claimable u/s
14 (2) can be carried forward to be claimed in following Income Year(s), and is deemed to be paid for
loans utilized in the following Income Year(s).
In the given case, the excess of Rs. 575,000 is carried forward to be claimed in following Income Year(s)
and the same amount is deemed to be paid as interest for the loans utilized in following Income Year(s).

Question 3- Page 95
Chapter 11- Income from Business

As per Sec. 14 (2), in case a “resident entity controlled by exempt organization” pays interest to the
“Controlling entity”, the maximum amount of interest that can be claimed as expenses shall be limited to
the sum of following:

a. Interest Income derived by the person during the Income Year, and
b. 50% of Adjusted taxable income derived for the person excluding interest income and interest
expenses.

In the given question, Ram Trading P. Ltd. is a “resident entity controlled by exempt organization” as it is
wholly owned by nonresident Mr. Thapar, where it would be “resident entity controlled by exempt
organization” even when 25% of its shares were held by nonresident persons. As it paid interest to its
foreign investors Sec. 14 (2) is applicable.

The calculation of adjusted taxable income is done as prescribed by Income Tax Manual 2066 (Updated
2068):
Sales Income 500,000
Less: Deductions
Admin Expenses (assuming as per Sec. 13) 300,000
Depreciation (assuming the deduction as per Sec. 19) 100,000
Adjusted Taxable Income (assuming there are no other expenses) 100,000

Therefore, eligible expenses u/s 14 (2): Lower of following

a. Actual Interest paid to foreign investors 40,000


b. Sum of following: 70,000
• Interest Income 20,000
• Add: 50% of ATI Calculated above 50,000

Eligible Expenses u/s 14 (2) 40,000


Eligible Interest Expenses u/s 14 (1)- assuming paid for loan used for business 60,000
Total Eligible Expenses u/s 14 100,000

Ineligible expenses -

Question 4- Page 95

As per Sec. 14 (2), in case a “resident entity controlled by exempt organization” pays interest to the
“Controlling entity”, the maximum amount of interest that can be claimed as expenses shall be limited to
the sum of following:

a. Interest Income derived by the person during the Income Year, and
b. 50% of Adjusted taxable income derived for the person excluding interest income and interest
expenses.

The calculation of adjusted taxable income is done as prescribed by Income Tax Manual 2066 (Updated
2068):
Sales Income 750,000
Less: Deductions
Cost of Sales (assuming calculated as per Sec. 15) 500,000
Admin Expenses (assuming as per Sec. 13) 120,000
Chapter 11- Income from Business

Depreciation (assuming as per Sec. 19) 20,000


Adjusted Taxable Income (assuming there are no other expenses) 110,000

Therefore, eligible expenses u/s 14 (2): Lower of following

a. Actual Interest paid to foreign investors 75,000


b. Sum of following: 60,000
• Interest Income 5,000
• Add: 50% of ATI Calculated above 55,000

Eligible Expenses u/s 14 (2) 60,000


Eligible Interest Expenses u/s 14 (1)- assuming paid for loan used for business -
Total Eligible Expenses u/s 14 60,000

Ineligible expenses 15,000

Treatment of Ineligible Expenses:


As per Sec. 14 (3), the excess of actual interest paid to controlling person over the interest claimable u/s
14 (2) can be carried forward to be claimed in following Income Year(s), and is deemed to be paid for
loans utilized in the following Income Year(s).
In the given case, the excess of Rs.15,000 is carried forward to be claimed in following Income Year(s)
and the same amount is deemed to be paid as interest for the loans utilized in following Income Year(s).
Chapter 11- Income from Business

Question 1- Page 98

Please refer to valuation of closing stock portion of the book. Note the following:

• Cost or Market value, whichever is lower


• FIFO or weighted average cost formula
• Use of Prime Costing or Absorption Costing Method

Question 2- Page 98

Calculation of Value of Closing Stock for Income Tax Purpose- using absorption costing method and
assuming FIFO Cost Formula

Quantity Amount

Calculation of Cost of Production

Consumption of raw materials 2,500,000


Wages 1,500,000
Power 1,200,000
Stores consumed (assuming direct cost) 800,000
Repairs & maintenance building (ignored as cannot be included) -
Machinery (ignored as depreciation is separately calculated) -
Furniture (ignored as depreciation is separately calculated) -
Computers (ignored as depreciation is separately calculated) -
Other Factory Overheads 250,000
Administrative Overhead (ignored, as deductible u/s 13) -
Interest on Bank Loan (ignored, as deductible u/s 14) -
Total cost of production 6,250,000
Units produced 120,000
Per unit cost 52.08
Cost of Closing Stock 2250 117,187.50
(2250*52.08)
Market Value (not given and expected to be higher)
Therefore, value of closing stock is Rs. 117,187.50
Chapter 11- Income from Business

Question 1- Page 101

Calculation of Allowable Repair & Improvement Expenses u/s 16 for IY 20X1/X2

Particulars Pool A Pool B Pool C Pool D Total


A. Depreciable Basis 100,000,000 800,000 16,000,000 - 116,800,000
B. 7% of A 7,000,000 56,000 1,120,000 -
C. Actual Repair & Improvement 2,000,000 100,000 1,600,000 - 3,700,000
Expenses
D. Allowable u/s 16 (B or C, 2,000,000 56,000 1,120,000 - 3,176,000
whichever is lower)
E. Excess (C-D) - 44,000 480,000 524,000

a) As calculated above, since it’s not the case of overhauling of aircraft as per the Standard of Civil
Aviation Authority of Nepal or repair and improvement expenditure of assets lost in earthquake, the
deductible repair and improvement expenditure is Rs. 3,176,000; and not 3,700,000.
b) The excess of repair and improvement expenditure over deductible amount as per Sec. 16 is
capitalized in respective pool while deriving at Opening Depreciation Base for next year. In the given
question, excess of Rs. 44,000 of Pool B and Rs. 480,000 of Pool C during IY 20X1/X2 is considered
for computing Opening Depreciation Base of Pool B and Pool C respectively for IY 20X2/X3.
Chapter 11- Income from Business

Question 1- Page 110

a. Calculation of Depreciation Allowance


S.N. Particulars Pool A Pool B Pool C Pool D Total
I Opening Depreciation Base 3,000,000 500,000 1,500,000 700,000 5,700,000
II Absorbed additions - 300,000 333,333 - 633,333
Up to Poush (100% of Cost) - 300,000 300,000
From Magh to Chaitra (2/3 of
Cost) 333,333 333,333
From Baisakh to Ashad (1/3 of
Cost) -
III Disposal Proceeds 50,000 175,000 225,000
IV Depreciable Basis (I + II - III) 3,000,000 750,000 1,658,333 700,000 6,108,333
V Rate of Depreciation 5% 25% 20% 15%
VI Depreciation Amount (IV X V) 150,000 187,500 331,667 105,000 774,167

b. Had it been the special industry (assuming XYZ & Co., an entity), the rate of depreciation for Pool A-
D would be accelerated. The depreciation calculation would be as follows:
S.N. Particulars Pool A Pool B Pool C Pool D Total
I Opening Depreciation Base 3,000,000 500,000 1,500,000 700,000 5,700,000
II Absorbed additions - 300,000 333,333 - 633,333
Up to Poush (100% of Cost) - 300,000 300,000
From Magh to Chaitra (2/3 of
Cost) 333,333 333,333
From Baisakh to Ashad (1/3 of
Cost) -
III Disposal Proceeds 50,000 175,000 225,000
IV Depreciable Basis (I + II - III) 3,000,000 750,000 1,658,333 700,000 6,108,333
V Rate of Depreciation 6.67% 33.33% 26.67% 20%
VI Depreciation Amount (IV X V) 200,100 249,975 442,278 140,000 1,032,353

Question 2- Page 110

S.N. Particulars Pool A


I Opening Depreciation Base 250,000
II Absorbed additions 27,500
Up to Poush (100% of Cost) 22,500
From Magh to Chaitra (2/3 of
Cost)
From Baisakh to Ashad (1/3 of
Cost) 5,000
III Disposal Proceeds 275,000
IV Depreciable Basis (I + II - III) 2,500
V Rate of Depreciation 25%
VI Depreciation Amount (IV X V) 625
Since the opening depreciation base for next year is higher than Rs. 2,000 (1875+10,000- unabsorbed
additions); the depreciation for the year is Rs. 625.

Question 3- Page 110

As per Sec. 119 (1), depreciation is allowable when all the following conditions are satisfied:

a. Ownership of Asset: The asset shall be owned by the person who is claiming such depreciation.
Chapter 11- Income from Business

b. Use of Asset during the Income Year: The asset shall be used during the Income Year.
c. Use of Asset for Income Generation: The asset shall be used to generate income from business, i.e.
the pool shall be used to generate income from business.
d. Nature/Type of Asset: The asset shall be a depreciable asset.

Since the depreciable asset is owned by Ms. Saloni and used in business for income generation during
the Income Year, the depreciation on such asset is allowable for tax purpose.

Question 4- Page 110

Condition 1:

S.N. Particulars Pool B


I Opening Depreciation Base 85,000
II Absorbed additions -
Up to Poush (100% of Cost) -
From Magh to Chaitra (2/3 of Cost) -
From Baisakh to Ashad (1/3 of Cost) -
III Disposal Proceeds 100,000
IV Depreciable Basis (I + II - III) (15,000)
V Rate of Depreciation 25%
VI Depreciation Amount (IV X V)
In this case, since the disposal proceeds exceed sum of opening depreciation base and absorbed
additions, there shall be gain on disposal of depreciable asset and the amount is charged to income as
per Sec. 7 (2) (d) of the Act.

Condition 2:

S.N. Particulars Pool B


I Opening Depreciation Base 85,000
II Absorbed additions -
Up to Poush (100% of Cost) -
From Magh to Chaitra (2/3 of Cost) -
From Baisakh to Ashad (1/3 of Cost) -
III Disposal Proceeds 30,000
IV Depreciable Basis (I + II - III) 55,000
V Rate of Depreciation 25%
VI Depreciation Amount (IV X V) 13,750

Question 5- Page 110

Please refer to Page 109 (specific cases of depreciation) of the book.

Question 6- Page 110

The readers shall note requirement of the Act to treat different assets of Pool E in different block and
calculate allowable depreciation separately for each block of asset. Similarly, it should also be noted that
the method of depreciation is Straight Line and rate shall be determined using useful life of asset. The
useful life shall be rounded off to nearest half year.
Chapter 11- Income from Business

In this solution, round off to nearest half year is computed using mathematical logic.

Particulars Pool E1 “AS” Pool E2 “IMS”


Cost of Asset 5 Million 3.15 Million
Life of asset 10 years 5 months 10 years 6 months
Life rounded off to nearest half year 10.5 years 10.5 years
Rate of Depreciation 100/10.5% 100/10.5%
Absorbed Additions 5 Million 1.05 Million
(as is purchased in (as is purchased in Baisakh 20X2)
Shrawan)
Depreciation (Absorbed Rs. 476,190 100,000
Additions * rate- for first year)
Cost * Rate- for subsequent
years

Question 7- Page 110

Note: Furniture is in Pool B and ATM & Generator are in Pool D, and considered accordingly while
working for additions of assets.

A. Calculation of Depreciation for Pool A to Pool D:

S.N. Particulars Pool A Pool B Pool C Pool D Total


I Opening Depreciation Base 120,000,000 5,550,000 12,000,000 13,500,000 151,050,000
II Absorbed additions - 1,966,667 - 4,200,000 6,166,667
Up to Poush (100% of Cost) - 500,000 2,000,000 2,500,000
From Magh to Chaitra (2/3 of Cost) 800,000 - 1,333,333 2,133,333
From Baisakh to Ashad (1/3 of
Cost) 666,667 866,667 1,533,333
III Disposal Proceeds 3,500,000 200,000 3,700,000
IV Depreciable Basis (I + II - III) 116,500,000 7,316,667 12,000,000 17,700,000 153,516,667
V Rate of Depreciation 5% 25% 20% 15%
VI Depreciation Amount (IV X V) 5,825,000 1,829,167 2,400,000 2,655,000 12,709,167

B. Calculation of Depreciation for Pool E


1. The depreciation of Opening Depreciation of Pool E1 is to be amortized for last installments,
which means- the depreciation is Rs. 1,236,000
2. There are two additions of Leasehold improvements in the question, both from Magh to Chaitra.
Though leasehold improvements are of permanent nature and falls under Pool A as per Schedule
2 of the Act, however Income Tax Manual has allowed an alternative treatment of leasehold
improvement by allowing amortizing the improvements over useful life including it in Pool E. As
Income Tax Authority is compelled to act as per its circulars and directives, and the matter is
beneficial to taxpayer, the solution is prepared by considering the alternative treatment allowed by
Income Tax Manual.
Particulars Pool E2 “Leasehold Pool E2 “Leasehold
improvement useful life 60 improvement useful life 120
months” months”
Cost 3,600,000 6,000,000
rd
Absorbed Additions (2/3 of 2,400,000 4,000,000
Cost)
Rate of Depreciation 100/5% 100/10%
Chapter 11- Income from Business

Depreciation 480,000 400,000


3. Total Depreciation of Pool E= Depreciation of Pool E1+ Depreciation of Pool E2+ Depreciation of
Pool E3= 2,116,000
C. Calculation of Total Depreciation for the Year
Total Depreciation for the year = 14,825,167

D. Calculation of Eligible Repair and Improvement expenses


S.N. Particulars Pool A Pool B Pool C Pool D Total
I Depreciable Basis 116,500,000 7,316,667 12,000,000 17,700,000 153,516,667
II 7% of I 8,155,000 512,167 840,000 1,239,000 10,746,167
III Actual Repair & Improvement Exp - 500,000 550,000 600,000 1,650,000
IV Allowable R & I u/s 16 - 500,000 550,000 600,000 1,650,000
V Excess (III- IV) - - - - -
Since the leasehold building is not owned, the repair of such building cannot be claimed as per Sec.
16 of the Act. However, it is a contractual obligation and is paid for the business purpose; thus, the
amount is allowable u/s 13 of the Act. Hence, the repair and maintenance expense of leasehold
building of Rs. 500,000 is allowable u/s 13.

Question 8- Page 111

Calculation of Opening Depreciation base of replaced asset:

Particulars Amount
Year 20X0/X1- Absorbed Additions 133,333,333 (400,000,000X1/3)
Depreciation for Year 20X0/X1 26,666,667 (133,333,333 X 20%)
Depreciation Base for 20X1/X2 373,333,333 (400,000,000-26,666,667)
Depreciation for Year 20X1/X2 74,666,667
Depreciation Base for 20X2/X3 298,666,667
Depreciation for Year 20X2/X3 59,733,333
Depreciation Base for 20X3/X4 238,933,333
Depreciation for Year 20X3/X4 47,786,667
Depreciation Base for 20X4/X5 191,146,667
Depreciation for Year 20X4/X5 38,229,333
Depreciation Base for 20X5/X6 152,917,333
Depreciation for Year 20X5/X6 30,583,467
Opening Depreciation Base for 20X6/X7 122,333,867

Calculation of Depreciation u/s 19 and Schedule 2 of the Act:

(in Millions)

Pool D
S.
Particulars Pool A Pool B Pool C Replaced Other Total
N.
Assets Assets
I Opening Depreciation Base 16,750.00 425.00 333.00 122.33 14,627.67 32,258.00
II Absorbed additions 6.67 10.00 - - 333.33 350.00
Up to Poush (100% of
Cost) - -
From Magh to Chaitra (2/3
of Cost) 10.00 - 333.33 343.33
From Baisakh to Ashad 6.67 6.67
Chapter 11- Income from Business

(1/3 of Cost)
III Disposal Proceeds 2.00 2.00
Depreciable Basis (I + II -
IV III) 16,756.67 435.00 331.00 122.33 14,961.00 32,606.00
V Rate of Depreciation 6.67% 33.33% 26.67% 100.00% 20.00%
Depreciation Amount (IV
VI X V) 1,117.67 144.99 88.28 122.33 2,992.20 4,465.47
Chapter 11- Income from Business

Question 1- Page 111

Statement of Assessable Income from Business of M/s B&B Cement P. Ltd:

Particulars Sec. Refer Amounts


Ref. W.N.
Amounts to be included u/s 7 of the Act:
Sales 7.2 1 35,660,000
Interest Income- assumed directly related to business objective of the 7.2 15,000
person
Bad debts recovered 25 1.1 -
Total Inclusions in Income (A) 35,675,000
Allowable Deductions
Interest Expenses 14 2 2,900,000
Cost of Trading Stock 15 3 24,540,000
Repair & Improvement Expenses 16 4 -
Pollution Control Cost 17 5 -
Research & Development Cost 18 5 -
Depreciation as per Income Tax Act 19 6 695,295
Salary 13 7 2,500,000
Rent 13 7 1,520,000
Vehicle Running Expenses 13 7 275,000
Insurance 13 7/8 143,000
Other Expenses 13 7/9 1,825,000
Total Deductions (B) 34,398,295
Assessable Income from Business (A-B) 1,276,705

Statement of Total Assessable Income & Taxable Income

Section Refer
Particulars Amount
Reference W. N.
Assessable Income from Business 1,276,705
Assessable Income from Investment 10 -
Assessable Income from Windfall Gain 10 -
Total Assessable Income 1,276,705
Less: Allowable Deductions from Total Assessable Income
to arrive at Taxable Income
A. Contribution to PM Relief Fund & National
Reconstruction Fund of GON 12B 11 -
B. Expenditure u/s 12A 12A 11 -
C. Donation u/s 12 (3) 11 -
D. Donation u/s 12 (1) & 12 (2) 11 -
Taxable Income 1,276,705

Statement of Tax Liability:

The corporate tax rate for Special Industry is 20%, in absence of information supporting applicability of
other concessions u/s 11 of the Act.

Tax Liability= taxable income * Corporate Tax rate = 1,276,705 * 20%= Rs. 255,341

Working Notes:
Chapter 11- Income from Business

1. As the IRD has applied Transfer Pricing Rules (Sec. 33) to determine the Arm’s Length Transaction,
the contention of IRD seems correct. It is assumed that the company has accepted the IRD’s verdict
and thus, the sales amount is adjusted to include Arm’s Length pricing to its sister concern. The
readjusted sale is Rs. 35,660,000
1.1. Since bad debts expenses at the time of booking in books were not allowed as deduction, the
recovery of bad debt is also not included in income.
2. B&B Cement P. Ltd. is a resident entity controlled by Exempt Organization and falls within the
purview of Sec. 14 (2) as 31% of its interest is held by a nonresident person, which is 25% or more of
the total interest of the company. In this case, the eligible interest expenses are sum of interest to
controlling person to the extent of limit by Sec. 14 (2) and other eligible interest expenses.
It is assumed that rest of the interest expenses of Rs. 2,270,000 is incurred on loan utilized for
business purpose and obtained from an unrelated party, which is totally deductible.
Calculation of Eligible Interest Expenses u/s 14 (2)
Lower of Following:
A. Interest paid to Controlling Entity 630,000
B. Sum of Following 2,095,853
a. Interest Income 15,000
b. 50% of ATI calculated without including interest income 2,080,853
& interest expenses (refer calculation below)
C. Eligible (Lower of A or B) 630,000
D. Total Eligible 2,900,000
Calculation of ATI for Sec. 14 (2) & 50% of ATI- based on interpretation of Income Tax Manual
Particulars Amount Notes
Inclusions:
Sales 35,660,000 As included in Assessable Income Calculation
Interest Income - Cannot be included as per Sec. 14 (2)
Recovery of Bad Debt - As included in Assessable Income Calculation
Total Inclusions (A) 35,660,000
Deductions
Interest Expenses - Cannot be deductible u/s 14 (2)
Cost of Trading Stock 24,540,000 As included in Assessable Income Calculation
Repair & Improvement Expenses - As included in Assessable Income Calculation
Pollution Control Cost - Total incurred during the year as per Income Tax
Manual
Research & Development Cost - Total incurred during the year as per Income Tax
Manual
Depreciation 695,295 As included in Assessable Income Calculation
Other Expenses (general 6,263,000 As included in Assessable Income Calculation
deductions)
Total Deductible expenses (B) 31,498,295
ATI (A – B) 4,161,705
50% of ATI 2,080,853

3. In absence of further information, the cost of trading stock is assumed to be equal to the Cost of
Goods sold as appeared in the question.
4. In absence of information, it is assumed that there are no repair and improvement expenses.
5. In absence of information, it is assumed that expenditure for Pollution Control & Research &
Development has not been incurred.
Chapter 11- Income from Business

6. Calculation of Depreciation
Since the company has accepted the Assessment Order issued by IRD u/s 102 of the Act, the
company is deemed to accept the charge of increasing the asset by purchasing a fraudulent VAT
invoice. As such, the depreciation for the year shall be based on the restated opening balance:
Impact of Fraudulent Invoice in Opening Balance
As the capitalization of the assets was made on Shrawan of 20X1, the depreciation on such assets
was charged for the whole year.
Particulars Pool A Pool B Pool C
A. Depreciation not deductible 10,000 20,000 30,000
B. Depreciation Rate 5% 25% 20%
Since accelerated rate was not used last year
C. Underlying base of the Asset (A/B) 200,000 80,000 150,000
D. Impact on Opening Balance 190,000 60,000 120,000

Restated Opening WDV of Assets


Particulars Pool A Pool B Pool C Pool D
A. Opening Balance (Given) 300,000 1,600,000 400,000 500,000
B. Impact of Fraudulent bill in Opening Balance (190,000) (60,000) (120,000)
C. Restated Opening Balance (A – B) 110,000 1,540,000 280,000 500,000

Since there are no additions or disposal of assets, the depreciation base is equal to the Restated
Opening Balances, and the depreciation is calculated accordingly.
Particulars Pool A Pool B Pool C Pool D
A. Restated Opening Balance (Dep. Base) 110,000 1,540,000 280,000 500,000
B. Depreciation Rate 6.67% 33.33% 26.67% 20%
As the company uses accelerated rate
C. Depreciation 7,337 513,282 74,676 100,000
The total of above = Rs. 695,295

Since the company has not used the accelerated rate until last previous year, there may be chance of
decreasing the depreciation base by the tax authority; as in practice the accelerated rate is made
mandatory, though the Act uses the word “the tax payer may” instead of “the taxpayer shall” for such
benefits. As the tax reassessment of this company is final and the tax officer has not raised any issue
concerning that, it is assumed that there is no chance of reduction on the Opening Depreciation Base.
6.1. Since it is Cement Industry, means a production based industry specified in Sec. 3 (a) of Industrial
Enterprises Act, 2049. This implies it is a special industry which is eligible for accelerated
rd
depreciation, i.e. 1/3 accelerated rate on normal depreciation rate.
7. It is assumed that these expenses satisfy all the conditions laid down by Sec. 13 of the Act, i.e.
incurred by the cement industry to generate income from business during the Income Year.
8. Since it is a company as per the definition of Income Tax Act, as per Sec. 22 it must follow accrual
basis of accounting, which means prepaid expenses are not allowed in this year. Thus, total
insurance expenses claimable is Rs. 143,000 (paid less prepaid)
9. It is assumed that the director is a non executive director. Since, the company has made payment for
personal expenses of director amounting to Rs. 100,000; as per Sec. 53, the payment tantamount to
distribution of profit and dividend tax is applicable as per Sec. 54.
Similarly as per Sec. 21 (1) (e), distribution of profit is not deductible for tax purpose, it means the
other expenses of Rs. 100,000 is not deductible, i.e. Rs. 1,825,000 is only deductible
Commission to marketing agent is normal business expenditure, and deductible u/s 13. There is no
any provision in Income Tax Act to disallow any expenditure merely on the basis of failure to withhold
Chapter 11- Income from Business

tax. Failure to withhold tax on any payment attracts fees and interests under Chapter 22 of the Act,
but the expenditures are allowed for tax purpose.
10. In absence of information, it is assumed that there are no other incomes.
11. In absence of information, it is assumed that there is no contribution to PM Relief Fund, or National
Reconstruction Fund of GON or Expenditure u/s 12A or donations u/s 12.
Chapter 11- Income from Business

Question 2- Page 112

Statement of Assessable Income from Business of M/s Gorkhali Cement P. Ltd:

Particulars Sec. Refer Amounts


Ref. W.N.
Amounts to be included u/s 7 of the Act:
Sales 7.2 31,000,000
Other Income – Interest Income directly related to the business objective 7.2 1 588,235
of the person
Total Inclusions in Income (A) 31,588,235
Allowable Deductions
Interest Expenses 14 2 2,000,000
Cost of Trading Stock 15 3 17,146,000
Repair & Improvement Expenses 16 4 400,000
Pollution Control Cost 17 5 -
Research & Development Cost 18 5 -
Depreciation as per Income Tax Act 19 6 9,068,000
Administrative Expenses 13 7 2,000,000
Selling & Distribution Expenses 13 7 600,000
Total Deductions (B) 31,214,000
Assessable Income from Business (A-B) 374,235

Statement of Total Assessable Income & Taxable Income

Section Refer
Particulars Amount
Reference W. N.
Assessable Income from Business 374,235
Assessable Income from Investment 8 -
Assessable Income from Windfall Gain 8 -
Total Assessable Income 374,235
Less: Allowable Deductions from Total Assessable Income
to arrive at Taxable Income
A. Contribution to PM Relief Fund & National
Reconstruction Fund of GON 12B 9 -
B. Expenditure u/s 12A 12A 9 -
C. Donation u/s 12 (3) 9 -
D. Donation u/s 12 (1) & 12 (2) 10 -
Taxable Income 374,235

Statement of Tax Liability:

The corporate tax rate for Special Industry is 20%, in absence of information supporting applicability of
other concessions u/s 11 of the Act.

Tax Liability= taxable income * Corporate Tax rate = 374,235 * 20%= Rs. 74,847
Chapter 11- Income from Business

Working Notes:

1. Interest income received by an entity is subject to 15% withholding tax, which means Rs. 500,000 is
85% of the actual interest income. Grossing up the same, the interest income is 588,235.
2. It is assumed that the construction of factory building is completed, and used during the year
20X1/X2, implying that the interest for the year is deductible u/s 14 (1).
3. Calculation of Cost of Trading Stock:
Factors to be considered:
• Cost of Trading Stock is sum of Value of Opening Stock and the cost of production of the year
less Value of Closing Stock.
• Closing Stock is valued at Cost or Market value, whichever is lower.
• Cost of Production is determined by using absorption costing method, as it is company following
Accrual basis of accounting; under which the components of cost are direct material, direct labor,
other direct expenses and factory overhead. Factory overhead does not include repair and
improvement expenses and depreciation of depreciable asset.
Opening Stock 700,000
Less: Repair & Improvement of Depreciable asset included (14,000)
(Assumed that Repair is of depreciable asset- 2% of valuation)
Restated Opening Balance as per Income tax law 686,000
Add: Cost of Production
Cost of clinker and other raw materials 18,000,000
Salary & wages for Factory (assumed Wages) 1,200,000
Repair & Maintenance (assumed of Depreciable asset) -
Other Overhead expenses (assumed factory overhead) 200,000 19,400,000
Less: Value of Closing Stock as per Income Tax
Accounting Value of Closing Stock 3,000,000
Less: Repair & Improvement included in it- 2% of valuation (60,000)
(assumed R&I be of Depreciable Asset & not for trading stock)
Restated Value of Closing Stock as per Income Tax Act (2,940,000)
Cost of Trading Stock 17,146,000
4. It is assumed that the repair and improvement expenses included in the cost of production are the
only repair and improvement expenses, which means the repair is of Plant & Machinery, i.e. Pool D
Asset.
A. Depreciation Base of Pool D 30,000,000
B. 7% of Above 2,100,000
C. Actual Expenses 400,000
D. Eligible (Lower of “B” or “C”) 400,000
The repair expenses included in Closing Stock is also part of Rs. 400,000.
5. In absence of information, it is assumed that expenditure for Pollution Control & Research &
Development has not been incurred.
6. Calculation of Depreciation
Particulars Pool A Pool B Pool D Total
Opening Depreciation Base Not given
Absorbed Additions 40,000,000 Not Given
Disposal Proceeds Not given
Depreciation Base 40,000,000 1,200,000 (given) 30,000,000
(given)
Depreciation Rate (Note 6.1) 6.67% 33.33% 20%
Depreciation 2,668,000 400,000 6,000,000 9,068,000
Chapter 11- Income from Business

6.1. Since it is Cement Industry, means a production based industry specified in Sec. 3 (a) of Industrial
Enterprises Act, 2049. This implies it is a special industry which is eligible for accelerated
rd
depreciation, i.e. 1/3 accelerated rate on normal depreciation rate.
7. It is assumed that these expenses satisfy all the conditions laid down by Sec. 13 of the Act, i.e.
incurred by the cement industry to generate income from business during the Income Year.
8. In absence of information, it is assumed that there are no other incomes.
9. In absence of information, it is assumed that there is no contribution to PM Relief Fund, or National
Reconstruction Fund of GON or Expenditure u/s 12A or donations u/s 12.
Chapter 11- Income from Business

Question 3- Page 113

Statement of Assessable Income from Business of M/s ABC P. Ltd:

Particulars Sec. Refer Amounts


Ref. W.N.
Amounts to be included u/s 7 of the Act:
Sales 7.2 9,050,000
Miscellaneous Income 7.2 1 100,000
Total Inclusions in Income (A) 9,150,000
Allowable Deductions
Interest Expenses 14 2 200,000
Cost of Trading Stock 15 3 5,500,000
(Raw Materials Consumed+ Decrease in Finished Goods Stock)
Repair & Improvement Expenses 16 4 50,000
Pollution Control Cost 17 5 -
Research & Development Cost 18 5 -
Depreciation as per Income Tax Act 19 6 258,333
Administrative Expenses 13 7 1,210,000
Total Deductions (B) 7,218,333
Assessable Income from Business (A-B) 1,931,667

Statement of Total Assessable Income & Taxable Income

Section Refer
Particulars Amount
Reference W. N.
Assessable Income from Business 1,931,667
Assessable Income from Investment 8 -
Assessable Income from Windfall Gain 8 -
Total Assessable Income 1,931,667
Less: Allowable Deductions from Total Assessable Income
to arrive at Taxable Income
A. Contribution to PM Relief Fund & National
Reconstruction Fund of GON 12B 9 -
B. Expenditure u/s 12A 12A 9 -
C. Donation u/s 12 (3) 9 -
D. Donation u/s 12 (1) & 12 (2) 10 96,583
Taxable Income 1,835,084

Statement of Tax Liability:

The corporate tax rate for XYZ Co. Ltd. is assumed to be 25%.

Tax Liability= taxable income * Corporate Tax rate = 1,835,084 * 25%= Rs. 458,771
Chapter 11- Income from Business

Working Notes:

1. Amount received in connection to acceptance of restriction related to business also form part of
business income.
2. In absence of information, it is assumed that the company is not a “resident entity controlled by
exempt organization” as clarified by Sec. 14, and the interest expenditure is deductible u/s 14 of the
Act.
It is assumed that Managing Director’s Office was put to use during year, which qualifies the claim of
interest expenses for tax purpose. Since, the renovation of managing director’s room serves proper
business purpose, the interest expenditure is deductible.
3. In absence of further information, it is assumed that the Cost of Sales in the question and cost of
trading stock calculated as per Sec. 15 of the Act are same.
4. Calculation of Eligible Repair & Improvement Expenses:
A. Depreciation Base (given in Question) 850,000
B. 7% of A above 59,500
C. Actual Repair & Improvement Expenses 50,000
D. Eligible u/s 16 (Lower of “B” or “C”) 50,000
5. In absence of information, it is assumed that there is no Pollution Control Cost & Repair and
Improvement Cost.
6. Calculation of Adjusted Depreciation as per Sec. 19 & Schedule 2 of Act:
Depreciation in Question 275,000
th
Less: Depreciation for Equipment used since 15 Baisakh (25,000)
Add: Eligible Depreciation of Equipment as per ITA 8,333
(one-third of 25% of 100,000)
Total Eligible Depreciation 258,333

6.1. Office Equipments fall under Pool B of Depreciable Asset; and the entity is not entitled to accelerated
depreciation. As such, the rate of Depreciation is 25% and since it’s added in the Pool in Baisakh,
one-third of the cost is absorbed additions.
7. It is assumed that the administrative expenses are incurred by ABC P. Ltd. for the purpose of
generating income from business during the Income year; which satisfies all the conditions laid down
by Sec. 13 of the Act.
Eligible Administrative Expenses u/s 13
Expenses Given in Question 1,360,000
Less: Repair & Improvement Expense included in it (50,000)
Less: Donation included in it (100,000)
Eligible Administrative Expenses 1,210,000
8. In absence of information, it is assumed that there are no other incomes.
9. In absence of information, it is assumed that there is no contribution to PM Relief Fund, or National
Reconstruction Fund of GON or Expenditure u/s 12A or notified donations u/s 12 (3).
10. Eligible Donation Expenditure- Donation to Exempt Organization
Minimum of Following:
a. Actual Donation 100,000
b. 5% of ATI 96,583
(ATI = Total Assessable Income – Contribution u/s 12B – Expenditure u/s 12A)
i.e. 5% of 1,931,667
c. Maximum 100,000

Therefore, eligible = Rs. 96,583


Chapter 11- Income from Business

Question 4- Page 113

The solution in this problem to calculate Assessable Income from Business is done on “Add back” basis
as detailed information as to the expenditures are not provided. Inland Revenue Authority has not
developed any such form to present the assessable income through indirect method, as such.

Books of KBC Bank Ltd


Calculation of Assessable Income from Business

Amount
Particulars Notes
(Rs.)
Net Profit Before Tax 100,000,000
Add:
Loan Loss Provision in the books 15,000,000 Loan loss provision are allowed u/s 59 and
treated separately (refer W. N. 1 for calculation of
LLP for accounting purpose)
Provision for Investment Loss 1,000,000 Investment losses, at the time of disposal of
investments, are allowed but not the provisions
against possible investment losses.
Fine paid to SEBON 100,000 Fine paid against non compliance of any statute
of any country is not deductible for tax purpose
u/s 21 (1) (b) of the Act.
Payment for Software 1,000,000 It is separately claimed as per Sec. 19 of the Act
Amortization of Pre-operating exp 500,000 Preoperating expenses are deductible u/s 13 in
the first year of operation; as such, the
amortizations are not deductible for tax purpose.
Sponsorship for sporting event 500,000 Is separately claimed u/s 12A
Donation to Cancer Relief Society 200,000 Is separately claimed u/s 12
Less:
Loan Loss Provision u/s 59 10,000,000 W.N. 5
Depreciation of Software 111,111 W. N. 6
Assessable Income from Business 108,188,889

Statement of Total Assessable Income & Taxable Income

Section Refer
Particulars Amount
Reference W. N.
Assessable Income from Business 91,811,111
Assessable Income from Investment 7 -
Assessable Income from Windfall Gain 7 -
Total Assessable Income 108,188,889
Less: Allowable Deductions from Total Assessable Income
to arrive at Taxable Income
E. Contribution to PM Relief Fund & National
Reconstruction Fund of GON 12B 8 -
F. Expenditure u/s 12A 12A 9 500,000
G. Donation u/s 12 (3) 8 -
H. Donation u/s 12 (1) & 12 (2) 10 100,000
Taxable Income 107,588,889
Statement of Tax Liability:
Since it is a bank, the applicable corporate tax rate is 30%.
Chapter 11- Income from Business

Tax Liability= taxable income * Corporate Tax rate = 107,588,889 * 30% = Rs. 32,276,667

Calculation of Fees & Interest u/s 117, 118 & 119 of the Act:

It is assumed that the tax return is filed on time, and final corporate tax liability was deposited on due
date. It is further assumed that estimated tax return was also filed on due date. Hence, fees u/s 117 and
interest u/s 119 is not applicable. Hence, we need to check the applicability of Sec. 118:

• Check whether interest u/s 118 is applicable.


Particulars Amount to be deposited Amounts actually Remarks
deposited
Up to Poush end 90% of 40% of actual Tax liability = 12,000,000 No Interest
11,619,600
Up to Chaitra end 90% of 70% of Actual tax Liability = 21,000,000 No Interest
20,334,300
Up to Ashad end 90% of 100% of Actual tax Liability= 30,000,000 No Interest
29,049,000
Since amount actually deposited is greater than the amount to be deposited, there is no application of
interest u/s 118 of the Act.

Working Notes:

1. Loan Loss provision charged to Profit & Loss


Closing Total LLP 75,000,000
Less: Opening balance of Total LLP 50,000,000
Total LLP charged to Income Statement (net of write backs) 25,000,000
2. Since Staff gratuity has been paid to retirement fund, as per the interpretation in Example 8.4.3 of
Income Tax Manual 2066 (Updated 2068), gratuity provision deposited in the approved retirement
fund on the basis of the exact name of the employees without any recourse of such investment to the
employer are eligible to be deducted for tax purpose. As such, no treatment has been made to staff
gratuity paid to Approved Retirement Fund.
3. The repair of office building taken on lease is deductible u/s 13 of the Act. As such, no treatment of
such expenditure is done while calculating assessable income from business.
4. Staff welfare expenses as part of reimbursement of personal tour expenses are deductible u/s 13 of
the Act so far the expenses are included while calculating income of the person getting such
reimbursement; since Definition of Personal & Domestic Nature Expenses exclude any such
expenses when it is included in assessable income. (Refer Page 89 of the book for further reference)
5. Calculation of Eligible Loan Loss Provision u/s 59
As per Sec. 59 (1Ka), Loan loss provision is eligible to the extent of 5% of Gross Loan Outstanding as
on Year end date. The calculation is given below:
A. Gross Loan Outstanding 1,200,000,000
B. 5% of Above 60,000,000
C. Total Loan Loss Provision till year end of Income Year 75,000,000
D. Eligible Expenses (Lower of B Or C Above) 60,000,000
E. Claim up to Previous Year 50,000,000
F. Claim for the Year (D – E) 10,000,000

The question has not provided the information as to the total loan loss provision claim up to previous
Income year. Since 5% Total Loan (Rs. 50,000,000) is less than total Loan loss Provision (Rs.
Chapter 11- Income from Business

60,000,000) in the immediately preceding Income Year, it is assumed that Loan loss provision to the
extent of 5% of Loan Outstanding up to previous income year has been claimed.

The above calculations are based on the interpretation of Income Tax Manual 2066 (Updated 2068).
Multiple interpretations prevail.

6. Calculation of Depreciation of Software u/s 19 & Schedule 2 of the Act


rd
Absorbed Additions of Software (1/3 of Cost) 333,333
Depreciation Rate (100/3%)
Depreciation 111,111
7. In absence of information, it is assumed that there are no other incomes.
8. In absence of information, it is assumed that there is no contribution to PM Relief Fund, or National
Reconstruction Fund of GON or Expenditure u/s 12A or notified donations u/s 12 (3).
9. Eligible Expenditure u/s 12A
As per Sec. 12A, in case a company with prior approval from IRD incurs expenditure for the purpose
of conservation and promotion of cultural heritage and public infrastructures related to sports, the
minimum of following three is eligible for deduction to arrive at taxable income:
a. Actual Expenditure 500,000
b. 10% of Assessable Income 10,818,889
c. Maximum 1,000,000

Therefore, eligible= 500,000

If we consider this amount as business promotion expenses rather expenditure for promotion of
cultural, religious or ancient heritages or development of physical public amenities related to sports,
the expenditure is deductible u/s 13 of the Act.

10. Eligible Donation Expenditure u/s 12


Minimum of Following:
a. Actual Donation 200,000
b. 5% of ATI 515,945
(ATI = Total Assessable Income – Contribution u/s 12B – Expenditure u/s 12A)
i.e 5% of 10,318,889
c. Maximum 100,000
Therefore, eligible = Rs. 100,000
Chapter 11- Income from Business

Question 5- Page 114

Statement of Assessable Income from Business of M/s ABC P. Ltd:

Particulars Sec. Refer Amounts


Ref. W.N.
Amounts to be included u/s 7 of the Act:
Sales 7.2 8,000,000
Other Income 7.2 50,000
Total Inclusions in Income (A) 8,050,000
Allowable Deductions
Interest Expenses 14 1 300,000
Cost of Trading Stock 15 2 5,000,000
(Raw Materials Consumed+ Decrease in Finished Goods Stock)
Repair & Improvement Expenses 16 3 -
Pollution Control Cost 17 4 600,000
Research & Development Cost 18 4 350,000
Depreciation as per Income Tax Act 19 5 50,000
Administrative Expenses 13 6 500,000
Total Deductions (B) 6,800,000
Assessable Income from Business (A-B) 1,250,000

Statement of Total Assessable Income & Taxable Income

Section Refer
Particulars Amount
Reference W. N.
Assessable Income from Business 1,250,000
Assessable Income from Investment 7 -
Assessable Income from Windfall Gain 7 -
Total Assessable Income 1,250,000
Less: Allowable Deductions from Total Assessable Income
to arrive at Taxable Income
A. Contribution to PM Relief Fund & National
Reconstruction Fund of GON 12B 8 -
B. Expenditure u/s 12A 12A 8 -
C. Donation u/s 12 (3) 8 -
D. Donation u/s 12 (1) & 12 (2) 9 62,500
Taxable Income 1,187,500

Statement of Tax Liability:

The corporate tax rate for ABC P. Ltd. is assumed to be 25%.

Tax Liability= taxable income * Corporate Tax rate = 1,187,500*25%= Rs. 296,875

Working Notes:

1. In absence of information, it is assumed that the company is not a “resident entity controlled by
exempt organization” as clarified by Sec. 14, and the interest expenditure is deductible u/s 14 of the
Act.
2. In absence of further information, it is assumed that the Cost of Sales in the question and cost of
trading stock calculated as per Sec. 15 of the Act are same.
Chapter 11- Income from Business

3. In absence of information, it is assumed that there are no repair expenses.


4. Calculation of Pollution Control Cost & Research and Development Cost:
The calculation below is based on the interpretation of the definition of Adjusted taxable income by
Income Tax Manual 2066 (Updated 2068) issued by IRD:
ATI for ATI for R
Particulars Notes
PCC &D
Sales 8,000,000 8,000,000
Other Income 50,000 50,000
Total Inclusions 8,050,000 8,050,000
Less: Deductions
Interest Expenses Total eligible u/s 14 (1) without applying 300,000 300,000
limit of Sec. 14 (2)
Cost of Trading Stock As eligible for assessable income from 5,000,000 5,000,000
business
Repair & Improvement As eligible for assessable income from - -
Expenses business
Pollution Control Cost Not deductible while calculating ATI for 1,500,000
PCC and deductible at full to calculate ATI
for R&D
Research & Development Deductible at full while calculating ATI for 1,000,000 -
Cost PCC and not deductible to calculate ATI for
R&D
Depreciation As eligible for assessable income from 50,000 50,000
business
Administrative Expenses As eligible for assessable income from 500,000 500,000
business
Donation Not deductible while computing Adjusted - -
Taxable Income
Total Deductions (B) 6,850,000 7,350,000
Adjusted Taxable Income (A-B) 1,200,000 700,000
Eligible Pollution Control Cost:
Pollution Control Cost is eligible to the extent of minimum of following two:
a. Actual Incurred for the year 1,500,000
b. 50% of ATI from all businesses 600,000

Implying that Rs. 600,000 is eligible and the rest of Rs. 900,000 will be considered while determining
Opening Depreciation Base of next year for Pool D Depreciable Asset.

Eligible Research & Development Cost:


Research & Development Cost is eligible to the extent of minimum of following two:
a. Actual Incurred for the year 1,000,000
b. 50% of ATI from all businesses 350,000

Implying that Rs. 350,000 is eligible and the rest of Rs. 650,000 will be considered while determining
Opening Depreciation Base of next year for Pool D Depreciable Asset.

5. It is assumed that the depreciation in question is calculated as per Sec. 19 and Schedule 2 of Income
Tax Act, 2058.
6. It is assumed that the administrative expenses are incurred by ABC P. Ltd. for the purpose of
generating income from business during the Income year; which satisfies all the conditions laid down
by Sec. 13 of the Act.
7. In absence of information, it is assumed that there are no other incomes.
Chapter 11- Income from Business

8. In absence of information, it is assumed that there is no contribution to PM Relief Fund, or National


Reconstruction Fund of GON or Expenditure u/s 12A or notified donations u/s 12 (3).
9. Eligible Donation Expenditure
Minimum of Following:
a. Actual Donation 100,000
b. 5% of ATI 62,500
(ATI = Total Assessable Income – Contribution u/s 12B – Expenditure u/s 12A)
c. Maximum 100,000
Therefore, eligible = Rs. 62,500
Chapter 11- Income from Business

Question 6- Page 114

The tax liability of M/s Buddhishree & Co. is Rs. 200,000 for the year, out of which Rs. 120,000 as TDS
has been deducted and deposited by Anubav & Co. (agent) against rental payments to Buddhishree & Co
(withholdee) as per Sec. 88 of the Act (10% of Rs. 1,200,000).

As per Sec. 93, the amount so withheld and deposited by agent is eligible to be claimed as advance tax
when such tax is not final withholding as per Sec. 92. While going through the list of final withholding
taxes u/s 92 of the Act; the rental income of a firm cannot be a final withholding payment. Thus, the
withholding tax withheld and deposited by the agent is eligible to be claimed as advance tax.

As such:

a. M/s Buddhishree & Co. is required to deposit additional Rs. 80,000 as tax after claiming Rs. 120,000
as advance tax.
b. Yes. If the tax liability was just Rs. 100,000; the advance tax claimable by the firm would be greater
than the total liability, which means no additional cash to be paid against such liability.
c. The implication would be such that the firm cannot claim such withholding tax as advance in the
returns in any income years other than the year when the related incomes are included in tax
assessments. As such, the ability of the firm to claim such amounts as advance tax would cease. The
only recourse would be applying for refund u/s 113 of the Act.
Chapter 11- Income from Business

Question 7- Page 114

Statement of Assessable Income from Business

Particulars Sec. Refer Amounts


Ref. W.N.
Amounts to be included u/s 7 of the Act:
Sales 7.2 500,000
Office Furniture Sales 1 -
Recovery of Bad Debt 25 2 -
Dividend Income 7.3/92 3 -
Total Inclusions in Income (A) 500,000
Allowable Deductions
Interest Expenses 14 -
Cost of Trading Stock 15 4 350,000
Repair & Improvement Expenses 16 -
Pollution Control Cost 17 -
Research & Development Cost 18 -
Depreciation as per Income Tax Act 19 5 45,250
Office Expenses 13 6 16,000
Total Deductions (B) 411,250
Assessable Income from Business (A-B) 88,750

Statement of Assessable Income from Employment

Sec. Refer
Particulars Amount
Ref. W. N.
Gross Salary Received (Net + TDS) 8.2 494,000
Employer’s Contribution to ARF 8.2 100,000
Compensation received from employer 8.2 70,000
Investment insurance paid by employer on his behalf 8 -
Emergency Medical Treatment by Employer 8.2 9 10,000
Reimbursement of Tour Expenses during official visit to Hong Kong 8.3 10 -
Retirement payments 8.3 11 -
Assessable Income from Employment 674,000

Statement of Assessable Income from Investment

Sec. Refer
Particulars Amount
Ref. W. N.
Gain on Disposal of Securities 9.2 100,000
Interest from Personal Deposit in Natural Bank 9.3 12 -
Assessable Income from Investment 100,000

Statement of Total Assessable Income, Taxable Income & Balance Taxable Income

Section Refer
Particulars Amount
Reference W. N.
Assessable Income from Business 88,750
Assessable Income from Employment 674,000
Assessable Income from Investment 100,000
Chapter 11- Income from Business

Assessable Income from Windfall Gain 13 -


Total Assessable Income 862,750
Less: Allowable Deductions from Total Assessable Income
to arrive at Taxable Income
A. Contribution to Approved Retirement Fund 14 250,000
B. Contribution u/s 12 B -
In absence of information, it is assumed there is not
any such contribution
C. Notified Donation u/s 12 (3) -
In absence of information, it is assumed there is no any
such donation
D. Donation u/s 12 (1) & 12 (2) 15 48,488
Taxable Income 921,262
Less: Allowable Reductions from Taxable Income as per 16 -
Schedule 1 (1) of the Act
Balance Taxable Income 921,262

Analysis of Income for application of varying tax Rate:


Balance Taxable Income 921,262
Less: Gain on Disposal of Securities 100,000
Remaining amount of Balance taxable income where normal rate of tax is applicable 821,262
Maximum 10% tax is applicable on Gain on Disposal of Non Business Chargeable Asset, which
means gain on disposal of securities is taxable at maximum rate of 10%, where the rate is 5% when
the securities are listed in recognized stock exchange. As the question is silent about the listing status
of securities, we have assumed the securities are listed.

Statement of Tax Liability


Refer
Particulars Rate Amount
W. N.
st
1 Rs. 250,000 1% 2,500
Next Rs. 100,000 15% 15,000
Next Rs. 471,262 25% 117,816
(821,262-350,000)
Balance Rs. 100,000 5% 5,000
Tax Liability before Tax Credits 140,316
Less: Tax Credits 12 -
Tax Liability after Tax Credits 140,316

Working Notes:

1. Sale of Office Furniture is considered while determining the depreciation pool of Pool B, and ignored
for inclusion.
2. Recovery of bad debts is included in income only if the relevant bad debt expenses were allowed at
the time of writing off of debts. Since the related bad debt expense was not allowed for deduction at
the time of write off, it is not included in income.
3. Dividend received from resident company is final withholding and final withholding income is not
included in income as per Sec. 7 (3) of the Act. As NIC Asia Bank Ltd. is a resident company, the
receipt of dividend is a final withholding payment.
4. Cost of Trading Stock:
Value of Opening Stock 80,000
Add: Purchases 390,000
Chapter 11- Income from Business

Less: Closing Stock Value (120,000)


Cost of Trading Stock 350,000
5. Calculation of Depreciation
Particulars Pool B Pool C Total
A. Opening Depreciation Base 145,000 80,000
(Furniture & Computer) (Motorcycle)
B. Absorbed Additions 7,000 -
(100% cost of Printer)
C. Disposal Proceeds 35,000 - -
(Spare table)
D. Depreciation Base (A+B-C) 117,000 80,000
E. Depreciation Rate 25% 20%
F. Depreciation 29,250 16,000 45,250

6. The gift to one of its customer is personal expenses and not deductible for tax purpose. As such
deductible office expense is Rs. 16,000. It is assumed that other office expense satisfies all the
conditions laid down by Sec. 13 of the Act.
7. Personal drawings are capital repayments and are neither expenses, nor income. Thus, ignored.
8. It is assumed that the insurance premium is deposited by employer after deducting proceeds from the
employee’s income. In case we assume, the premium to be added in addition to the salary paid, the
amount should be included in income.
9. Any discharge or reimbursement of cost incurred by an individual is included in income
10. Any discharge or reimbursement of cost incurred by an individual is included in income, however, if it
serves the proper business purpose of employer; it is not included in income as per Sec. 8 (3). Since
the reimbursement is for official purpose, it serves the proper business purpose of employer.
11. Treatment of Retirement Payments:
th
The retirement benefits accrued up to the date of implementation of Income Tax Act, 2058, i.e. 18
th
Chaitra 2058 is exempted from income tax, which means encashment of un-availed leave up to 18
Chaitra 2058 of Rs. 50,000; gratuity accrued up to the same date of Rs. 400,000 and payment from
approved retirement fund out of contribution up to the same date of Rs. 400,000 is not taxable.
11.1. Encashment of Un-availed Leave:
th
Un-availed leave encashed for such leave accrued after 18 Chaitra 2058 is subject to 15% final
withholding payments as per Sec. 88 and 92 of the Act. It means Rs. 170,000 is taxable at 15%
final withholding.
11.2. Payment of Gratuity and Provident Fund from Approved Retirement Fund:
As per Sec. 65, Sec. 88 and Sec. 92; retirement payments from approved retirement fund or GON
is subject to 5% tax on gain calculated as per Sec. 65 on final withholding basis.
The tax is calculated as follows:
A. Actual Payments (gratuity plus Provident Fund) 1,300,000
B. Higher of following TWO: 650,000
a. 50% of (A) Above 650,000
b. To the extent 500,000
C. Gain (A – B) 650,000
D. Tax @ 5% 32,500
12. It is assumed that the interest is not received in connection to business, which implies that it is not a
final withholding payment. As per Sec. 9 (3), final withholding payments do not form part of
assessable income.
13. In absence of information, it is assumed that there is no income from windfall gain. Had there been
income from Windfall Gain, it would be final withholding payment.
14. Contribution to Approved Retirement Fund:
Chapter 11- Income from Business

The minimum of following three amounts is eligible for deduction from Total Assessable Income to
arrive at Taxable Income:
A. Actual Contribution across all Approved Retirement Funds 250,000
(Employer’s Contribution + His own Contribution)
rd
B. 1/3 of Total Assessable Income 406,583
rd
(1/3 of 1,219,750)
C. Maximum 300,000
The minimum amount is actual contribution, which is deductible.
15. Gift/donation to Exempt Organization
In case a person makes donation or gives gift to any exempt organization, the minimum of following
three amounts is deductible for tax purpose. A political party registered with Election Commission is
an Exempt Organization. We have assumed that the political party in question is registered with
Election commission:
Eligible Donation:
A. Actual Donation 50,000
B. 5% of ATI 48,488
(5% of {Total Assessable Income- Eligible Contribution to ARF- Contribution u/s 12B)
C. Maximum 100,000
D. Eligible (Minimum of “A” or “B” or “C”) 48,488
16. In absence of information, it is assumed that there is no any reductions availed from taxable income
as per Sec. 1 of Schedule 1 of the Act.
17. Since his spouse is also an earning member, it is not prudent to select Couple for tax Assessment.
Thus, it is assumed the assessment to be Individual not Couple.
18. In absence of information, there are no tax credits.
Chapter 11- Income from Business

Question 8- Page 115

a. As per Sec. 14 (1), interest expenses incurred for the generation of income from business during any
income year is eligible for deduction under the following conditions:
1. In case the interest is accrued/paid on loan which has been utilized for the purpose of
creation/purchase of assets, the asset shall be utilized at least for a point of time during the
Income Year, or
2. In other cases, the loan shall be utilized for business purpose.
Income Tax Manual issued by IRD interpreted the provision to claim the interest expenses of the
entire Income Year, when the asset is used for at least a point of time during the Income Year, which
means if an asset is used for a day during the Income Year, the interest incurred against loan for
financing of such assets is deductible for the entire Income Year.
Based on the above facts, the practice of M/s Simple Innovations P. Ltd. to capitalize the interest
th
incurred till 10 Baisakh is not as per the provisions of the Act, and the tax officer’s contention is
correct.
b. The question is not clear about the nature of repair and improvement expenses. The treatment of
repair and improvement expenses as per Income Tax Act is as follows:
1. Incurred for business asset- becomes part of business asset
2. Incurred for NBCA- treated as part of outgoings for NBCA
3. Incurred for Trading Stock- treated as part of cost of trading stock
4. Incurred for depreciable asset- claimed as per Sec. 16
In this case, it’s been assumed that the repair and improvement expenses in question are incurred for
factory assets, which are depreciable asset; in which case these cannot be part of trading stock.
The tax officer’s contention is correct in this regard.
Chapter 11- Income from Business

Question 9- Page 115

Statement of Assessable Income from Business:

Particulars Sec. Refer Amounts


Ref. W.N.
Amounts to be included u/s 7 of the Act:
Sales 7.2 25,000,000
Miscellaneous Income 7.2 1 5,000
Dividend Income 7.3/92 1.1 -
Total Inclusions in Income (A) 25,005,000
Allowable Deductions
Interest Expenses 14 2 300,000
Cost of Trading Stock 15 3 13,300,000
Repair & Improvement Expenses 16 4 700,000
Pollution Control Cost 17 5 -
Research & Development Cost 18 5 -
Depreciation as per Income Tax Act 19 6 280,000
Other Expenses 13 7 1,400,000
Repair Expenses of Hired Vehicles 13 8 50,000
Government Penalties 21 9 -
Total Deductions (B) 16,030,000
Assessable Income from Business (A-B) 8,975,000

Statement of Total Assessable Income & Taxable Income

Section Refer
Particulars Amount
Reference W. N.
Assessable Income from Business 8,975,000
Assessable Income from Investment 10 -
Assessable Income from Windfall Gain 10 -
Total Assessable Income 8,975,000
Less: Allowable Deductions from Total Assessable Income
to arrive at Taxable Income
A. Contribution to PM Relief Fund & National
Reconstruction Fund of GON 12B 11 -
B. Expenditure u/s 12A 12A 11 -
C. Donation u/s 12 (3) 11 -
D. Donation u/s 12 (1) & 12 (2) 12 100,000
Taxable Income 8,875,000

Statement of Tax Liability:

The corporate tax rate is 25% (as stated in question).

Tax Liability= taxable income * Corporate Tax rate = 8,875,000 * 25%= Rs. 2,218,750

Working Notes:

1. It is assumed that other income is also part of business income.


1.1. Dividend income is deemed to be received from resident company, which implies that it is final
withholding payments and thus, exempted from inclusion in income u/s 7 (3).
Chapter 11- Income from Business

2. In absence of information, it is assumed that the company is not a “resident entity controlled by
exempt organization” as clarified by Sec. 14, and the interest expenditure is deductible u/s 14 of the
Act.
It is assumed that interest is incurred for loan utilized for business.
3. Calculation of Cost of Trading Stock:
Factors to be considered:
• Cost of Trading Stock is sum of Value of Opening Stock and the cost of production of the year
less Value of Closing Stock.
• Closing Stock is valued at Cost or Market value, whichever is lower.
• Cost of Production is determined by using absorption costing method, as it is company following
Accrual basis of accounting; under which the components of cost are direct material, direct labor,
other direct expenses and factory overhead. Factory overhead does not include repair and
improvement expenses and depreciation of depreciable asset.
• Non Refundable duties form part of cost of trading stock as per NAS.

Opening Stock 1,500,000


Add: Cost of Production
Raw Material 11,000,000
Freight 900,000
Custom Duty 1,500,000
Repair & Maintenance (assumed of Depreciable Asset) - 13,400,000
Less: Value of Closing Stock (1,600,000)
Cost of Trading Stock 13,300,000

4. Calculation of Eligible Repair & Improvement Expenses:


It is assumed that repair included in Cost of Stock is within 7% of Depreciation Base of Pool D, in
absence of further information. As such, Rs. 700,000 is allowed as deduction u/s 16.
5. In absence of information, it is assumed that there is no Pollution Control Cost & Repair and
Improvement Cost.
6. Calculation of Depreciation as per Sec. 19 & Schedule 2 of Act:
Since, the information of Plant & Machinery is not given; the depreciation effect of plant & machinery
cannot be calculated. The question is not complete in this regard, in which case the reader may
assume the repair cost to be of trading stock.
In absence of detailed information, depreciation is computed for vehicles only assuming that
accumulated depreciation up to Previous Year was as per Income Tax Act, and not just the
accounting depreciation.
Opening Depreciation Base 1,000,000
Add: Absorbed Additions (100%- as added before Poush) 500,000
Less: Disposal Proceeds (100,000)
Depreciation Base 1,400,000
Depreciation Rate 20%
Depreciation 280,000

7. It is assumed that the Other Expenses are incurred by ABC P. Ltd. for the purpose of generating
income from business during the Income year; which satisfies all the conditions laid down by Sec. 13
of the Act.
Eligible Other Expenses u/s 13
Expenses Given in Question 2,000,000
Chapter 11- Income from Business

Less: Personal Nature telephone Expenses (100,000)


(assuming the amount is not included in individual’s income)
Less: Personal nature travel expenses (500,000)
(assuming the amount is not included in individual’s income)
Eligible Other Expenses 1,400,000
Alternate Solutions:
If it has been assumed that the personal nature expenses were included in the concerned individual’s
income, the amount would have been deductible; as it would be beyond the scope of definition of
Domestic & Personal Nature Expenses.
8. Repair Expenses of hired vehicles are business expenses incurred during the income year by the
person, and hence allowed for deduction as per Sec. 13.
The substance of such expenses is that these expenses are part of hire charges to the owner of
vehicle, as hire charges of operating lease of asset are deductible u/s 13 of the Act, these
expenditures are also deductible under the same provision.
9. As per Sec. 21 (1) (b), penalties paid as a result of non compliance with any statute of any country
are not deductible for tax purpose. It is assumed that the penalties to government are paid due to the
infringement of law of land.
10. In absence of information, it is assumed that there are no other incomes.
11. In absence of information, it is assumed that there is no contribution to PM Relief Fund, or National
Reconstruction Fund of GON or Expenditure u/s 12A or notified donations u/s 12 (3).
12. Eligible Donation Expenditure- Donation to Exempt Organization
It is assumed that the donation is provided to an exempt organization.
Minimum of Following:
a. Actual Donation 200,000
b. 5% of ATI 448,750
(ATI = Total Assessable Income – Contribution u/s 12B – Expenditure u/s 12A)
i.e. 5% of 1,931,667
c. Maximum 100,000

Therefore, eligible = Rs. 100,000

13. Deferred Tax Liabilities are not current tax liabilities and are calculated for accounting purpose for any
taxable or deductible temporary differences.
Chapter 11- Income from Business

Question 10- Page 117

a. Computation of Eligible Pollution Control Cost


As per Sec. 17, the pollution control cost incurred for the operation of business of any person is
eligible during the Income Year to the extent of Minimum of following two:
i. Actual Pollution Control Cost
ii. 50% of Adjusted Taxable Income from all businesses

The following adjusted taxable income from all businesses for the purpose of Sec. 17 is calculated
on the basis of interpretation through an example in Income Tax Manual 2066 (Updated 2068):

Particulars Sec. Refer Amounts


Ref. W.N.
Amounts to be included u/s 7 of the Act:
Sales 7.2 12,300,000
Dividend Received net of Tax 7.3 1 -
Scrap Sales 7.2 140,000
Consulting Income 7.2 450,000
Total Inclusions in Income (A) 12,890,000
Allowable Deductions
Interest Expenses 14 2 3,020,000
Cost of Trading Stock 15 4,510,000
(Raw Materials Consumed+ Decrease in Finished Goods Stock)
Repair & Improvement Expenses 16 3 230,000
Pollution Control Cost 17 4 -
Research & Development Cost 18 5 1,300,000
Depreciation as per Income Tax Act 19 432,000
Salary & Wages 13 6 1,800,000
Administrative Overhead 13 6 600,000
Donation 7 -
Total Deductions (B) 11,892,000
Adjusted Taxable Income for Sec. 17 (A-B) 998,000
50% of Above 499,000
Therefore, eligible = lower of Rs. 690,000 or 499,000 = 499,000

b. Computation of Eligible Research & Development Cost


As per Sec. 18, the Research & Development cost incurred for the operation of business of any
person is eligible during the Income Year to the extent of Minimum of following two:
i. Actual Research & Development Cost
ii. 50% of Adjusted Taxable Income from all businesses

The following adjusted taxable income from all businesses for the purpose of Sec. 18 is calculated
on the basis of interpretation through an example in Income Tax Manual 2066 (Updated 2068):

Particulars Sec. Refer Amounts


Ref. W.N.
Amounts to be included u/s 7 of the Act:
Sales 7.2 12,300,000
Dividend Received net of Tax 7.3 1 -
Scrap Sales 7.2 140,000
Consulting Income 7.2 450,000
Total Inclusions in Income (A) 12,890,000
Allowable Deductions
Chapter 11- Income from Business

Interest Expenses 14 2 3,020,000


Cost of Trading Stock 15 4,510,000
(Raw Materials Consumed+ Decrease in Finished Goods Stock)
Repair & Improvement Expenses 16 3 230,000
Pollution Control Cost 17 4 690,000
Research & Development Cost 18 5 -
Depreciation as per Income Tax Act 19 432,000
Salary & Wages 13 6 1,800,000
Administrative Overhead 13 6 600,000
Donation 7 -
Total Deductions (B) 11,282,000
Adjusted Taxable Income for Sec. 18 (A-B) 1,608,000
50% of Above 804,000
Therefore, eligible = lower of Rs. 1,300,000 or 804,000 = 804,000

c. Computation of Eligible Interest Expenses


It is assumed that JICA and DFID are associated persons. As such, 60% of the interest of M/s Solar
Power P. Ltd. is held by an associated person of an exempt entity, implying the company to qualify as
“Resident Entity controlled by Exempt Organization” and thereby, the provisions of Sec. 14 (2) are
applicable while claiming deduction of interest paid to JICA by the company.
As per Sec. 14 (2), interest paid to a controlling person by resident entity controlled by exempt
organization shall be deductible to the extent of minimum of the following:
i. Actual interest paid to the controlling entity, or
ii. Sum of following:
a. Interest Income, and
b. 50% of ATI calculated without including interest income and interest expenses
Calculation of ATI, and maximum limit for Sec. 14 (2)

Particulars Sec. Refer Amounts


Ref. W.N.
Amounts to be included u/s 7 of the Act:
Sales 7.2 12,300,000
Dividend Received net of Tax 7.3 1 -
Scrap Sales 7.2 140,000
Consulting Income 7.2 450,000
Total Inclusions in Income (A) 12,890,000
Allowable Deductions
Interest Expenses 14 2 -
Cost of Trading Stock 15 4,510,000
(Raw Materials Consumed+ Decrease in Finished Goods Stock)
Repair & Improvement Expenses 16 3 230,000
Pollution Control Cost 17 4 690,000
Research & Development Cost 18 5 1,300,000
Depreciation as per Income Tax Act 19 432,000
Salary & Wages 13 6 1,800,000
Administrative Overhead 13 6 600,000
Donation 7 -
Total Deductions (B) 9,562,000
Adjusted Taxable Income for Sec. 14 (2) (A-B) 3,328,000
50% of Above 1,664,000
Therefore, eligible interest expenses u/s 14 (2) = Lower of Rs. 2,950,000 or Rs. 1,664,000 =
1,664,000
Chapter 11- Income from Business

Total Eligible Interest Expenses = Eligible exp u/s 14 (1) + Eligible Exp u/s 14 (2) =
70,000+1,664,000= 1,734,000
d. Computation of Corporate Tax Liability
Statement of Assessable Income from Business

Particulars Sec. Refer Amounts


Ref. W.N.
Amounts to be included u/s 7 of the Act:
Sales 7.2 12,300,000
Dividend Received net of Tax 7.3 1 -
Scrap Sales 7.2 140,000
Consulting Income 7.2 450,000
Total Inclusions in Income (A) 12,890,000
Allowable Deductions
Interest Expenses 14 1,734,000
(As per Solution [C] above)
Cost of Trading Stock 15 4,510,000
(Raw Materials Consumed+ Decrease in Finished Goods Stock)
Repair & Improvement Expenses 16 3 230,000
Pollution Control Cost 17 499,000
[as per solution (a) above]
Research & Development Cost 18 8,04,000
[as per solution (b) above]
Depreciation as per Income Tax Act 19 432,000
Salary & Wages 13 6 1,800,000
Administrative Overhead 13 6 600,000
Total Deductions (B) 9,805,000
Assessable Income from Business (A – B) 3,085,000

Statement of Total Assessable Income & Taxable Income


Section Refer
Particulars Amount
Reference W. N.
Assessable Income from Business 3,085,000
Assessable Income from Investment 8 -
Assessable Income from Windfall Gain 8 -
Total Assessable Income 3,085,000
Less: Allowable Deductions from Total Assessable
Income to arrive at Taxable Income
A. Contribution to PM Relief Fund & National
Reconstruction Fund of GON 12B 9 -
B. Expenditure u/s 12A 12A 9 -
C. Donation u/s 12 (3) 9 -
D. Donation u/s 12 (1) & 12 (2) 10 100,000
Taxable Income 2,985,000

Statement of Tax Liability:


It is a production based industry, means, special industry u/s 11; implying the Corporate Tax rate to
be 20%.
Tax Liability= taxable income * Corporate Tax rate = 2985000*20%= Rs. 597,000

Working Notes:
Chapter 11- Income from Business

1. Dividend is assumed to be received from a resident company. As per Sec. 92, dividend received from
resident company is final withholding payments. As per Sec. 7 (3), final withholding payments do not
form part of income.
2. As per the Interpretation by Income Tax Manual, the total interest expenses eligible as per Sec. 14 (1)
regardless of ceiling mentioned in Sec. 14 (2) should be included while calculating adjusted taxable
income for the purpose of Sec. 17 and 18, and ignored for the purpose of Sec. 14 (2).
3. It is assumed that the repair of plants is within 7% of Depreciation Base of Pool D. In absence of
detailed information, gross up of depreciation does not seem feasible.
4. Pollution Control Cost is ignored for the purpose of calculation of ATI for the purpose of Sec. 17, and
considered at actual to calculate ATI for the purpose of Sec. 14 (2) and 18.
5. Research & Development Cost is ignored for the purpose of calculation of ATI for the purpose of Sec.
18, and considered at actual to calculate ATI for the purpose of Sec. 14 (2) and 17.
6. It is believed that all conditions as per Sec. 13 are satisfied by the expenses.
7. Donation cannot be deducted while computing Adjusted Taxable Income as per the definition.
8. In absence of information, it is assumed that there are no other incomes.
9. In absence of information, it is assumed that there is no contribution to PM Relief Fund, or National
Reconstruction Fund of GON or Expenditure u/s 12A or notified donations u/s 12 (3).
10. Eligible Donation Expenditure
Minimum of Following:
a. Actual Donation 310,000
b. 5% of ATI 154,250
(ATI = Total Assessable Income – Contribution u/s 12B – Expenditure u/s 12A)
c. Maximum 100,000
Therefore, eligible = Rs. 100,000
Chapter 12- Income from Employment

Answer to Question No. 1- Page 127

a. Calculation of Tax Liability of Mr. Wooster (based on provisions for 2072/73)


Statement of Assessable Income from Employment
Section Refer
Particulars Amount
Reference W. N.
Basic Salary 8.2 350,000
Uniform Allowance 8.2 1 7,000
Technical Allowance 8.2 1 420,000
Accommodation Facility 8.2/27/ 2 7,000
Rule 13
Telephone facility 8.2/27 3 3,500
Vehicle Facility 8.2/27/ 4 1,750
Rule 13
Compensation paid by current employer to previous 8.2 5 200,000
employer
Advance salary for Shrawan 2072 & Bhadra 2072 6 -
Leave pay 8.2 7 50,000
Value of accumulated leave provision 22 8 -
Tuition fee directly paid to Tuition center 8.2 5 40,000
Airfare of Mr. Wooster from USA to Nepal 8.2 5 100,000
Canteen bill paid by employer 8.2 5 40,000
Driver salary of vehicle provided by employer 9 -
Assessable Income from Employment 1,219,250

Statement of Total Assessable Income, Taxable Income & Balance Taxable Income
Section Refer
Particulars Amount
Reference W. N.
Assessable Income from Business 10 -
Assessable Income from Employment 1,219,250
Assessable Income from Investment 10 -
Assessable Income from Windfall Gain 10 -
Total Assessable Income 1,219,250
Less: Allowable Deductions from Total Assessable 11 -
Income to arrive at Taxable Income
Taxable Income 1,219,750
Less: Allowable Reductions from Taxable Income as 11 -
per Schedule 1 (1) of the Act
Balance Taxable Income 1,219,750

Statement of Tax Liability


Refer
Particulars Rate Amount
W. N.
st
1 Rs. 250,000 1% 2,500
Next Rs. 100,000 15% 15,000
Balance Rs. 869,250 25% 217,312.50
Tax Liability before Tax Credits 234,812.50
Chapter 12- Income from Employment

Less: Tax Credits 12 -


Tax Liability after Tax Credits 234,812.50

Conclusion:
The Withholding Tax as per Sec. 87, withheld by the employer of Rs. 265,000 is not correct.

Working Notes:
1. All types of allowance form part of assessable income from employment.
2. In case the employer provides accommodation facility, whether it is in owned building of employer
or the space occupied by employment under rental agreement, 2% of Khai paai aaeko talab shall
be included in income.
Income Tax Manual, 2066 (Updated 2068) interprets Khai Paai aaeko talab as sum of basic
salary and grade generated by the employee during the Income Year.
Therefore, amounts to be included in income = 2% of basic salary (as there is no information as
to the grade) = 2% of 350,000 = 7,000
(Please note that when employer pays rent, it’s the accommodation facility and is not construed
as rent allowance)
3. When employer provides facilities other than accommodation facility, vehicle facility, concessional
loan or remuneration in kind; the cost borne by the employer shall be included in income.
4. In case the employer provides vehicle facility, 0.5% of Khai paai aaeko talab shall be included in
income.
Income Tax Manual, 2066 (Updated 2068) interprets Khai Paai aaeko talab as sum of basic
salary and grade generated by the employee during the Income Year.
Therefore, amounts to be included in income = 0.5% of basic salary (as there is no information as
to the grade) = 0.5% of 350,000 = 1,750
5. It is reimbursement of personal expenses of the employee, which is included in income.
6. As per Sec. 22 of the Act, natural person must follow cash basis of accounting while calculating
income from employment; i.e. any income for prospective employment received in cash shall also
be included in income.
However, the general practice of employers is to avoid payment of salary in advance, rather they
provides advance against salary, which is loan to be settled against salary to be drawn in future.
In case the income is provided in advance, the amount shall be included in income whilst the
amount is not considered in case when it is loan against salary.
In this case, it is assumed that the amount is not advance income but loan against salary and
thus, not included in income.
Alternatively, in case when it is assumed that the amount is advance income; it must be included
in income as per basis of accounting.
7. Leave pay may be made during the tenure of employment or at the time of retirement. The
payment made during the tenure of employment (as in this case) is included in income, and the
payment at the time of retirement is final withholding and not included in income.
8. Natural person must follow cash basis of accounting, and thus any amount in relation to
employment received in cash is only included. Since, the amount is accrued amount and not
received in cash, hence, it is not included in income.
9. Since vehicle facility is provided, all incidental cost of the vehicle is deemed to be covered by the
amount so quantified in Note (4) above, and thus, the amount is not included in income.
10. In absence of information, it is assumed that there is no other income than the amounts disclosed
in question. Further, it is asked to check the WHT calculation by employer, as such, there is no
need to consider other income, if any.
Chapter 12- Income from Employment

11. In absence of information as to Mr. Wooster’s contribution to ARF, Donation to Exempt


organization, or Contribution to PM Relief Fund or National Reconstruction Fund of GON; and
other information, it is assumed that there is no reductions or deductions to arrive at taxable
income and balance taxable income.
12. In absence of information, it is assumed that there are no any tax credits
13. Assumed Individual Assessment.
b. As per Sec. 97 (1), unless IRD requires in writing or through public circular a resident natural person
satisfying all the conditions laid down in Sec. 4 (3) is relieved from submission of income return
unless the taxable income exceeds Rs. 40 lakhs as per Sec. 97 (2).
The conditions laid down in Sec. 4 (3) are as follows:
a. The person shall derive income only from employment having source in Nepal.
b. The employer shall be resident during the Income Year and the person shall have only one
employer at a time.
c. The person shall only claim the Contribution to Approved Retirement Fund paid by the employer
and Medical Tax Credit on medical expenditure incurred by the employer and the person shall not
claim Donation u/s 12 for deduction.
In this case, since Mr. Wooster is a resident natural person (satisfying 183 days test), having taxable
income not more than Rs. 40 Lakhs and satisfying all the above conditions, he is not required to
submit tax return and tax liability as calculated by employee as per the Act & rules is final.
c. Person’s tax liability is determined by residential status and source of such income, and not by
Citizenship. Moreover, the taxability is determined by the law of land. Since Mr. Wooster is resident of
Nepal (satisfying 183 days test), he is required to pay tax on global income (as per Sec. 6) in Nepal.
Hence, his contention is not correct.
Even if he were nonresident of Nepal, as he is providing service within the territory of Nepal; the
income is having source in Nepal and as per Sec. 6, nonresident is required to pay tax in Nepal
Sourced Income. Hence he would not be exempted from payment of tax.

Answer to Question No. 2- Page 127

Calculation of Tax Liability of Mr. Ram Bahadur (based on provisions for 2072/73)
Statement of Assessable Income from Employment
Section Refer
Particulars Amount
Reference W. N.
Basic Salary 8.2 1,200,000
Metro Allowance 8.2 1 120,000
Accommodation Facility 8.2/27/ 2 24,000
Rule 13
Vehicle Facility 8.2/27/ 3 6,000
Rule 13
Assessable Income from Employment 1,350,000

Statement of Total Assessable Income, Taxable Income & Balance Taxable Income
Section Refer
Particulars Amount
Reference W. N.
Assessable Income from Business 4 -
Assessable Income from Employment 1,350,000
Assessable Income from Investment 4 -
Assessable Income from Windfall Gain 4 -
Chapter 12- Income from Employment

Total Assessable Income 1,350,000


Less: Allowable Deductions from Total Assessable Income 5 -
to arrive at Taxable Income
Taxable Income 1,350,000
Less: Allowable Reductions from Taxable Income as per 5 -
Schedule 1 (1) of the Act
Balance Taxable Income 1,350,000

Statement of Tax Liability


Refer
Particulars Rate Amount
W. N.
st
1 Rs. 250,000 1% 2,500
Next Rs. 100,000 15% 15,000
Balance Rs. 1,000,000 25% 250,000
Tax Liability before Tax Credits 267,500
Less: Tax Credits 6 -
Tax Liability after Tax Credits 267,500

Working Notes:
1. All types of allowance form part of assessable income from employment.
2. In case the employer provides accommodation facility, whether it is in owned building of employer
or the space occupied by employment under rental agreement, 2% of Khai paai aaeko talab shall
be included in income.
Income Tax Manual, 2066 (Updated 2068) interprets Khai Paai aaeko talab as sum of basic
salary and grade generated by the employee during the Income Year.
Therefore, amounts to be included in income = 2% of basic salary (as there is no information as
to the grade) = 2% of 1,200,000 = 24,000
(Please note that when employer pays rent, it’s the accommodation facility and is not construed
as rent allowance)
3. In case the employer provides vehicle facility, 0.5% of Khai paai aaeko talab shall be included in
income.
Income Tax Manual, 2066 (Updated 2068) interprets Khai Paai aaeko talab as sum of basic
salary and grade generated by the employee during the Income Year.
Therefore, amounts to be included in income = 0.5% of basic salary (as there is no information as
to the grade) = 0.5% of 1,200,000 = 6,000
4. In absence of information, it is assumed that there is no other income than the amounts disclosed
in question.
5. In absence of information as to Mr. Ram Bahadur contribution to ARF, Donation to Exempt
organization, or Contribution to PM Relief Fund or National Reconstruction Fund of GON; and
other information, it is assumed that there is no reductions or deductions to arrive at taxable
income and balance taxable income.
6. In absence of information, it is assumed that there are no any tax credits
7. Assumed Individual Assessment since it is not compulsory to elect couple even when he is
married.

Answer to Question No. 3- Page 127

Calculation of Tax Liability of Mr. Sailesh (based on provisions for 2072/73)


Chapter 12- Income from Employment

Statement of Assessable Income from Employment


Section Refer
Particulars Amount
Reference W. N.
Salary & Allowances 8.2 1 600,000
Dashain Allowance 8.2 1 20,000
Bonus 8.2 40,000
Employer’s Contribution to ARF 8.2 2 12,000
Pension Income from GON 8.2 3 150,000
Assessable Income from Employment 822,000

Statement of Total Assessable Income, Taxable Income & Balance Taxable Income
Section Refer
Particulars Amount
Reference W. N.
Assessable Income from Business 4 -
Assessable Income from Employment 822,000
Assessable Income from Investment 4 -
Assessable Income from Windfall Gain 4 -
Total Assessable Income 822,000
Less: Allowable Deductions from Total Assessable Income
to arrive at Taxable Income
A. Contribution to ARF 63, Rule 21 5 24,000
B. Contribution to PM Relief Fund & National 12 6 -
Reconstruction Fund of GON
C. Donation u/s 12 (3) 6 -
D. Donation u/s 12 (1) & 12 (2) 6 -
Taxable Income 798,000
Less: Allowable Reductions from Taxable Income as per
Schedule 1 (1) of the Act
A. Reduction due to residence in remote area 7 30,000
B. Reduction of “Foreign Allowance” -
C. Reduction of Pension Income 8 75,000
D. Reduction to an Incapacitated Person 9 150,000
E. Reduction for payment of Investment Insurance 10
Premium 20,000
F. Reduction for payment of Medical Insurance Premium
to Resident Insurance company -
Balance Taxable Income 523,000

Statement of Tax Liability


Refer
Particulars Rate Amount
W. N.
st
1 Rs. 300,000 1% 3,000
Next Rs. 100,000 15% 15,000
Balance Rs. 123,000 25% 30,750
Tax Liability before Tax Credits 48,750
Less: Tax Credits 11 -
Chapter 12- Income from Employment

Tax Liability after Tax Credits 48,750

Working Notes:

1. All types of allowance form part of assessable income from employment.


2. In absence of further information, it is assumed that the total contribution (from employer and
employee) during the year is Rs. 24,000. As per general practice, employers’ add 10% of basic salary
and deducts the same and deposit it to RF account maintained in the name of employee. As such,
out of total contribution 50% is employer’s contribution and hence, in this case Rs. 12,000 is assumed
to be contribution by employer.
3. Pension income is the amount received in connection to past employment, which is a part of
employment income and is included as per the Cash basis of accounting.
4. In absence of information, it is assumed that there is no other income than the amounts disclosed in
question.
5. Calculation of eligible amount of deduction for Contribution to ARF
As per Sec. 63 & Rule 21, Minimum of following three amounts is deductible for tax purpose:
a. Actual Contribution to ARF 24,000
(Total amount deposited across all ARF during the year in the
RF account of Natural person)
rd
b. 1/3 of Total Assessable Income 274,000
(in this case total assessable income is Rs. 822,000)
c. Rs. 300,000

The minimum of above three is Rs. 24,000; which is deductible

6. In absence of information as to the Donation to Exempt organization, or Contribution to PM Relief


Fund or National Reconstruction Fund of GON; it is assumed that there is no deductions to arrive at
taxable income.
7. He resides in Dailekh throughout the year. As per the classification by GON, Dailekh district falls
under “Category C” remote area where the allowable reduction is Rs. 30,000. (See page 70 of the
book for classification of areas)
8. Calculation of allowable reduction as a result of pension Income
The allowable reduction in case of pension income for a resident natural person is lower of following
two:
a. Actual pension Income 150,000
b. 25% of Basic Exemption Limit 75,000
(As he opted for couple assessment, the basic exemption limit is 300,000)
Allowable amount is Rs. 75,000
9. An incapacitated person is allowed a reduction of 50% of Basic Exemption Limit from taxable Income
before calculating tax liability. Since he opts for couple assessment, the eligible amount is 150,000
(50% of Rs. 300,000)
10. Calculation of allowable reduction against payment of Investment Insurance Premium
The allowable reduction in case of against payment of Investment Insurance Premium for a resident
natural person is lower of following two:
a. Actual payment 25,000
b. Maximum 20,000
Allowable amount is Rs. 20,000
11. In absence of information, it is assumed that there are no any tax credits
Chapter 12- Income from Employment

Answer to Question No. 4- Page 127

Calculation of Tax Liability of Mr. Ramesh Karki (based on provisions for 2072/73)
Statement of Assessable Income from Employment
Section Refer
Particulars Amount
Reference W. N.
Salary for the year 8.2 600,000
Allowances 8.2 1 120,000
Employer’s Contribution to ARF 8.2 2 60,000
Accommodation facility 8.2 3 12,000
Assessable Income from Employment 792,000

Statement of Total Assessable Income, Taxable Income & Balance Taxable Income
Section Refer
Particulars Amount
Reference W. N.
Assessable Income from Business 4 -
Assessable Income from Employment 822,000
Assessable Income from Investment 4 -
Assessable Income from Windfall Gain 4 -
Total Assessable Income 822,000
Less: Allowable Deductions from Total Assessable Income
to arrive at Taxable Income
A. Contribution to ARF 63, Rule 21 5 120,000
B. Contribution to PM Relief Fund & National 12 6 -
Reconstruction Fund of GON
C. Donation u/s 12 (3) 6 -
D. Donation u/s 12 (1) & 12 (2) 6 -
Taxable Income 672,000
Less: Allowable Reductions from Taxable Income as per
Schedule 1 (1) of the Act
A. Reduction due to residence in remote area 7 10,000
B. Reduction of “Foreign Allowance” -
C. Reduction of Pension Income -
D. Reduction to an Incapacitated Person -
E. Reduction for payment of Investment Insurance
Premium -
F. Reduction for payment of Medical Insurance Premium
to Resident Insurance company -
Balance Taxable Income 662,000

Statement of Tax Liability


Refer
Particulars Rate Amount
W. N.
st
1 Rs. 300,000 1% 3,000
Next Rs. 100,000 15% 15,000
Balance Rs. 262,000 25% 65,5000
Chapter 12- Income from Employment

Tax Liability before Tax Credits 83,500


Less: Tax Credits 8 -
Tax Liability after Tax Credits 83,500

Working Notes:

1. All types of allowance form part of assessable income from employment.


Out of allowance of Rs. 170,000; Rs. 50,000 is paid for official purpose. As per Sec. 8 (3) any
reimbursements meeting the employer’s business objective is not required to be included in income.
As such, Rs. 50,000 is not included in income.
2. In absence of further information, it is assumed that the total contribution (from employer and
employee) during the year is Rs. 24,000. As per general practice, employers’ add 10% of basic salary
and deducts the same and deposit it to RF account maintained in the name of employee. As such,
out of total contribution 50% is employer’s contribution and hence, in this case Rs. 12,000 is assumed
to be contribution by employer.
3. In case the employer provides accommodation facility, whether it is in owned building of employer or
the space occupied by employment under rental agreement, 2% of Khai paai aaeko talab shall be
included in income.
Income Tax Manual, 2066 (Updated 2068) interprets Khai Paai aaeko talab as sum of basic salary
and grade generated by the employee during the Income Year.
Therefore, amounts to be included in income = 2% of basic salary (as there is no information as to the
grade) = 2% of 600,000 = 12,000
(Please note that when employer pays rent, it’s the accommodation facility and is not construed as
rent allowance)
4. In absence of information, it is assumed that there is no other income than the amounts disclosed in
question.
5. Calculation of eligible amount of deduction for Contribution to ARF
As per Sec. 63 & Rule 21, Minimum of following three amounts is deductible for tax purpose:
a. Actual Contribution to ARF 120,000
(Total amount deposited across all ARF during the year in the
RF account of Natural person, i.e. employers’ contribution to
EPF Plus Ramesh’s contribution to CIT)
b. 1/3rd of Total Assessable Income 264,000
(in this case total assessable income is Rs. 792,000)
c. Rs. 300,000

The minimum of above three is Rs. 120,000; which is deductible

6. In absence of information as to the Donation to Exempt organization, or Contribution to PM Relief


Fund or National Reconstruction Fund of GON; it is assumed that there is no deductions to arrive at
taxable income.
7. He resides in Dhunche of Rasuwa district throughout the year. As per the classification by GON,
Dhunche of Rasuwa district falls under “Category E” remote area where the allowable reduction is Rs.
10,000. (See page 70 of the book for classification of areas)
8. In absence of information, it is assumed that there are no any tax credits
9. Assumed Couple Assessment
Chapter 12- Income from Employment

Answer to Question No. 5- Page 128

Accommodation facility

In case the employer provides accommodation facility which is evidenced by:

a. Agreement between house owner and the employer or the owned building of employer, and
b. The withholding tax of the rental payments are withheld as per Sec. 88 by the employer, in case
of leased building

The amount to be included in the employee’s assessable income is guided by Sec. 27 of the Act; which
requires inclusion of accommodation facility in income and the quantification is directed by Rule 13 of
Income Tax Regulation, which is 2% of Khai Paai Aaeko talab.

There are debates in the interpretation on what constitutes Khai paai aaeko talab, however, the Income
Tax Manual issued by IRD has made it simple to describe Khai paai aaeko talab, which explains the sum
of basic salary and grade derived by an individual during the Income Year as Khai Paai aaeko talab.

In contrast, when the employer provides reimbursement of rental expenses of the employee; it is not
benefit attracting Sec. 27 of the Act, but the reimbursement of expenses of the employees which is to be
included in income as per Sec. 8 (2) of the Act. The key difference between these two are:

a. In facility, it is the employer responsible to deal with house owner, and to make the payment of
house rent where the employee is unconcerned about the rental expenses WHEREAS in
reimbursement, the dealing with the owner is done by the employee and the employer has no
concern about when and where the employee resides.

As such, in the given case:

a. 2% of Khai paai aaeko talab, i.e. 2% of 60,000 * 12 shall be included in Mr. Rabin Sharma’s
Income as per Sec. 27 and Rule 13
b. Rs. 25,000 per month shall be included in Mr. Binod Shrestha’s income as per Sec. 8 (2) as
“Payments providing any discharge or reimbursement of costs incurred by the individual
or an associate of the individual”

Answer to Question No. 6- Page 128

Let the Gross pay from employer be “X”

In which case the balance taxable income become “X-20000” (Perform the test to calculate BTI from Total
Assessable Income)

Since the net income is Rs. 5 Million, the taxable income of the individual exceed Rs. 25 Lakhs; and as
per question:

Gross Pay- Tax Liability = 5,000,000

Calculation of Tax Liability as we have gross pay of Rs. X


st
1 Rs. 300,000 (Couple) 1% 3,000
Next Rs. 100,000 15% 15,000
Next Rs. 2,100,000 25% 525,000
Balance Rs. (X-20,000-2500000) 35% 0.35X- 882,000
Chapter 12- Income from Employment

Total Tax Liability 0.35X-339000


Equating, Gross Pay – Total Tax Liability = 5,000,000
X- (0.35X-339000)= 5000000
0.65X = 4661000
X= 7,170,770
Tax Liability= 2,170,770

Answer to Question No. 7- Page 128

Refer BOX in Page 120 of the book

Answer to Question No. 8- Page 128

In case of employment Income, the tax liability is 1% of Rs. 150,000.

The tax liability would be Zero if the income was from business or investment as the income falls within
the Basic Exemption Limit.

Answer to Question No. 9- Page 128

Sec. 8 (2) of the Act requires any benefit provided by the employer to employee to be included in the
income. Sec. 27 of the Act describes what benefits are to be included and method of quantification of
such benefits. In case of vehicle and accommodation facility, the quantification is prescribed in Rule 13 of
Income Tax Regulation; which requires inclusion of 0.5% of KHAI PAAI AAEKO TALAB in the employee’s
income when the facility is provided to employee (salaried individual) and 1% p.a. of Market Value of
Vehicle when the facility is provided to persons other than employee (self-employed person, consultant).
(See the difference of salaried individual and self employer individual in page 118 and 119 of the book)

Though the rule provides guidelines to quantify the amount, the Act and the rule has not defined what
constitutes KHAI PAAI AAEKO TALAB, and there is no consensus in academic discussions regarding
the same. But Income Tax Manual issued by IRD has made it simpler by interpreting Khai paai aaeko
talab as sum of basic salary and grade derived by the individual during the Income Year.

As explained above:

a. The amounts to be included in income of Mr. A (as employee of XYZ Co. Ltd.) is 0.5% of basic
salary and grade derived by him during the income year, which is 0.5% of (12 *
(150000+75000))= Rs. 13500
b. In case he is consultant and not the employee, the amount to be quantified as 1% of Market
Value of Vehicle which is 1% p.a. of Rs. 5,700,000. As the facility is to be provided for four
months, the amount is 4/12*57000= 19,000

Answer to Question No. 10- Page 128

Statement of Assessable Income from Employment of Mr. Yubaraj Thapa for 20X1/X2
In absence of other information as to Mr. Thapa’s income other than retirement payments; it is assumed
that retirement payments are the only income of Mr. Thapa for IY 20X1/X2.
As the payment is made from Approved Retirement fund and assuming all amounts accrued after the
implementation of the Act, the tax withheld by ARF at the time of payment u/s 88 of the Act is final as per
Sec. 95; which is Rs. Zero, as less than Rs. 500,000.
Chapter 12- Income from Employment

As per Sec. 97 (1) (b), person deriving income only from final withholding payments are not required to
submit income return for the year unless otherwise required by IRD in writing or through public circular.
Hence, as there is no specific information as IRD’s demand of income return in the question and Mr.
Thapa has no any other income, he is not required to submit income return in that regard.

Calculation of Tax Liability of Mr. Thapa for 20X2/X3


Statement of Assessable Income from Employment
Section Refer
Particulars Amount
Reference W. N.
Pension Income 8.2 1 252,000
Dashain Allowance 8.2 2 21,000
Assessable Income from Employment 263,000

Statement of Assessable Income from Business


Section Refer
Particulars Amount
Reference W. N.
Consultancy Income 7.2 3 25,000
Assessable Income from Business 25,000

Statement of Assessable Income from Investment


Section Refer
Particulars Amount
Reference W. N.
Interest Income 92/8.3 4 -
Dividend Income 92/8.3 4 -
Assessable Income from Investment -

Statement of Total Assessable Income, Taxable Income & Balance Taxable Income
Section Refer
Particulars Amount
Reference W. N.
Assessable Income from Business 25,000
Assessable Income from Employment 263,000
Assessable Income from Investment -
Assessable Income from Windfall Gain 5 -
Total Assessable Income 288,000
Less: Allowable Deductions from Total Assessable Income
to arrive at Taxable Income
A. Contribution to ARF 63, Rule 21 6 -
B. Contribution to PM Relief Fund & National 12 6 -
Reconstruction Fund of GON
C. Donation u/s 12 (3) 6 -
D. Donation u/s 12 (1) & 12 (2) 6 -
Taxable Income 288,000
Less: Allowable Reductions from Taxable Income as per
Schedule 1 (1) of the Act
A. Reduction due to residence in remote area 7 -
Chapter 12- Income from Employment

B. Reduction of “Foreign Allowance” -


C. Reduction of Pension Income 7 -
D. Reduction to an Incapacitated Person 7 -
E. Reduction for payment of Investment Insurance
Premium 7 -
F. Reduction for payment of Medical Insurance Premium
to Resident Insurance company 7 -
Balance Taxable Income 288,000

Statement of Tax Liability


Refer
Particulars Rate Amount
W. N.
st
1 Rs. 250,000 0% 8 2,500
Next Rs. 38,000 15% 5,700
Tax Liability before Tax Credits 8,200
Less: Tax Credits 9 -
Tax Liability after Tax Credits 8,200

Working Notes:

1. Pension income is the amount received in connection to past employment, which is a part of
employment income and is included as per the Cash basis of accounting.
2. All types of allowance form part of assessable income from employment.
3. Assuming net profit is calculated as per Income Tax Law, the income from business is reported.
4. Dividend from resident company and interest received by natural person not related to business and
paid by resident banks in deposit is final withholding and thus, not included in income.
5. In absence of information, it is assumed that there is no other income than the amounts disclosed in
question.
6. In absence of information as to the Donation to Exempt organization, or Contribution to PM Relief
Fund or National Reconstruction Fund of GON or Contribution to ARF; it is assumed that there are no
deductions to arrive at taxable income.
7. In absence of further information, it is assumed that there are no reductions as per Schedule 1 of the
Act.
8. Since there is no income from employment except pension income, there shall be no 1% SST
applicable on it.
9. In absence of information, it is assumed that there are no any tax credits

Requirement for Submission of Income Return:

The return shall be filed u/s 96, none of the conditions in Sec. 97 (1) is satisfied in this case.

Calculation of Tax Liability of Mrs. Thapa for 20X2/X3


Statement of Assessable Income from Employment
Section Refer
Particulars Amount
Reference W. N.
Salary 8.2 120,000
Assessable Income from Employment 120,000
Chapter 12- Income from Employment

Statement of Assessable Income from Business


Section Refer
Particulars Amount
Reference W. N.
Business Income net of deductions 7.2 1 150,000
Service Fee 7.2 2 105,000
Assessable Income from Business 255,000

Statement of Assessable Income from Investment


Section Refer
Particulars Amount
Reference W. N.
Rental Income 92/8.3 3 -
Assessable Income from Investment -

Statement of Total Assessable Income, Taxable Income & Balance Taxable Income
Section Refer
Particulars Amount
Reference W. N.
Assessable Income from Business 120,000
Assessable Income from Employment 255,000
Assessable Income from Investment -
Assessable Income from Windfall Gain 4 -
Total Assessable Income 375,000
Less: Allowable Deductions from Total Assessable Income
to arrive at Taxable Income
A. Contribution to ARF 63, Rule 21 5 -
B. Contribution to PM Relief Fund & National 12 5 -
Reconstruction Fund of GON
C. Donation u/s 12 (3) 5 -
D. Donation u/s 12 (1) & 12 (2) 5 -
Taxable Income 375,000
Less: Allowable Reductions from Taxable Income as per
Schedule 1 (1) of the Act
A. Reduction due to residence in remote area 6 -
B. Reduction of “Foreign Allowance” -
C. Reduction of Pension Income 6 -
D. Reduction to an Incapacitated Person 6 -
E. Reduction for payment of Investment Insurance
Premium 6 -
F. Reduction for payment of Medical Insurance Premium
to Resident Insurance company 6 -
Balance Taxable Income 375,000

Statement of Tax Liability


Refer
Particulars Rate Amount
W. N.
st
1 Rs. 120,000 1% 1,200
Chapter 12- Income from Employment

Rs. 130,000 0% -
Next Rs. 100,000 15% 15,000
Balance Rs. 25,000 25% 6,250
Tax Liability before Tax Credits 22,450
Less: Tax Credits 7 -
Tax Liability after Tax Credits 22,450

Working Notes:

1. Assuming net profit is calculated as per Income Tax Law, the income from business is reported.
2. Setting question paper and checking answer sheets is service fee. Sec. 88 (4) of the Act exempts
withholding of tax on such amounts, which cannot be construed as exemption from the requirement of
tax payment. Assuming the questions are not set and answers are not checked for school, the
amount is to be included as per Sec. 7 (2) (a).
Had we assumed that the payment for setting question paper and checking answer sheets are made
by School employing her, the amount would have been included while calculating assessable income
from employment.
3. It is assumed that she is not involved in operation of rental business, as such, amount received as
rent for her house property not connected to business is final withholding and not included in income.
4. In absence of information, it is assumed that there is no other income than the amounts disclosed in
question.
5. In absence of information as to the Donation to Exempt organization, or Contribution to PM Relief
Fund or National Reconstruction Fund of GON or Contribution to ARF; it is assumed that there are no
deductions to arrive at taxable income.
6. In absence of further information, it is assumed that there are no reductions as per Schedule 1 of the
Act.
7. In absence of information, it is assumed that there are no any tax credits

Requirement for Submission of Income Return: The return shall be filed u/s 96, none of the conditions in
Sec. 97 (1) is satisfied in this case.

Answer to Question No. 11- Page 128

Statement of Assessable Income from Employment


Section Refer
Particulars Amount
Reference W. N.
Monthly Remuneration 8.2 480,000
Dashain Allowance 8.2 1 40,000
Education Allowance 8.2 1 24,000
Expenditure Allowance 8.2 1 24,000
Bonus 8.2 60,000
Remuneration from Shankar Dev Campus 8.2 72,000
Vehicle Facility 8.2/27/ 2 2,760
Rule 13
Driver’s Remuneration 3 -
Petrol Expenditure for Vehicle 3 -
Repair expenses of vehicle 3 -
Accommodation Facility 8.2/27/ 4 11,040
Chapter 12- Income from Employment

Rule 13
Telephone Facility 8.2/27/8.3 5&6 12,000
Security Guard Facility 8.2/27 6 36,000
Loan at Concessional Interest Rate 8.2/27 7 30,000
Leave Pay 8.2 8 46,000
Contribution of Employer in Retirement Fund Account of 8.2 9 55,200
Employee
Assessable Income from Employment 893,000

Statement of Total Assessable Income, Taxable Income & Balance Taxable Income
Section Refer
Particulars Amount
Reference W. N.
Assessable Income from Business 10 -
Assessable Income from Employment 893,000
Assessable Income from Investment 10 -
Assessable Income from Windfall Gain 10 -
Total Assessable Income 893,000
Less: Allowable Deductions from Total Assessable Income
to arrive at Taxable Income
A. Contribution to ARF 63, Rule 21 11 297,667
B. Contribution to PM Relief Fund & National 12 12 -
Reconstruction Fund of GON
C. Donation u/s 12 (3) 12 -
D. Donation u/s 12 (1) & 12 (2) 13 29,767
Taxable Income 565,566
Less: Allowable Reductions from Taxable Income as per
Schedule 1 (1) of the Act
A. Reduction due to residence in remote area -
B. Reduction of “Foreign Allowance” -
C. Reduction of Pension Income -
D. Reduction to an Incapacitated Person -
E. Reduction for payment of Investment Insurance
Premium 14 20,000
F. Reduction for payment of Medical Insurance Premium
to Resident Insurance company -
Balance Taxable Income 545,566

Statement of Tax Liability


Refer
Particulars Rate Amount
W. N.
st
1 Rs. 300,000 1% 3,000
Next Rs. 100,000 15% 15,000
Balance Rs. 145,566 25% 36,392
Tax Liability before Tax Credits 54,392
Less: Tax Credits
A. Female Tax Credit (not eligible) -
Chapter 12- Income from Employment

B. Medical Tax Credit 51 15 750


C. Foreign Tax Credit (no Information) -
Tax Liability after Tax Credits 53,642

Working Notes:

1. All types of allowance form part of assessable income from employment.


2. In case the employer provides vehicle facility, 0.5% of Khai paai aaeko talab shall be included in
income.
Income Tax Manual, 2066 (Updated 2068) interprets Khai Paai aaeko talab as sum of basic salary
and grade generated by the employee during the Income Year.
Therefore, amounts to be included in income = 0.5% of basic salary (as there is no information as to
the grade) = 0.5% of 552,000 = 2,760
Nepalese income tax law is based on self assessment, and as the manual suggests inclusion of basic
salary and grade in determining Khai paai aaeko talab, all basic salaries drawn from all employers
during the year shall be considered to determine Khai paai aaeko talab; and as such the amount is
composed of Remuneration from Bank and Shankar Dev Campus.
Provision in Income Tax Manual:
Income Tax Manual, 2066 (Updated 2068) through its Example 17.14.4 (Pg. 344/336 in PDF
Version of the Manual) demonstrates that the quantification of the vehicle or accommodation facility
is based on the Basic Salary and Grade received from the employer providing such facility during
the income year. Which means:
Khai Paai aaeko talab= Rs. 480,000 (i.e. the basic salary- as there is no information as to grade-
derived from the bank, and excludes the salary derived from Shankar Dev Campus)
The answer may be different in this regard. Students are advised to follow Income Tax Manual
quoting the same in examination.

3. Since the vehicle is provided, all associated cost of vehicles are included as part of quantification as
per Sec. 27 and Rule 13 and hence ignored for tax calculation.
4. In case the employer provides accommodation facility, whether it is in owned building of employer or
the space occupied by employment under rental agreement, 2% of Khai paai aaeko talab shall be
included in income.
Income Tax Manual, 2066 (Updated 2068) interprets Khai Paai aaeko talab as sum of basic salary
and grade generated by the employee during the Income Year.
Therefore, amounts to be included in income = 2% of basic salary (as there is no information as to the
grade) = 2% of 552,000 = 11,040
(Please note that when employer pays rent, it’s the accommodation facility and is not construed as
rent allowance)
Nepalese income tax law is based on self assessment, and as the manual suggests inclusion of basic
salary and grade in determining Khai paai aaeko talab, all basic salaries drawn from all employers
during the year shall be considered to determine Khai paai aaeko talab; and as such the amount is
composed of Remuneration from Bank and Shankar Dev Campus.
5. Since out of total telephone expenses, 50% is for official purpose; and the reimbursement or
discharge serving proper business purpose of employer is not required to be included in income as
per Sec. 8 (3). As such, only 50% of discharge of telephone expenses is included in income.
6. In case of other facilities, the cost to be borne by employer shall be included in income.
7. Loan at Concessional Interest Rate:
Chapter 12- Income from Employment

When the employer provides loan at concessional interest rate, the amount of interest calculated by
using the rate derived from difference in market interest rate and charged interest rate on loan utilized
shall be included in income.
The Income Tax Act & Regulation has not defined market interest rate, and as such, the rate 0f 8%
charged by bank in similar loans to other customer is taken as market interest rate.
Amounts to be included = 3% (8%-5%) p.a. of Rs. 10 Lakhs = 30,000
8. In absence of other information, it is assumed that both the bank and Shankar Dev Campus has
made payment of leave.
Since the basis of accounting is cash and the amount is obtained during the tenure of employment;
the amount shall be included in income.
9. In absence of further information, it is assumed that the both the bank and Shankar Dev Campus
makes contribution to ARF, as such the amount to be included is 10% of 552,000, i.e. 55,200.
10. In absence of information, it is assumed that there is no other income than the amounts disclosed in
question.
11. Calculation of eligible amount of deduction for Contribution to ARF
As per Sec. 63 & Rule 21, Minimum of following three amounts is deductible for tax purpose:
a. Actual Contribution to ARF 350,400
(Total amount deposited across all ARF during the year in the
RF account of Natural person; i.e. 55200*2+ 240000)
rd
b. 1/3 of Total Assessable Income 297,667
(in this case total assessable income is Rs. 893,000)
c. Rs. 300,000

The minimum of above three is Rs. 297,667; which is deductible

12. In absence of information as to the Contribution to PM Relief Fund or National Reconstruction Fund
of GON; it is assumed that there are no deductions to arrive at taxable income.
13. Donation to Exempt Organization:
As per Sec. 12 (1) and 12 (2), Minimum of following three amounts is deductible for tax purpose:
a. Actual Donation or Gift to Exempt Organization 6,000
b. 5% of Adjusted Taxable Income 29,767
(ATI= 893,000-297,667): 5% of 595,333
c. Max. Rs. 100,000 100,000

The minimum of above three is Rs. 29,767; which is deductible

14. Since it is couple assessment, insurance premium paid for both the spouses is eligible to be
considered for actual payment.
The minimum of the following two is deductible:
a. Actual 33,000
b. Max. 20,000

The minimum of above three is Rs. 20,000; which is deductible

15. Eligible Amount of Medical tax Credit:


Minimum of the following three amounts can be claimed as tax credit:
a. 15% of Approved Medical Expenditure 3,000
b. Max. 750

Requirement of Submission of Tax return:


Chapter 12- Income from Employment

As per Sec. 97 (1), the following persons are exempted from submission of tax return:

a. a person who has no income tax payable for the year under section 3 (a)
b. a person who derives income exclusively from Final Withholding Payments (discussed in Chapter
8) during the Income Year,
c. A Resident Natural Person satisfying all the following conditions:
• The person’s income for the year consists exclusively of income from any employment
having a source in Nepal
• The person has only one employment at a time during the year, even if the employment
changes during the year, and each employment is by a resident employer; and
• The person does not claim the following for the Income Year:
i. a medical tax credit under section 51, other than with respect to medical tax credit paid
through the employer
ii. a reduction in taxable income under section 63 (Contribution to Approved Retirement
Fund), other than with respect to retirement contributions paid through the employer
iii. a deduction from Assessable under Sec. 12 (Donation)
d. In case a person who pays tax on income generated from operation of vehicles as per Sec. 1 (13)
of Schedule 1 happens to be a natural person (The matter of vehicle tax is discussed in Chapter
in 4)

Since the person does not satisfy any of the above conditions, he is not exempted from submission of
Income Tax Return.

Answer to Question No. 12- Page 129

Statement of Assessable Income from Employment


Section Refer
Particulars Amount
Reference W. N.
Salary 8.2 3,600,000
House Rent Allowance 8.2 1 240,000
Other Allowance 8.2 1 120,000
Contribution of Employer in Retirement Fund Account of 8.2 360,000
Employee
Pension from GON 8.2 2 144,000
Vehicle Facility 8.2/27/ 3 18,000
Rule 13
Assessable Income from Employment 4,482,000

Statement of Total Assessable Income, Taxable Income & Balance Taxable Income
Section Refer
Particulars Amount
Reference W. N.
Assessable Income from Business 4 -
Assessable Income from Employment 4,482,000
Assessable Income from Investment 4 -
Assessable Income from Windfall Gain 4 -
Total Assessable Income 4,482,000
Less: Allowable Deductions from Total Assessable Income
to arrive at Taxable Income
Chapter 12- Income from Employment

A. Contribution to ARF 63, Rule 21 5 300,000


B. Contribution to PM Relief Fund & National 12 6 -
Reconstruction Fund of GON
C. Donation u/s 12 (3) 6 -
D. Donation u/s 12 (1) & 12 (2) 6 -
Taxable Income 4,182,000
Less: Allowable Reductions from Taxable Income as per
Schedule 1 (1) of the Act
A. Reduction due to residence in remote area 8 -
B. Reduction of “Foreign Allowance” 8 -
C. Reduction of Pension Income 7 75,000
D. Reduction to an Incapacitated Person 8 -
E. Reduction for payment of Investment Insurance
Premium 8 -
F. Reduction for payment of Medical Insurance Premium
to Resident Insurance company 8 -
Balance Taxable Income 4,107,000

Statement of Tax Liability


Refer
Particulars Rate Amount
W. N.
st
1 Rs. 300,000 1% 3,000
Next Rs. 100,000 15% 15,000
Next Rs. 2,100,000 25% 525,000
Balance Rs. 1,607,000 35% 401,750
Tax Liability 944,750

Working Notes:

1. All types of allowance form part of assessable income from employment.


2. Pension income is the amount received in connection to past employment, which is a part of
employment income and is included as per the Cash basis of accounting.
3. In case the employer provides vehicle facility, 0.5% of Khai paai aaeko talab shall be included in
income.
Income Tax Manual, 2066 (Updated 2068) interprets Khai Paai aaeko talab as sum of basic salary
and grade generated by the employee during the Income Year.
Therefore, amounts to be included in income = 0.5% of basic salary (as there is no information as to
the grade) = 0.5% of 3,600,000 = 18,000
4. In absence of information, it is assumed that there is no other income than the amounts disclosed in
question.
5. Calculation of eligible amount of deduction for Contribution to ARF
As per Sec. 63 & Rule 21, Minimum of following three amounts is deductible for tax purpose:
d. Actual Contribution to ARF 720,000
(Total amount deposited across all ARF during the year in the
RF account of Natural person; i.e. 360,000*2)
rd
e. 1/3 of Total Assessable Income 1,494,000
(in this case total assessable income is Rs. 4,482,000)
Chapter 12- Income from Employment

f. Rs. 300,000

The minimum of above three is Rs. 300,000; which is deductible

6. In absence of information as to the Contribution to PM Relief Fund or National Reconstruction Fund


of GON or donation u/s 12 of the Act; it is assumed that there are no deductions to arrive at taxable
income.
7. Eligible reductions due to Pension Income
The MINIMUM of following TWO is eligible for reductions from taxable income:
a. 25% of Basic Exemption Limit 75,000
(Basic Exemption Limit is 300,000 as its couple assessment)
b. Actual Pension Income 144,000
8. In absence of information, it is assumed that there is no any other reduction from taxable income as
per Schedule 1 of the Act.

Answer to Question No. 13- Page 129

Statement of Assessable Income from Employment


Section Refer
Particulars Amount
Reference W. N.
Salary 8.2 480,000
Traveling Allowance 8.2 1 48,000
Contribution of Employer in Retirement Fund Account of 8.2 2 48,000
Employee
Dividend Income from Surya Moon Corporation 8.3 3 -
Prize on National 1KM Race Competition 8.3/88A 4 -
Assessable Income from Employment 576,000

Statement of Total Assessable Income, Taxable Income & Balance Taxable Income
Section Refer
Particulars Amount
Reference W. N.
Assessable Income from Business 5 -
Assessable Income from Employment 576,000
Assessable Income from Investment 5 -
Assessable Income from Windfall Gain 5 -
Total Assessable Income 576,000
Less: Allowable Deductions from Total Assessable Income
to arrive at Taxable Income
A. Contribution to ARF 63, Rule 21 6 96,000
B. Contribution to PM Relief Fund & National 12 7 -
Reconstruction Fund of GON
C. Donation u/s 12 (3) 7 -
D. Donation u/s 12 (1) & 12 (2) 7 -
Taxable Income 480,000
Less: Allowable Reductions from Taxable Income as per
Schedule 1 (1) of the Act
A. Reduction due to residence in remote area 9 -
B. Reduction of “Foreign Allowance” 9 -
Chapter 12- Income from Employment

C. Reduction of Pension Income 9 -


D. Reduction to an Incapacitated Person 9 -
E. Reduction for payment of Investment Insurance
Premium 8 15,000
F. Reduction for payment of Medical Insurance Premium
to Resident Insurance company 9 -
Balance Taxable Income 465,000

Statement of Tax Liability


Refer
Particulars Rate Amount
W. N.
st
1 Rs. 300,000 1% 3,000
Next Rs. 100,000 15% 15,000
Balance Rs. 65,000 25% 16,250
Tax Liability 34,250

Working Notes:

1. All types of allowance form part of assessable income from employment.


1.1. Traveling Allowance related to official travel is exempted from inclusion u/s 8 (3) of the Act, and not
the traveling allowance related to his personal travel. Thus, the allowance for office to home and back
is included in income.
2. Since the question contains additional information as to Mr. Sinha’s additional contribution to ARF,
the amount given in the question is deemed to be the Employer’s contribution.
3. It is assumed that Surya Moon Corporation is resident company, which implies the dividend received
to be final withholding and thus, not included in income.
4. Open competitions are open to all and not related to employment. When a person competes in open
competition, the amount received from victory of such competition cannot be linked to employment,
business or investment income and is related to windfall gain. As such, the organizers are required to
withhold tax at 25% on prize money as per Sec. 88A and as per Sec. 92, such amounts are final
withholding.
5. In absence of information, it is assumed that there is no other income than the amounts disclosed in
question.
6. Calculation of eligible amount of deduction for Contribution to ARF
As per Sec. 63 & Rule 21, Minimum of following three amounts is deductible for tax purpose:
a. Actual Contribution to ARF 96,000
(Total amount deposited across all ARF during the year in the
RF account of Natural person; i.e. 48,000*2)
b. 1/3rd of Total Assessable Income 192,000
(in this case total assessable income is Rs. 576,000)
c. Rs. 300,000

The minimum of above three is Rs. 96,000; which is deductible

7. In absence of information as to the Contribution to PM Relief Fund or National Reconstruction Fund


of GON or donation u/s 12 of the Act; it is assumed that there are no deductions to arrive at taxable
income.
8. Eligible reductions in relation to payment of investment insurance premium
Chapter 12- Income from Employment

The MINIMUM of following TWO is eligible for reductions from taxable income:
a. Actual Payment 15,000
b. Maximum 20,000
9. In absence of information, it is assumed that there is no any other reduction from taxable income as
per Schedule 1 of the Act.

Answer to Question No. 14- Page 129

Refer Page 122 to 125 of the book


Chapter 13- Income from Investment

Answer to Question No. 1- Page 133

Particulars: Inclusion Reason to include or not include


Amount to be included in Income (Sec. 9)
House rent income net of TDS - It is final withholding assuming that Mr. Narayan does
not conduct business of leasing or subleasing of house
Bank interest income net of TDS- Nepal - This is final withholding- bank interest received by
Bank Ltd. natural person and not related to operation of business
Natural resources payments net of TDS 200,000 Included as per Sec. 9 (2) and as its not final
withholding. Grossed up as it is net of TDS. This
payment attracts 15% WHT, means 170000 is 85% of
actual payment, Gross payment is Rs. 200,000
Interest income net of TDS from ABC Ltd. 2,500,000 This interest is not final withholding as it is not a
resident bank or FI, and the interest is assumed not for
debenture or bonds. If we assume the interest is
against debenture and bonds, and ABC Ltd. is
authorized to issue such securities, it becomes final
withholding. WHT on such interest is 15% and thus
2125000 is grossed up to 100%
Compensation received from loss of last 25,000 Included as per Sec. 9 & sec. 31 as the amount is
year investment included when the compensation is actually received
Income from investment insurance net of - Final withholding as Rastriya Beema Sansthan is
TDS- Rastriya Beema Sanstha resident insurance company
Gift related to investment income 50,000 Included as per Sec. 9 (2)- gift received in connection
to investment
Dividend income net of TDS from Sanima - Final withholding. Dividend received from resident
Bank Ltd. company is final withholding
A. Total Amount to be included in 2,775,000
Income
Deductions to arrive at Assessable Income
Expenses related to collection of house rent - Not allowed as per Sec. 21, expense related to
collection of Final withholding payments
Expenses related to natural resources 8,500 Sec. 13- general expenses
Allowable depreciation allowance as per the 5,500 Sec. 19- as it is said its allowable
Act
Life insurance premium paid to Rastriya - As related to Final withholding and is considered by
Beema Sanstha Beema Sansthan while computing gain on Investment
Insurance
B. Total deductible Expenses 14,000
Assessable Income from Investment (A- 2,761,000
B)
Less: Allowable Deductions
a. Contribution to ARF -
b. Donation paid to tax exempt 40,000 As per Sec. 12, lower of:
organization a. Actual
b. 5% of ATI- 5% of 2,761,000
c. Rs. 100,000
Taxable Income 2,721,000 As the question says there is no any assessable
income from business or employment
Tax Liability 617,350 Married, and the maximum tax rate for investment is
st
1 Rs. 300,000 Nil - 10%.
Next Rs. 100,000@ 15% 15,000
Next Rs. 2,100,000 @ 25% 525,000 There is no additional surcharge of 40% on tax above
Balance Rs. 221,000 @35% 77,500 taxable income of Rs. 25 Lakhs as the maximum tax
rate for investment is 10% as explained by Sec. 1 (4)
Chapter 13- Income from Investment

(Kha) of Schedule 1 which comes after Sec. 1 (1) and


Sec. 1 (2) of Schedule 1.
Advance tax paid (WHT considered as There is no net tax liability payable, the amount is
advance tax) refundable
Natural Resource payment 30,000
Interest from ABC Co. 375,000
Chapter 14- Disposal of Assets & Liabilities

Question 1- Page 137

There are four types of assets covered by Income Tax Act, 2058; the gain on disposal of which attracts
income tax. In case any asset does not fall under any of the categories of such assets, the gain on
disposal of such assets are beyond the ambit of Income Tax Act, and income tax is levied on such gain.
The types of assets covered by Income Tax Act are as follows:

1. Business Asset
2. Non Business Chargeable Asset
3. Trading Stock
4. Depreciable Asset

Since Mr. Sundar’s wife possesses the jewelry, which does not fall under any of the categories of asset
above; the gain on disposal of such asset is not taxable in both the case.

As Mr. Sundar has carried on business of jewelry by injecting his wife’s jewelry as capital in his business,
there must be an agreement between his wife and him to consider the jewelry as trading stock, on which
case the cost of trading stock would be market value of asset. In either of the cases listed in the question,
the cost of trading stock is Rs. 30 Million.

The gain on Disposal is:

Sales Proceed: 37,500,000

Less: Cost of Trading Stock (30,000,000)

Gain 7,500,000

Since it is trading stock, the sales are considered part of Sales income u/s 7 (2) (kha) and cost of trading
stock is calculated u/s 15 of the Act. As such, Mr. Sundar is responsible to pay tax on business income.
Chapter 14- Disposal of Assets & Liabilities

Question 1- Page 140

In this question, there are two assets- business asset (land) and depreciable asset (building) and the gain
on disposal for each of the assets is calculated separately.

Gain on Disposal of Land- Business Asset:


Incomings for the Land= 1,500,000
Outgoings for the land= 1,000,000 + 150,000 = 1,150,000
Gain = Incomings – Outgoings = 450,000

Gain on Disposal of Building


Building is depreciable asset falling under Pool A as per Schedule 2 of Income Tax Act. As the question
states, there are other buildings as well in the Pool since many years, and the disposal of building is for
Rs. 2.5 Million, the gain on disposal of new building cannot be calculated on standalone basis, but rather
on Pool basis. As all relevant information related to Pool “A” is given, the gain is calculated as under:

A. Opening Depreciation Base of Pool A for IY 20X3/X4 3,385,000


B. Absorbed additions -
C. Disposal Proceeds 2,500,000
D. Depreciation base (A + B – C) 885,000

Since, the depreciation base as calculated by the formula given in Schedule 2 of the Act is positive with
some asset remaining in the pool, there shall be no gain on disposal.

Working Note:

A. Opening Depreciation Base of Pool A for IY 20X2/X3 1,000,000


B. Absorbed additions (new building-100% as before Poush 20X2) 2,500,000
C. Disposal Proceeds -
D. Depreciation base (A + B – C) 3,500,000
E. Depreciation @ 5% (D * 5%) 175,000
F. Excess Repair & Improvement as given in question 60,000
G. Opening Depreciation Base for 20X3/X4 (D-E+F) 3,385,000
Chapter 14- Disposal of Assets & Liabilities

Question 1- Page 141

As per Sec. 43, in the event of transfer of asset to the spouse of ex-spouse as a result of bonafide
separation or divorce and when any of the spouses elects the provisions of Sec. 43 to be applied in
writing, the transferor of asset is deemed to receive the Outgoings for the assets at the time of disposal,
and the transferee’s Outgoings for the assets so received is deemed to be the amount equal to that
received by the transferee.

In the event of failure to elect Sec. 43 to be applied in writing by either of the spouses, then provisions of
Sec. 45 (1) comes into effect, which means the transferor is deemed to receive an amount equal to higher
of market value of asset or Outgoings to date of such assets. The transferee, in other hand, is deemed to
incur cost equivalent to the amount that is deemed to be received by transferor.

In the given question, since Mr. Ram has transferred his asset as a result of divorce with his wife Ms. Sita
without any financial consideration, if Mr. Ram notifies IRD of the application of provisions of Sec. 43, the
incomings for the disposal of the assets in his hand is deemed to be the amount equal to the Outgoings
incurred by him for the asset, which means

Incomings for Assets = 10,000,000


Outgoings for Assets= 10,000,000
Gain = Incomings – Outgoings = Zero, in the hand of Mr. Ram

While Ms. Sita is deemed to pay the amount equivalent to that received by Mr. Ram, i.e. outgoings at the
time of transfer of asset to Ms. Sita is incomings considered for Mr. Ram and any additional cost for the
asset borne by Ms. Sita, i.e. Rs. 10,000,000 plus Rs. 20,000 of transfer cost.

Incomings for Sita (sales of land on 2069.12.15) = 15,000,000

Gain = Incomings – Outgoings = 15,000,000 – 10,020,000= 4,980,000

In case Mr. Ram fails to notify IRD of his application of Sec. 43, then Sec. 45 (1) comes into effect, which
means,

Incomings for Mr. Ram at the time of Transfer = Higher of Market Value of Asset or Outgoings incurred for
the asset

Outgoings for Ms. Sita at the time of Transfer = Amount considered as Incomings for Mr. Ram + Any cost
incurred by Sita in the process of Transfer

Gain for Mr. Ram (on 2069.10.15) = Incomings derived after calculation as above – 10,000,000

Gain for Ms. Sita (on 2069.12.15) = 15,000,000- Outgoings as calculated above

As the market value at the time of transfer is not given, the problem cannot be solved to derive the
amount. The treatment is well explained above.
Chapter 14- Disposal of Assets & Liabilities

Question 1- Page 142

Transfer in case of death (Sec. 44)

In case of death of a natural person, the asset and liability of the natural person is deemed to be disposed
just before his death (Sec. 40 (3) (a)). Sec. 44 requires the disposal of such asset in market value. The
section is silent about the value of disposal of liability after death.

In the given question, right before the death of Mr. X, the asset and liability owned and owed by Mr. X
st
was deemed to be disposed on 31 Chaitra 20X1. This means, an income return of the income of Mr. X
st st
from the operation of business from 1 Shrawan 20X1 till 31 Chaitra 20X1 and the gain on disposal of
asset calculated as per Sec. 44 of the Act, shall be submitted by the legal heir or any person acting in
authorized or personal capacity to manage the property of such person.

In this case, as Mr. X’s son are legal heir deciding to take 50% ownership by each of them, the assets
and liabilities of the concern shall be transferred from Mr. X to them on the date of death of Mr. X. The
cost of acquiring the various assets of Mr. X by his sons is deemed to be the market value of such assets
taken; and for the period after his death, his sons are responsible to pay tax on business after completion
of legal formality. As such, the act of the sons of Mr. X submitting the income return in the name of their
father even for the period after his death is not correct as per the Act. They should make an arrangement
to address the provision of the Act, and divide the property as agreed and pay tax on the business
conducted after Mr. X’s death separately.
Chapter 14- Disposal of Assets & Liabilities

Question 1- Page 146

If all the following conditions are satisfied, the Incomings and Outgoings for the disposed asset shall be
calculated as follows:

Conditions:
1. The asset shall be disposed by way of conditions mentioned in Sec. 40 (1)
2. The disposal shall be due to involuntary, i.e. without the will of disposer,
3. The person disposing the asset shall acquire similar type of asset within one year of the involuntary
disposal of asset, and
4. The disposer shall elect the provisions of Sec. 46 to be applied in writing

Incomings for the disposed Assets:


• If the disposal proceeds of the disposed assets is greater than the cost of acquiring similar other
asset within one year:
Outgoings of the disposed Assets
Plus: Incomings received at the time of involuntary disposal of asset
Less: Outgoings incurred to acquire similar other asset within one year of disposal
• If the disposal proceeds of the disposed assets is less than the cost of acquiring similar other asset
within one year:
Outgoings of Disposed Assets

Outgoings for the Disposed Assets:


• If the disposal proceeds of the disposed assets is greater than the cost of acquiring similar other
asset within one year:
Outgoings of the Disposed Assets
• If the disposal proceeds of the disposed assets is less than the cost of acquiring similar other asset
within one year:
Outgoings of the Disposed Assets
Plus: Outgoings incurred while acquiring similar other asset within one year of disposal
Less: Incomings from the involuntary disposal of asset

In the given question, since all the above conditions are satisfied, the calculation of incomings and
incomings of disposed assets shall be calculated as per Sec. 46, i.e. as per above:

Calculation of gain on Disposal

A. Outgoings of the Disposed Asset 5,000,000


B. Incomings on involuntary disposal of Asset 8,000,000
C. Outgoings while acquiring similar other asset within one year of disposal 6,000,000
D. Incomings of the Disposed Assets (as per Sec. 46) 7,000,000
Since, B>C, A+B-C, Else it would be value of “A”
E. Outgoings of the Disposed Asset (as per Sec. 46) 5,000,000
Since. B>C, Value of “A”, else it would be A+C-B
F. Gain on Disposal (D – E) 2,000,000

In case Mr. Ram failed to elect Sec. 46 in writing, the gain would be Rs. 8 Million less Rs. 5 Million (as
calculated u/s 37, 38 & 39 of the Act).

In case Mr. Ram could not acquire similar other asset within a year of involuntary disposal, the gain would
be Rs. 8 Million less Rs. 5 Million (as calculated u/s 37, 38 & 39 of the Act)
Chapter 15- Treatment of Donation

Question 1- Page 151

Refer Provision of Sec. 12Ka in Page 151 of the Book

Question 2- Page 151

As per Sec. 12 (1), in case a person makes donation or provides gifts to any Tax Exempt Organization
approved from Inland Revenue Department during any Income Year, the amount of such donation or gift
is deductible while calculating taxable income of such person.

As per Sec. 12 (2), the limit of such donation or gift shall not exceed the minimum of following two:

a. Rs. 100,000
b. 5% of Adjusted Taxable Income

However, Sec. 12 (3) has given an unconditional right to GON to increase the limit mentioned in sub-
section 2 of Sec. 12 to such extent as specified in a notification published for the purpose by GON in
Nepal Gazette.

As such, since Mr. Sumendra Pradhan is a person (as person includes both natural person and entity,
and Mr. Pradhan is covered by the definition of natural person), the donation to an exempt organization
by Mr. Pradhan is deductible subject to the provisions of Sec. 12 (2) and Sec. 12 (3) of the Act.

For this purpose, Mr. Pradhan shall submit income return (joint reading of Sec. 4 (3) and Sec. 97 (1))
Chapter 16- Miscellaneous Provisions for Characterization & Quantification

Question 1- Page 154

As per Sec. 31, in case a person or associated person of such person receives any compensation as
follows including that from an Insurance company, the amount so received shall be included in business,
employment or investment income as per the circumstances at the time of receipt of such payment:
• Compensation against income or an amount to be included in calculating income of the person from
an employment, business, or investment, which the person expects or expected to derive, or
• Compensation against a loss or an amount to be deducted in calculating income of the person from a
business or investment, which the person has incurred or which the person expects or expected to
incur

In the given case, compensation received against the loss of gold chain and diamond ring of Rs. 150,000
does not form part of income as it is not a business asset, nor a Non Business Chargeable Asset, nor a
Trading Stock, nor a Depreciable Asset.

Similarly as the compensation towards loss of stock of Rs. 700,000 is received against a loss or an
amount to be deducted in calculating income of the person from a business, which the person has
incurred or which the person expects or expected to incur; the amount shall be included in business
income of Ms. Thapa.

Likewise, as the compensation towards damage of machinery of Rs. 2,500,000 is received against a loss
or an amount to be deducted in calculating income of the person from a business, which the person has
incurred or which the person expects or expected to incur; the amount shall be included in business
income of Ms. Thapa. This amount shall be considered as disposal proceeds against the actual disposal
of depreciable asset.

Alternatively, as the loss results into involuntary disposal of assets, the provisions of Sec. 46 may be
applied at the option of taxpayer in case Ms. Thapa wishes to acquire trading stock or machinery within
one year of loss of stock or machinery.

Question 2- Page 154

As per Proviso Clause (1) to Sec. 31, In case a Resident natural person receives any compensation in
relation to the physical loss (bodily injury) of such person, the amount of compensation so received shall
not be included in income. As the compensation is not included in income, the amount incurred for the
treatment of such physical part shall not be claimed as Medical Tax Credit.

In the given scenario, the staff of M/s XYZ P. Ltd., a resident natural person receives compensation of Rs.
900,000 against his physical loss (bodily injury). Since the condition of Proviso (1) of Sec. 31 is satisfied,
the amount shall not be taxable in the hand of the staff. The staff cannot claim medical tax credit against
any approved medical expenditure incurred for the treatment of such injury.

Since the amount is not taxable in the hand of recipient, as per Sec. 88 (4), a person is not required to
withhold tax on the amount exempted from tax. As such, XYZ P. Ltd. shall not withhold tax while making
payment of compensation to its employee.

Question 3- Page 154

As per Sec. 30 of the Act, For the purpose of computing income from investment made jointly by a person
with another person, the amount to be included and expenses to be deducted shall be computed on the
basis of proportion of the investment made by such person under joint arrangement.
Chapter 16- Miscellaneous Provisions for Characterization & Quantification

In the given case, since Mr. Ram and Mr. Shyam jointly owned house property contributing certain sum;
the provision of Sec. 30 is inflicted. It requires apportionment of the incomings and outgoings in the
individual person’s gain calculation on the basis of proportion of investment held by each individual. The
proportion of investment is 1:4, as given in question.

Apportioned to Apportioned to
Particulars Total
Mr. Ram Mr. Shyam
Incomings from the disposal of Joint 64,000,000 12,800,000 51,200,000
Investment (10/50*64,000,000) (40/50*64,000,000)
Outgoings for the Asset 50,700,000 10,140,000 40,560,000
(Cost plus brokerage and other incidental cost (10/50*50,700,000) (40/50*50,700,000)
on disposal)
Gain 13,300,000 2,660,000 10,640,000

Question 4- Page 154

As per Sec. 30, for the purpose of computing income from investment made jointly by a person with
another person, the amount to be included and expenses to be deducted shall be computed on the basis
of proportion of the investment made by such person under joint arrangement.

In the given case, Ram and Krishna jointly invested by in land contributing Rs. 20 Laks and Rs. 30 Lakhs
respectively after deriving interest at 18% p.a. for six months from bank. As it is the case of joint
investment, provisions of Sec. 30 are applicable. It means, the incomings and outgoings related to the
investment shall be apportioned in the ratio of 2:3, investment proportion, and the gain shall be calculated
accordingly:

Apportionment of Incomings and Outgoings related to land and calculation of gain on disposal of each
individual:

Apportioned to Apportioned to
Particulars Total
Mr. Ram Mr. Krishna
Incomings from the disposal of Joint 8,600,000 3,440,000 5,160,000
Investment (2/5*8,600,000) (3/5*8,600,000)
Outgoings for the Asset 5,600,000 2,240,000 3,660,000
(Cost plus brokerage and other incidental cost (2/5*5,600,000) (3/5*5,600,000)
on acquiring the asset and on disposal)
Gain 3,000,000 1,200,000 1,800,000

The interest income is:

18% * 5,000,000*1/2= 450,000

The income will be apportioned to Mr. Ram and Mr. Krishna in the proportion of their investment, i.e. Rs.
180,000 and Rs. 270,000 respectively. Since, the joint investment account cannot be treated as natural
person’s interest income to the satisfaction of Sec. 88 (3), the bank needs to withhold tax at 15% on
interest amount, which will be apportioned each of them in their proportion of income and they need to
include interest income while calculating assessable income from investment.
Chapter 16- Miscellaneous Provisions for Characterization & Quantification

Question 5- Page 154

Mr. Govind and the bank is not associated person. When the market liquidity is abundant, the interest rate
on deposit is lesser than the circumstances when there is liquidity crisis. As such, loan interest rate is
determined by market factor and the ability of negotiation of borrower. In the situation of lower cost of
fund, the bank may reduce interest rate of loan. As such, the interests charged by bank to its borrower
are always on arm’s length. So, the tax officer contention to include the difference of Rs. 100,000 as
interest saving is not justifiable, and is against the normal market practice jeopardizing the right to obtain
competitive rate of interest in borrowing.

Question 6- Page 154

Refer Answer to Question No. 5 of Page 128 (Income from Employment) for the hint and answer.
Chapter 17- Set Off & Carry Forward of Losses

Clarification Added:

Page 156- Carry Backward of Losses

The readers should note that the carry backward is allowed only after obtaining permission of IRD. As
such, the carry backward must be performed as directed by IRD.

Question 1- Page 158

• Check out Sec. 20 (2), refer page 155 and 156 of the book

Question 2- Page 158

We need to test the type of asset, Mr. Praveen is possessing. Elephant is a biological asset, but Mr.
Praveen is not conducting any business of elephant and raising elephants just for fun. As such, the
elephant is neither business asset (see definition of business asset), nor depreciable asset, nor trading
stock, nor non business chargeable asset (NBCA includes land, building or securities or interest in any
entity and no other). It means: gain or loss on disposal of elephant is beyond the scope of Income Tax
Act. As such, he cannot set off notional loss on sale of elephant with any other income.

Question 3- Page 158

As per Sec. 20 (8), tax loss sustained by any person during the tax holiday period (i.e. when no corporate
tax is levied) cannot be carried forward for set off in following Income Years, which means carry forward is
not allowed only when the person is enjoying tax holiday; not the concessional rate of corporate tax. As
such, the loss sustained by a company situated at underdeveloped area and paying tax at concessional
rate is allowed to be carried forward.

Question 4- Page 158

Note that Singa Road Ltd. is not a project carrying on Public Infrastructure projects that will ultimately be
handed over to Government of Nepal, but a construction company carrying on government’s construction
work. As such, it is allowed for the carry forward of its business loss to be set off against investment
income or business income within next seven income years, not twelve income years.

(Had it been a public infrastructure project under BOT, the carry forward is allowed for next 12 years. The
distinguishing factor is person carrying on public infrastructure projects invest for construction of such
projects, generates revenue for certain years as agreed with Government and hand over the assets
including the infrastructure to GON after the expiry of specified years. The Government gets a project
without any cost, except that agreed in agreement)

On the basis of above, the taxable income for 2069/70 after set off of carried forward loss is:

Income for the Year 1,500,000


Less:
th
Loss Sustained for IY 62/63 (69/70 is 7 Year) (500,000)
Loss sustained for IY 63/64 (600,000)
Loss sustained for IY 64/65 (200,000)
Loss sustained for IY 65/66 (150,000)
Loss sustained for IY 66/67 (100,000)
Loss sustained for IY 67/68 (75,000)
Loss sustained for IY 68/69 (10,000) (1,635,000)
Chapter 17- Set Off & Carry Forward of Losses

Loss for the Year (135,000)

Since the Income is just Rs. 1,500,000 and claimable carried forward loss is Rs. 1,635,000; the loss yet to
be carried forward is Rs. 135,000. Through Income of IY 2069/70, loss up to IY 65/66 is completely set off
and loss of Rs. 50,000 pertaining to IY 2066/67 is yet to be set off, which can be set off against income till
2073/74.

As the company sustained loss till 2068/69, which is last year for the recovery of unrelieved loss
pertaining to IY 2061/62, the loss of 61/62 cannot be set off against any business or investment income.
However, it can be carried forward to be set off against any gain on disposal of business asset, business
liability or non business asset of investment in the future period(s) u/s 36 of the Act.

Question 5- Page 158

As per Sec. 20 (4), in case a person carrying on construction contract (long term contract)
through international competitive bidding (ICB) sustains loss in the year of disposal of
contract either through completion or otherwise, as a result of loss sustained during the period or due
to set off of carried forward loss related to same contract, the person may carry back such losses
to be set off against the income of same contract after obtaining permission from IRD.

In the given case, as the loss calculated as per Sec. 26 during the year of completion of contract is Rs.
800,000 and the profit reported for Year I and Year II under the same contract as per Sec. 26 is Rs.
500,000 and Rs. 200,000 respectively; and the contract is obtained through ICB, ABC Ltd. may carry
rd
back its loss of 3 Year to be set off against the Income under the same contract, i.e. Rs. 800,000 of loss
can be set off against the taxable profit of Year I and Year II, however, the set off is allowed only to the
extent of available profit under the same contract, i.e. the loss of Year III to the extent of Rs. 700,000 can
only be carried back for set off. The company’s intent to obtain IRD’s permission will make it able to carry
back the loss as aforesaid. The remaining Rs. 100,000 can be carried forward to be set off as per Sec. 20
(1) and Sec. 20 (2).

Had it not an International bidding, one of the basic criteria, i.e. the contract to be obtained from
International competitive bidding would not be fulfilled; as such, the loss cannot be carried back as per
Sec. 20 (4). The loss of Rs. 800,000 may, however, be carried forward to be set off against business or
investment income to be generated in next seven income year(s).

Question 6- Page 158

No. As per Sec. 20 (1), in case a person has taxable income from any business during any income year,
the income can be applied for set off of loss as follows:

a. Loss from any other business of the same person during the same Income Year, or
b. Any unrelieved business loss of the same persons carried forward since last seven Income Years
(s).
The provision of carry forward is extended to 12 Years for entities involved in construction of
power houses, generation and transmission of electricity; public infrastructure projects that are
ultimately handed over to GON or persons carrying on Petroleum transactions under Petroleum
Act, 2040.

Through this provision, the Act has not envisioned the set off of investment loss from business income.
As such, investment loss of Rs. 50,000 of Mr. Shyam cannot be set off against business income of Rs.
120,000.
Chapter 17- Set Off & Carry Forward of Losses

Similarly, as Sec. 20 (3), foreign sourced losses can only be set off against foreign source income subject
to provisions of Sec. 20 (1) and Sec. 20 (2). As such, foreign source business loss of Rs. 24,000 cannot
be set off against Nepal Sourced business income of Rs. 200,000.
Chapter 18- Installment Tax & Estimated Tax Return

Question 1- Page 161

As per Sec. 94, the tax shall be paid in advance as follows:

Up to Poush end: 40% of Total Estimated Tax Liability

Up to Chaitra end: Remaining Tax Liability to make it 70% of Total Estimated Tax Liability

Up to Ashad end: Remaining Amount to make it 100% of Total Estimated Tax liability

As such,
Up to Poush 20X1: 40% of Estimated Tax liability = Rs. 400,000; of which Rs. 20,000 is already paid out
of tax withheld by agents- so amounts to be deposited = Rs. 380,000

Up to Chaitra end 20X1: Make total payments covering 70% if Estimated Tax liability= Rs. 840,000; of
which Rs. 380,000 is paid in first installment in Poush, Rs. 20,000 as WHT up to Poush and Rs. 10,000
as WHT from Magh to Chaitra; so additional amount to be deposited is Rs. 430,000

Up to Ashad end 20X2: Make total payments covering 100% of Estimated Tax Liability= Rs. 1,200,000, of
which Rs. 840,000 has been paid by way of WHT and advance tax up to Chaitra 20X1 and Rs. 15,000
from Baisakh to Ashad by way of WHT from agents. The additional amount to be paid is Rs. 345,000.

Question 2- Page 161

a. Person receiving amounts after receiving withholding tax does not mean all payments are final
withholding. The person having less than Rs. 5,000 tax liability during any year is not required to pay
installment tax. So far as the person has tax liability exceeding Rs. 5,000 during the Year, the person
is required to pay tax in installment no matter whether it’s all payments are made of payments
attracting WHT. The tax withheld by the agent can also be considered as part of installment if it has
been deposited by agent. They are required to file estimated tax return.
b. In case a person having income only from remuneration, the resident employers are required to
calculate tax as per Sec. 87; and the Act has passed on the responsibility to calculate tax to the
employers, as such person deriving remuneration income from resident employers are not required to
calculate tax and pay installment tax. However, in case of nonresident employer, installment tax shall
be paid by the person deriving such income. The person requiring payment of installment tax are
required to submit estimated tax return.
Chapter 19- Final Tax & Final Tax Return

Question 1- Page 167

a. For tax withheld or deemed to be withheld for every month, within 25 days of the end of the month.
b. Within the deadline as specified in Sec. 96 (1)
c. Within the due date as specified in the Assessment Order issued u/s 102
d. Within the due date as specified in the Assessment Order issued u/s 102
e. Refer Page 162 of the book.

Question 2- Page 167

Refer Page 166 of the Book (heading: Time Limit to File Income Tax Return and Extension of Submission
Deadline)

Question 3- Page 167

Write the provisions of Sec. 4 (3) of the Act; refer Page 166 of the book.

Question 4- Page 167

The due date of filing tax return as per Sec. 96 (1) is within three months from the end of Fiscal Year. As
per Sec. 57 (3), in case of change in control the entity shall treat a Year as two separate Income years,
one beginning from Shrawan 1 to date of change of control and the other from the date of change of
control to Ashad end of next year. As such, the income year ends on the date of change in control.
Hence, the person shall submit tax return within 3 months from 2068.11.10 in case of failure to extend
deadline as per Sec. 98.

Similarly the amended assessment shall be completed within 4 years from the due date of filing tax return
i.e. four years from 2069.2.10.
Chapter 20- Tax Assessment

Question 1- Page 170

Refer Issuance of Assessment Order Section of Book in Page 170.

Question 2- Page 170

Sec. 100 of the Act- complete (Page 168 and 169 of the Book)
Chapter 21- Entity & Distribution

Question 1- Page 174

As per Sec. 54 (3), in case a resident entity distributes dividend out of the amount on which distribution
tax as per Sec. 54 (1) is already levied, there shall be no further distribution tax.

In the given case, Mr. Ram has received dividend from M/s Balgopal Ltd. which is a resident entity.
Assuming A One Ltd. a resident company, the dividend distributed by Balgopal Ltd. is from the profit out
of dividend received from A One Ltd. which levied dividend tax to Balgopal Ltd. while distributing the
dividend. As such, as per the principle of Sec. 54 (3), the receipt of dividend by Mr. Ram is exempted
from Tax. As per Sec. 88 (4), an agent cannot withhold tax on amounts which are exempted from tax, as
such; Balgopal Ltd. is not required to withhold tax on distribution of dividend.

In case Balgopal Ltd. is a nonresident company, as per Sec. 54 (2), the dividend received by Mr. Ram
shall be included in income.

Yes, in case Balgopal Ltd. is a Cooperative based on rural area defined by Sec. 11 (2) of the Act, there is
no need of application of withholding of tax and if not so, tax shall be withheld at normal rate.

In case the company being an industry based on agro products or forest products, so far as the entity is
resident partnership or company, tax shall be withheld at 5%.

Question 2- Page 174

As per Sec. 54 (1), the dividend distributed by a resident partnership firm is taxable in the hands of
recipient on final withholding basis.

As per Sec. 54 (3), in case a resident entity distributes dividend out of the amount on which distribution
tax as per Sec. 54 (1) is already levied, there shall be no further distribution tax. Since, in case the
partnership firm’s income is only out of dividend received from resident company, the resident company
has already levied dividend tax on such amount; as such, there shall be no dividend tax on re-distribution
of such profit by partnership firm to its partners.

Question 3- Page 174

The definition of distribution includes:

a. Payment of any amount in any capacity by an entity to its beneficiaries, while such payment is not
distribution in case it satisfies both the following conditions:
1. In case the payment exceeds the value of consideration received by the entity from the
beneficiary, and
2. In case the payment, exceeding the value of consideration received by the entity from the
beneficiary, does not include the following amounts:
a. Amounts included while calculating taxable income of the beneficiary, or
b. Final withholding payments, except when it is final withholding as a result of distribution
b. Capitalization of Profit

Issuance of bonus share out of share premium is not payment to the beneficiaries. As such, we need to
test whether such issue of bonus share out of share premium constitutes Capitalization of Profit.

As per the clarification Clause to Sec. 53, Capitalization of Profit includes:

a. Capitalization by way of issuing bonus shares or any other similar interest,


Chapter 21- Entity & Distribution

b. Increase in Paid up value of the interest of any entity, or


c. Capitalization of profit and premium of the entity in the capital account of the company

Since, definition (c) covers Capitalization of Premium as “Capitalization of Profit”, issuance of bonus share
out of share premium satisfies the test of Capitalization Profit, implying that it is distribution.

As per Sec. 53 (4), the capitalization of profit shall be treated as part of distribution of profit. Similarly,
Sec. 53 (3) requires any distribution to be treated as “Capital Repayment” or “Distribution of Profit” only
when such distribution decreases the value of asset or liability of the company.

As Sec. 53 (4) clearly states that Capitalization of Profit is part of Distribution of Profit, the contention of
tax officer is correct.

Question 4- Page 174

As per Sec. 54:

1. In case of dividend paid by a resident Entity, the tax shall be levied as follows:
a. In case a company distributes dividend to its shareholder or a partnership firm distributes
dividend to its partner, dividend tax shall be levied as per final withholding basis.
b. In case the dividend is paid by any other entity, no dividend tax is levied
2. In case of payment of dividend by Nonresident person, the dividend so received by a resident person
shall be included while calculating taxable income of such person.
3. There shall be no dividend tax while redistributing the amount received as dividend after levying
dividend tax as per (1) above.

It means, there shall be no further dividend tax while distributing the amount received as dividend after
levying dividend tax; i.e. out of Rs. 5 Laks received by Mr. Bibhu, there shall be no dividend tax on such
amount which has been distributed out of profit where dividend tax is already levied.

The amount on which dividend tax is levied= 90/100*500,000= 450,000

Dividend Tax = 5% of 450,000= Rs. 22,500

Question 5- Page 174

Similar as Question (4), in this case total dividend tax to be withheld by company is to be computed.

Total Profit of A Ltd.: Rs. 400,000 plus (200,000/0.85) = Rs. 635,294

Dividend Declared by A. Ltd.: Rs. 450,000

Amount on which dividend tax is levied: 235294/635294*450,000 = Rs. 166,667

Dividend Tax = 5% of Rs. 166,667 = Rs. 8,333.33


Chapter 22- Change in Control & Tax Treatment

Question 1- Page 179

As per Sec. 57 (2) (b), an entity cannot set off carried forward loss pertaining to Period before change in
Control from business/investment income of the same entity generated after the period of Change in
Control as per Sec. 20

An entity’s control is deemed to be changed when 50% or more of its underlying ownership is changed in
comparison to its ownership structure three years previously. While determining the change in control,
only such changes as attributable to the beneficiaries holding 1% or more interest in the entity or the
associated person of the beneficiaries holding 1% or more interest in the entity shall only be considered.

In the given case, M/s Ganapati Industries P. Ltd. is being operated since last many years as the question
provides information of the loss of last four years with Mr. Shyam as sole shareholder of the company,
and Mr. Shyam sold 60% of the company’s stake to Mr. Mohan on Ashad 31, 2069.

As Mr. Shyam is the only shareholder (with 100% ownership), all change in control, i.e. 60%, shall be
considered; which means there is change in control on Ashad 31, 2069.

This means, as aforesaid in the first paragraph, the loss cannot be carried forward for set off after the
change in control. As such, the tax officer’s contention is correct.
Chapter 23- Long Term Contract

Question 1- Page 183

Calculation of Profit for each Year

(In ‘000)

Particulars 20X1/X2 20X2/X3 20X3/X4


A. Total Revenue of the Contract 4,000,000 4,070,000 4,150,000
B. Total Actual Cost to Date 1,800,000 2,628,000 4,020,000
C. Estimated Cost to Completion 3,600,000 3,650,000 N/A
D. Percentage Completion (B/C) 50% 72% 100%
E. Revenue to Date (A*D) 2,000,000 2,930,400 4,150,000
F. Profit to Date (E – B) 200,000 302,400 130,000
G. Profit up to Previous Year - 200,000 302,400
H. Profit/(Loss) for the Year (F – G) 200,000 102,400 (172,400)
Treatment of Loss on Completion of Contract:

As per Sec. 20 (4), in case a person carrying on construction contract (long term contract)
through international competitive bidding (ICB) sustains loss in the year of disposal of
contract either through completion or otherwise, as a result of loss sustained during the period or due
to set off of carried forward loss related to same contract, the person may carry back such losses
to be set off against the income of same contract after obtaining permission from IRD.

As such, loss of Rs. 172,400 can be carried back to be set off against taxable income of 20X1/X2 or
20X2/X3 in case the permission thereof from IRD is obtained; as it is a long term contract obtained
through ICB and the loss is sustained at the completion of contract.

Working Notes:

1. Revenue to Completion for Year 2= Initially Agreed Revenue plus Variation Order for Year 2
2. Revenue to Completion for Year 3 = Initially Agreed Revenue Plus Additional Revenue Agreed in all
Variation Orders
3. Estimated Expenditure to Completion for Year 2 is increased
4. For the year of Contract completion, since contract is 100% completion, estimated cost to completion
is no relevant.
5. Rectification work also forms part of contract cost and as it already been incurred, it’s been included
in expenses.

Question 2- Page 183

Calculation of Profit for each Year

Particulars 2066/67 2067/68 2068/69


A. Total Revenue of the Contract 1,500,000 1,500,000 1,526,250
B. Total Actual Cost to Date 378,000 848,000 1,331,000
C. Percentage Completion (Given) 30% 65% 100%
D. Revenue to Date (A*D) 450,000 975,000 1,526,250
E. Profit to Date (E – B) 72,000 127,000 195,250
F. Profit up to Previous Year - 72,000 127,000
G. Profit/(Loss) for the Year (F – G) 72,000 55,000 68,250
Working Notes:

1. Revenue for third Year:


Chapter 23- Long Term Contract

Since the market price of the material and wages go up by 7% (which is less than 10%) in 2066/67,
the escalation clause shall not be effective for the year. As such, revenue of the contract till the end of
2067/68 does not change.

However, the market prices of material and wages have gone up by 14% at the end of 2067/68. As
per the Contract terms, if the price of material and wages go up by more than 10%, the remaining
contract price will increase by 5%.

Since, the price increase is identified only at the end of 2067/68; the remaining contract revenue
(revenue for third year) shall be increased by 5%.

Remaining Contract Revenue = 1,500,000 * 35% (as 65% is already completed and remaining is
35%) = Rs. 525,000

Escalation Clause = Rs. 525,000 * 5% = Rs. 26,250

Total Revenue = Initially Agreed Revenue + Escalation Clause = 1,500,000 + 26,250 = 1,526,250

Part (b) of the question:


Refer Page 180 and 181 of the book

Question 3- Page 183

Calculation of Profit for each Year

(In ‘000)

Particulars Year I Year II Year III


A. Total Revenue of the Contract 60,000,000,000 60,000,000,000 60,000,000,000
B. Total Actual Cost to Date 10,000,000,000 25,000,000,000 40,000,000,000
C. Estimated Cost to Completion 40,000,000,000 40,000,000,000 40,000,000,000
D. Percentage Completion (B/C) 25% 62.5% 100%
E. Revenue to Date (A*D) 15,000,000,000 37,500,000,000 60,000,000,000
F. Profit to Date (E – B) 5,000,000,000 12,500,000,000 20,000,000,000
G. Profit up to Previous Year - 5,000,000,000 12,500,000,000
H. Profit/(Loss) for the Year (F – G) 5,000,000,000 7,500,000,000 7,500,000,000

Refer book for the definitions.


Chapter 24: Finance Lease

Question 1: Page 185

Cite Sec. 32 of the Act, with Sec. 32 (5) being the conditions for treatment as Finance Lease

Question 2: Page 185

It is assumed that the interest rate agreed is 15% p.a. Please note that the Act has not specified any
interest rate to quantify the interest and principal repayment, as Sec. 32 (3) and Sec. 32 (4) requires
identifying a semiannually compounded interest rate to re-characterize the lease payments into principal
and interest portion when no interest rate is agreed.

The general interest rate specified in Sec. 32 (8) is to discount the Lease Payments to arrive at Present
Value to test the condition of Sec. 32 (5) (d).

Five Installments- beginning of the Year means: Four installments at the end of the Year

Remaining
Total Principal as
End Loan
Installment including on Beginning
of Amount at Interest (C-D)
(B) Interest (C = of Year (D=
Year the Yearend
A+B) C/1.15)
(A)
IV - 1,000,000.00 1,000,000.00 869,565.22 130,434.78
III 869,565.22 1,000,000.00 1,869,565.22 1,625,708.88 243,856.33
II 1,625,708.88 1,000,000.00 2,625,708.88 2,283,225.12 342,483.77
I 2,283,225.12 1,000,000.00 3,283,225.12 2,854,978.36 428,246.75
0 2,854,978.36 1,000,000.00 3,854,978.36
Chapter 25- Income Tax in BFIs and Insurance Companies

Question 1- Page 188

As per Sec. 59 (1Kha), cooperatives are allowed to claim as expenses an amount set aside in risk
bearing fund, which is equivalent to 5% of Loan Outstanding, and not 15%. As such, the statement is
incorrect.

Question 2 & 3- Page 188

Refer Page 187- Treatment of Loan Loss Provision

Question 4- Page 188

Errors Rectified: Total Loan Loss Provision for Year I: Rs. 1,800 instead of Rs. 18,000.

The following calculation is based on the method as specified by Income Tax Manual 2066 (Updated
2068), multiple solution prevail.

Particulars X1/X2 X2/X3 X3/X4 X4/X5 X5/X6 X6/X7


A. Loan Outstanding 30,000 35,000 45,000 55,000 70,000 80,000
B. 5% of A 1,500 1,750 2,250 2,750 3,500 4,000
C. Total Loan Loss Provision to Date 1,800 1,575 2,070 3,300 3,220 5,600
D. Claimable Loan Loss Provision till this
Year (Minimum of “B” or “C”) 1,500 1,575 2,070 2,750 3,220 4,000
E. Claim up to Previous Year - 1,500 1,575 2,070 2,750 3,220
F. Eligible for the Year (D – E) 1,500 75 495 680 470 780
Chapter 25- Income Tax in BFIs and Insurance Companies

Question 1- Page 191

Calculation of Assessable Income from Investment Insurance Business

Sec. Refer
Particulars Amounts
Ref. W. N.
Inclusions:
Insurance Premium Receipts 61.2 1 -
Commission received against reinsurance ceded 7.2 30,000
Interest Income on Investments directly related to business 7.2 3,000,000
Other Miscellaneous Income 7.2 60,000
Total Inclusions 3,090,000
Allowable Deductions:
Commission Paid against Re-insurance Ceded 13 20,000
Amount paid against policy surrendered 2 -
Insurance Claim paid on Maturity 61.2 3
Insurance Claim paid on Death 61.2 3
Commission Expenses 13 200,000
Management Expenses 13 1,000,000
Medical Fee 13 50,000
Depreciation Expenses 19 200,000
Total Deductions 1,470,000
Assessable Income from Business 1,620,000

Calculate rest of the part on yourself

Working Notes:

1. The insurance premium including premium on re-insurance ceded is not required to be included in
income of the person (Sec. 61 (2) (ka))
2. It is part of Capital repayment, as such, it is not expenses or income.
3. These do not form part of expenses.
Chapter 27- Income of Nonresident Air & Water Transport Operator and Telecommunication
Operator

Question 1- Page 198

As per Sec. 70 (1), the income of a nonresident (a resident entity, the head office of which is outside
Nepal) operating air, water transportation or chartered service shall include the following, except as a
result of transshipment:

a. Transportation of passengers who disembark from Nepal, and


b. Transportation of goods, mail or livestock who disembark from Nepal

Similarly, as per Sec. 70 (2), the income of a nonresident (a resident entity, the head office of which is
outside Nepal) operating telecommunication service includes any amount received by the person in
respect of the transmission of messages by apparatus established in Nepal; whether or not such
message originate in Nepal.

Based on the above facts:

a. Sec. 70 is applicable on the payment of air-ticket for departure from Nepal, as it is related to
Transportation of passengers who disembark from Nepal and not as result of transshipment.
b. Sec. 70 is not applicable as it is not for Transportation of passengers who disembark from Nepal, and
Transportation of goods, mail or livestock who disembark from Nepal.
However, restrictive clause of Sec. 2 (7) of Schedule 1 is applicable which requires payment of 2%
ticket money as corporate tax when tickets are purchased/ sold in Nepal.
c. As it is result of transshipment, Sec. 70 is not applicable.
d. Same as (c), since it is as a result of transshipment, Sec. 70 is not applicable.

Question 2- Page 198

Since, Nepal Airlines has head office in Nepal; the income of Nepal Airlines is subject to tax as per
normal other provisions of the Act and not u/s 70.

As per Sec. 70 (1), the income of a nonresident (a resident entity, the head office of which is outside
Nepal) operating air, water transportation or chartered service shall include the following, except as a
result of transshipment:

a. Transportation of passengers who disembark from Nepal, and


b. Transportation of goods, mail or livestock who disembark from Nepal

Since, the foreign Airline does not operate any flights from Kathmandu, Sec. 70 is not applicable.

However, as per restrictive clause of Sec. 2 (7) of Schedule 1 of the Act; in case when the
disembarkment is not from Nepal and from any country outside Nepal but the airlines sells ticket in Nepal,
2% tax is applicable. As such, 2% of 800,000 (from Delhi to other locations) shall be the corporate tax
liability.

Question 3- Page 198

As per Sec. 70 (1), the income of a nonresident (a resident entity, the head office of which is outside
Nepal) operating air, water transportation or chartered service shall include the following, except as a
result of transshipment:

a. Transportation of passengers who disembark from Nepal, and


b. Transportation of goods, mail or livestock who disembark from Nepal
Chapter 27- Income of Nonresident Air & Water Transport Operator and Telecommunication
Operator

Since Den Airlines sold air tickets from the passengers departing from Nepal of Rs. 500 Million, as per
Sec. 70 (1) (a) explained above, the amount is taxable. The tax rate is 5%. No expenses related to any
flight are deductible.

As the collection of Rs. 100 Million is for the passengers departing from other countries; Sec. 70 is not
applicable. However, as per restrictive clause of Sec. 2 (7) of Schedule 1 of the Act; in case when the
disembarkment is not from Nepal and from any country outside Nepal but the airlines sells ticket in Nepal,
2% tax is applicable.

Question 4- Page 198

As per Sec. 70 (2), the income of a nonresident (a resident entity, the head office of which is outside
Nepal) operating telecommunication service includes any amount received by the person in respect of the
transmission of messages by apparatus established in Nepal; whether or not such message originate in
Nepal.

As Singtel Ltd. has a communication hub in Nepal, the information is transmitted through apparatus
established in Nepal. Any amount received in that respect is taxable in Nepal. As such, USD 1 Million is
taxable in Nepal.

Question 5- Page 198

Calculation of Tax Liability as per Sec. 70 of the Act:

Particulars Refer Amount


W. N.
Amount received from Sale of air tickets to passengers disembarking from 1.1 10,000,000
Nepal
Amount received for goods to be transported from Nepal, Kathmandu being 1.1 5,000,000
first point of sending goods
Amount received from passengers from Bhutan & Lhasa, Kathmandu being 2 -
transit
Amount received for goods sent from Bhutan & Lhasa 2
Taxable Amount (u/s 70) 15,000,000
Tax Liability @ 5% 750,000
Working Notes:

1. Section 70 is applicable on the following:


• As per Sec. 70 (1), the income of a nonresident (a resident entity, the head office of which is
outside Nepal) operating air, water transportation or chartered service shall include the following,
except as a result of transshipment:
a. Transportation of passengers who disembark from Nepal, and
b. Transportation of goods, mail or livestock who disembark from Nepal
• Similarly, as per Sec. 70 (2), the income of a nonresident (a resident entity, the head office of
which is outside Nepal) operating telecommunication service includes any amount received by
the person in respect of the transmission of messages by apparatus established in Nepal;
whether or not such message originate in Nepal.

As such, amounts received from operation of restaurant and ground handling service is not
chargeable to tax as per Sec. 70, but is chargeable to tax as per other provisions of the Act, i.e. as
applicable to normal entity.
Chapter 27- Income of Nonresident Air & Water Transport Operator and Telecommunication
Operator

1.1. Since it is for the disembarkment of passengers from Nepal, the amount is taxable.
1.2. Since it is for the disembarkment of goods from Nepal, the amount is taxable.
2. As explained in Working Note (1) above, in case of disembarkment from Nepal as a result of
transshipment of goods or services, Sec. 70 is not applicable. In such cases, the amount is not
taxable in Nepal.
3. As per Proviso to Sec. 70, any expense related to operation of air transport, water transport or
chartered services, or telecommunication services by a nonresident person is not deductible in Nepal.
Chapter 28- International Taxation including Transfer Pricing Arrangements

Question 1- Page 203

Calculation of Tax Liability of Permanent Establishment:

Income 5,000,000
Less: Expenses (4,000,000)
Taxable Income 1,000,000
Tax Liability @ 30% (as banking business) 300,000
Net Profit after Tax 700,000

The net profit after tax of Rs. 700,000 is repatriated to USA, after applicable repatriation tax. As per Sec.
3 (b), the PE in Nepal is responsible for payment of repatriation tax which is 5% of amount repatriated. As
such, Repatriation Tax:

Repatriated Amount + 5% of Repatriated Amount = Total Profit available after Corporate tax

105% of Repatriated Amount = Rs. 700,000

Repatriated Amount = 700,000/105% = Rs. 666,667

Repatriation Tax = Rs. 33,333

Note:

The Income Tax Manual has computed such amount as 5% of 700,000.


Chapter 28- International Taxation including Transfer Pricing Arrangements

Question 1- Page 212

As per Article 20 (1) of DTAA signed between Nepal and China; remuneration which an individual who is
or was, immediately before visiting a Contracting State (China), a resident of other Contracting State
(Nepal) and who is present in first-mentioned country (China) for the purpose of teaching, giving lectures
or conducting research at a university, college, school or educational institution or scientific research
institution recognized by Government of the first-mentioned country (China) derives for the purpose
such teaching, lectures, or research shall not be taxed in first-mentioned country (China), for a period of
two years from the date of his first arrival in first-mentioned country (China).

As per sub-article 2 of Article 20, the provisions of article 20 (1) shall not be applied to income for
research, if undertaken not in the public interest but primarily for the private benefit of a specific person or
persons.

Since,

You might also like